Property

Pataasin ang iyong marka sa homework at exams ngayon gamit ang Quizwiz!

Jana applies for a patent on a method of making a ham sandwich. The heart of Jana's technique is to apply mustard to both sides of the ham slice before placing it between two pieces of bread; thus, the mustard is applied to the ham, not to the bread. The PTO issues the patent. Two years later, Jana notices that her method is being used to make ham sandwiches in DDD, a famous New York deli. Jana sues DDD. Who will win the lawsuit?

A DDD, under the fair use defense.Incorrect. Jana's claim is, at best, a patent claim. Patent law does not recognize the fair use defense. B DDD, because Jana's procedure does not qualify for a patent.Correct. Jana's procedure does not qualify for a patent because it is obvious. C Jana, based on literal infringement.Incorrect. Jana's procedure does not qualify for a patent because it is obvious. Therefore, anyone may copy it without liability. D Jana, under the doctrine of equivalents.Incorrect. Jana's procedure does not qualify for a patent because it is obvious. Therefore, anyone may copy it without liability.

Which of the following is not protected by copyright?

A Literary WorksIncorrect. Protected. B Musical WorksIncorrect. Protected. C IdeasCorrect. Ideas can't be copyrighted because then you would monopolize the market if you granted such broad protection. Section 102(b) of the Copyright Act expressly precludes copyright protection for ideas, stating: "In no case does copyright protection for an original work of authorship extend to any idea, procedure, process, system, method of operation, concept, principle, or discovery, regardless of the form in which it is described, explained, illustrated, or embodied in such work." 17 U.S.C. §102(b). D Architectural Works.Incorrect. Protected.

Frank owned a 50-acre farm in an agricultural region which he used to grow carrots, lettuce, and other crops for many years during the summer growing season. All the properties in the region were used to grow similar crops. Frank decided to retire and give his land to his nephew Ned and Ned's wife Wilma. As a gift, Frank executed and delivered a deed that purported to convey title to the farm to Ned and Wilma as joint tenants. Frank, Ned, and Wilma were not aware that the deed was legally defective, and that Frank accordingly still owned the farm. Ned, Wilma, and their three children visited the land almost every day each summer for six years. They often invited Frank to join them on their trips to the land, and he frequently did. On each visit, they would have a picnic, hike on the land, watch birds, and generally enjoy being outdoors. Seven years after the deed was created, Frank's attorney found it while reviewing records for a lawsuit. When the attorney mentioned to Frank that the deed was a nullity, Frank exclaimed: "Well, I'm going to take my farm back now. Ned and Wilma were never grateful for my gift." The adverse possession period in the jurisdiction is five years. Who owns the land?

A Ned and Wilma, because all adverse possession elements are satisfied.Incorrect. The actual possession element is not satisfied because a reasonable owner of such land would use it as a farm, which Ned and Wilma did not do. Instead, they used it only for recreation, which is a less intensive use. B Frank, because Ned and Wilma did not have actual possession.Correct. The actual possession element requires that the claimant physically use the land in the manner that a reasonable owner would, given its character, location, and nature. The land at issue is a farm, located in a farming region. The reasonable owner of such a farm would use it to grow crops as Frank did. Ned and Wilma could have satisfied the actual possession element if they had used the land as a farm or if they had used it for a more intensive purpose (e.g., operating a factory). But Ned and Wilma used the farm only for recreation, which is a much less intensive use than farming. Most courts would accordingly hold that they did not have actual possession. C Frank, because Ned and Wilma did not have adverse and hostile possession.Incorrect. True, Frank, Ned, and Wilma all believed in good faith that Ned and Wilma owned the land. But it is not necessary for Ned and Wilma to have a subjective intent to wrest title from the true owner. Because they held possession in good faith reliance on the invalid deed, they satisfy the good faith test; and they would also satisfy the objective test but for the lack of actual possession. D Frank, because Ned and Wilma did not have exclusive possession.Incorrect. Frank's occasional presence on the land did not interrupt the exclusive possession held by Ned and Wilma for two reasons. First, he entered the land at their invitation and as their guest, not as a competing title claimant. Second, he did not use the land for farming, which is how a reasonable owner would use such land.

Which is NOT a requirement of a trade secret?

A The information of a trade secret must have actual or potential economic value.Incorrect. This is a requirement. B The information of a trade secret must be impossible to reverse engineer.Correct. Answer B is correct, because it is not an element required to establish a trade secret. The three elements of a trade secret are (1) A trade secret is information that has either actual or potential independent economic value by virtue of not being generally known, (2) a trade secret has value to others who cannot legitimately obtain the information, and (3) is subject to reasonable efforts to maintain its secrecy. It is not necessary to establish that a trade secret is impossible to reverse engineer. Reverse engineering is one of the affirmative defenses against trade secret infringement. C The trade secret has value to others who cannot legitimately obtain the information.Incorrect. This is a requirement. D Reasonable efforts must be made to maintain secrecy.Incorrect. This is a requirement.

Larry owns a delivery serivce called SUNFLOWER MAILINGS and uses the mark in interstate commerce sufficient to get trademark rights. A year later, Annie opens a hardware store, using the mark SUNFLOWER NAILINGS as the name. What factors would NOT weigh in favor of a likelihood of confusion?

A There is a strong similarity between the marks.Incorrect. Strong similiarity in sight, sound, etc suggest likeliihood of confusion. B Annie and Larry operate their businesses in different fields.Correct. The fact that they operate in different fields suggest lack of confusion. C Annie acted in good faith in adopting the name as she did not know of Larry's business.Incorrect. Good faith in selecting the mark suggests a lack of likelihood of confusion. D Larry and Annie both operate their stores in Buffalo. Incorrect. The fact that both stores are in the same geographic area suggests a likelihood of confusion.

Lindsay is blind. She likes to buy goods on Bestbuy.com but the website is not accessible to her using her screen reader. The year is 1999. Does Lindsay have a cognizable claim under the Americans with Disabilities Act?

A Yes, Amazon must make its website accessible to the blind under the ADA regardless of whether or not it impeded her access to info and services connected to the store.Incorrect. See Natl Federation for the Blind v. Target where the court ruled that an online retailer is not a public accomodation under the ADA. B Perhaps, although Bestbuy.com is not considered a "public accomodation" under the ADA, Lindsay could have a claim to the extent the inaccessibility of the website impeded her full and equal enjoyment of the goods and services of Best Buy.Correct. This is the facts of the Natl Fed. of the Blind v. Target.com case but substituting in Bestbuy.com Online retailers are not considered public accommodations under the ADA divorced from their physical stores. C No, Lindsay has no claim at all, except to the extent state law may provide a cause of action.Incorrect. See Natl Fed. for Blind v. Target.com. That case provided a limited remedy. D No, although Bestbuy.com is considered a public accommodation under the ADA, Lindsay won't be able to show that she could not just go to the store and buy it.Incorrect. Online retailers are not public accommodations under the ADA; Lindsay could still possibly make out a case like the plaintiff in Natl Fed. of the Blind v. Target.com.

Bonnie Beater has created a computer program that recommends a monthly amount that a person should save for retirement based on income and monthly expenses. Can she get a patent?

A Yes, provided it is novel and nonobvious.Incorrect. This isn't patentable subject matter under Alice. B Yes, provided she invented it before anyone else.Incorrect. This isn't patentable subject matter. C No, because it is both unpatentable subject matter and obvious.Correct. Under Alice, this is an abstract idea that is unpatentable as well as obvious. There is no inventive concept because the method is well-known. D No, because no one can get a patent on a software program.Incorrect. Software patents are not per se excluded. One can get a patent on software so long as it satisfies the Mayo/Alice test and show that is not abstract and has some inventive concept.

Evan owns an apartment in a high rise condominium. His most valued possession, a Picasso painting, hangs on his living room wall. Evan is currently visiting a distant city and has met Juan, his closest friend. A few years earlier, Juan had graciously helped Evan when he was down on his luck. Evan feels greatly indebted to Juan for his past kindness and is thinking about making a gift of his Picasso painting to him. Which of the following statements (made by Evan) would result in a valid inter vivos gift of the painting?

A "Juan, I want you to have my Picasso painting when I die."Incorrect. A valid inter vivos gift requires the immediate transfer of property rights from the donor to the donee. Here the painting is only intended to be transferred when Evan dies. A testamentary gift requires a valid will. B "Juan, I am pretty sure I want you to have my Picasso painting."Incorrect. Evan's statement demonstrates no finality of intent, only that he is thinking about transferring his rights to the painting. For a valid inter vivos gift, the donor must intend to make an immediate transfer of his property rights. C "Juan, I want you to have my Picasso painting now, but I want to keep it until I die."Correct. Evan intends to give Juan an interest in the painting now. Juan gains a vested remainder in the painting (a future interest -remember all future interests are present interests, present possession is merely delayed until some future date) and Evan retains a life estate in the painting. Because Evan intends to make an immediate transfer of property (a vested remainder), a valid inter vivos gift occurs. D "Juan, I have written my will and it provides you get my Picasso painting immediately when I die."Incorrect. A will is legally operative only upon the testator's death. Wills are ambulatory documents that can be amended or revoked at any time prior to the testator's death. Because Evan has no intent to make an immediate transfer of property, no gift is made.

Catalina owned Desertacre in 2000 when Mira took possession. Catalina was mentally incompetent at the time Mira began her occupancy. In 2002, Catalina died intestate never having regained mental capacity. Her sole heir was her brother Felipe who was also mentally incompetent. Felipe regained mental competency in 2020. The statutory period in this jurisdiction in ten years. A statute in this jurisdiction also provides: "If a person entitled to bring an action in adverse possession is, at the time the cause of action accrues, within the age of minority or of unsound mind, the person, after the expiration of ten years from the time of the cause of action accrues, may bring the action within five years after the disability is removed." When is the earliest that Mira can perfect title by adverse possession?

A 2007Incorrect. Although Mira's disability ended upon her death (2002) and the statute provides for a five year additional period following the disability's removal (2007), an adverse possessor cannot perfect title any earlier than the basic statutory period (here 10 years, or in 2010). B 2010Correct. The earliest Mira can perfect title is in 2010. At minimum, she must always occupy the property for the standard statutory period, here ten years. While Catalina's disability does not provide her (or her heirs) any additional time beyond the statutory period, it can never shorten the statutory period. C 2015Incorrect. The five year additional period begins when the disability ends (2002), not when the standard statute of limitations ends (2010). D 2025Incorrect. Remember disabilities cannot be tacked. The only disabilities that are taken into account are the disabilities of the owner when the adverse possessor begins her occupancy. Felipe's disability is irrelevant as is the fact that Felipe regained competency in 2020.

Willie owned a small house on a quarter acre of land (Greenacre). In 2000, Sean entered Greenacre and began to meet all the elements for adverse possession. Willie was born in 1995. The statutory period in this jurisdiction is ten years. A state statute also suspends the running of the statutory period during the disability and for five years after the disability ends. The age of majority is 18. What is the earliest date that Sean could perfect title by adverse possession?

A 2010Incorrect. Sean would have perfected title in 2010 only if Willie were not under a disability when Sean entered Greenacre. B 2013Incorrect. Although Willie's minority ended in 2013, the statute provides that the adverse possessor must continue his occupancy for an additional period of five years after the disability terminates before title will vest. C 2015Incorrect. The five year period is added to the date the disability ends (2013), not to the end of the normal statutory period (2010). D 2018Correct. Willie's disability ended in 2013 (he was five when Sean entered and became 18 in 2013). Adding the additional five year period that the statute grants an owner to protect his interest, the earliest Sean could gain title is 2018 (2013 + 5).

A bailment is a rightful possession of goods by someone who is not the owner. A bailee has the duty to return the item to the bailor and typically owes some level of care in her handling of the bailed item. Which of the following situations correctly identifies the duty of care of the respective bailee?

A A valet is responsible for any damages caused when she parks a car.Incorrect. When a bailment is for the mutual benefit of the bailor (valet receives money) and bailee (individual profits from having his car parked), the bailee has the duty to take reasonable care of the property. The valet would be liable if she negligently parked the car, but not otherwise. B An individual who borrows her neighbor's lawnmower must pay to have a broken wheel fixed even if she was not negligent when using the lawnmower.Correct. When the bailment is for the primary benefit of the bailee (use of the lawnmower), the bailee must use extraordinary care. The individual who borrows her neighbor's lawnmower must return it in the same condition that she received it. C A law student who agrees to take care of her friend's cat while her friend goes on vacation owes her friend a duty of extraordinary care when taking care of the cat.Incorrect. When the bailment is for the primary benefit of the bailor (care of her cat), the bailee is liable only if the property is damaged because of gross negligence or bad faith. If the cat unexpectedly jumped out of a window to its death, the law student would not be liable. However, if the cat died because the law student failed to feed or water it for two weeks, the student would be liable because she was grossly negligent. D A jeweler who finds a lost diamond ring on the public sidewalk is responsible if she negligently damages the ring.Incorrect. Finders are bailees. When a lost item is found, a constructive bailment for the primary benefit of the bailor is created. If the bailment is for the primary benefit of the bailor, the bailee is liable only if the property is damaged because of gross negligence or bad faith. Liability is not established if the jeweler (finder) is merely negligent.

Newspaper AAA designs a computerized form for its employees to report on their deliveries of newspapers. The form contains the following information: route number; date; hours worked that day; hourly pay rate; total pay due for that day; and the employee's signature. Each employee must fill out the form every day that he or she works. Newspaper BBB copies this form for use by its own employees. AAA sues BBB. Who will win the lawsuit?

A AAA, under the doctrine of equivalents.Incorrect. AAA has, at most, a copyright claim. The doctrine of equivalents does not apply, because it is a patent concept. B AAA, because of striking similarity.Incorrect. The two forms are strikingly similar, but AAA's form is not sufficiently original under the Feist test to qualify for copyright protection. C BBB, under the fair use defense.Incorrect. AAA's form is not sufficiently original under the Feist test to qualify for copyright protection, so BBB will win the case without having to establish the fair use defense---which it probably could not do. D BBB, because the form cannot be copyrighted.Correct. AAA's form is not sufficiently original to qualify for copyright protection under the Feist test.

The best explanation for the Supreme Court's ruling in Johnson v. M'Intosh that Native Americans did not have the authority to transfer their ancestral lands to private buyers is that the Native Americans:

A Abandoned their lands.Incorrect. This theme is mentioned in the case, but it is not seriously argued. B Waited too long to assert their right to transfer.Incorrect. This theme is mentioned in the case, but it is not seriously argued. C Did not have the right to transfer under the laws of the United States.Correct. Johnson reflects legal positivism. Under the laws of the United States, the Native Americans did not have the right to transfer title to their ancestral lands, as the Court stated: "All of our institutions recognize the absolute title of the crown, subject only to the Indian right of occupancy." As the successor to the crown's rights, the United States obtained the right to transfer title. Accordingly, "[c]onquest gives a title which the Courts of the conqueror cannot deny." D Did not establish first-in-time possession.Incorrect. The Court acknowledges that the Native Americans had possession of their traditional lands before European settlers arrived and, accordingly, that they had certain rights based on that prior possession.

Leroy owned a four-story apartment building, painted light blue. He leased the units on the lower three floors to tenants, but retained the fourth floor penthouse as his personal residence. Leroy extended the penthouse balcony outward for 20 feet in order to install a new hot tub; the extension was painted light blue to match the rest of the building. Although Leroy did not realize it, 10 feet of the new balcony extension protruded into airspace over a vacant lot owned by Bill. Leroy used the hot tub once, did not like it, and never used his balcony again for the next four years. Leroy then sold the building to Abby, who immediately occupied the penthouse apartment and used the hot tub daily for the next two years; Abby did not notice that any part of the balcony extended into Bill's airspace. Bill never visited his lot after the balcony extension was built. The period for adverse possession in the jurisdiction is five years. Who owns the airspace occupied by the outermost 10 feet of the balcony extension in most jurisdictions?

A Abby, because the adverse possession elements are satisfied.Correct. The best answer is that the adverse possession elements are satisfied. The balcony extension owned by Leroy/Abby physically occupied the airspace; this occupancy was exclusive; it was open and notorious because the extension could have been seen from the ground surface; it was adverse and hostile under the good faith test because neither Leroy or Abby knew the balcony extended into Bill's airspace, and under the objective test their conduct satisfies the other adverse possession requirements; the balcony extension continually occupied the airspace; and Abby can tack her two years of possession onto Leroy's four years of possession to meet the six year period because Leroy's conveyance to Abby created privity between them. B Bill, because the possession of the airspace was not continuous.Incorrect. True, Leroy used the balcony only once in four years, while Abby's use was daily. But the balcony extension physically occupied Bill's airspace constantly for more than five years, which meets the continuity requirement. C Bill, because the possession of the airspace was not open and notorious.Incorrect. The extension was painted light blue, so it is possible that there might be some difficulty seeing it from the ground surface on a very sunny day. On most days, however, a person standing on the edge of Bill's land who looked straight up would easily notice the balcony, especially because the light blue color would contrast sharply with clouds in the sky. D Bill, because Abby did not have possession of the airspace for five years.Incorrect. The adverse possession periods of two successive occupants may be added together to meet the statutory period if they are in privity. Here, Leroy and Abby were in privity because Leroy conveyed title to the building to Abby. The combined possession of Leroy (four years) and Abby (two years) exceeded the required five-year period.

Will owned a mountain cabin that he used during his summer vacations. He borrowed $50,000 from Bank, secured by a mortgage on the cabin. He then leased the cabin to Tina for a ten-year term pursuant to a written lease. Tina planned to use the cabin during her summer vacations, but she became so busy at work that she was unable to visit it. Abe noticed that the cabin was vacant, took possession, and lived there on a full-time basis for six years. Neither Will nor Tina ever visited the cabin during this period. The adverse possession period in the jurisdiction is five years. Assuming that Abe satisfied all adverse possession elements, what is the state of title to the property?

A Abe has a fee simple absolute.Incorrect. Adverse possession only affects the interests of persons who have the present right to possession. Abe's successful adverse possession gave him Tina's interest, a term of years tenancy, because she was entitled to possession. Because neither Will nor Bank was entitled to possession, Abe's adverse possession did not affect their interests. B Abe has a fee simple absolute, and the property is encumbered by Bank's mortgage.Incorrect. Adverse possession only affects the interests of persons who have the present right to possession. Abe's successful adverse possession gave him Tina's interest, a term of years tenancy, because she was entitled to possession. But because Will was not entitled to possession, Abe's adverse possession did not affect his reversion. C Abe has a term of years tenancy, and Will has a reversion.Incorrect. Adverse possession only affects the interests of persons who have the present right to possession. Abe's successful adverse possession gave him Tina's interest, a term of years tenancy, because she was entitled to possession. But because Bank was not entitled to possession, Abe's adverse possession did not affect its mortgage. D Abe has a term of years tenancy, Will has a reversion, and the property is encumbered by Bank's mortgage.Correct. Abe's adverse possession had no effect on Will's reversion or the Bank's mortgage because neither of them was entitled to present possession of the cabin. Abe received only Tina's term of years tenancy.

Abe owned a residential condominium unit on the 12th floor of a highrise tower in a large city. But Abe never used the unit because he lived in Switzerland. Abe's daughter Beth bribed the building superintendent to give her a key to the unit; over the next 7 years, she spent 12 hours each day practicing the cello there. When Beth decided to move to Japan, her cousin Connie asked if she could use the unit to compose songs; Beth replied "Of course. Here's the key." Connie visited the unit every Tuesday for 10 hours to write songs. When Abe returned 4 years later, Connie claimed title by adverse possession. Assume that the period for adverse possession in the state is 5 years. Connie sued Abe for a declaratory judgment that she owned the unit. Who will win the lawsuit?

A Abe, because Beth did not have actual possession of the unit.Incorrect. It is possible that a person might use a condominium unit for the purpose of practicing a musical instrument. More importantly, Beth used the unit as if it were her home, except that she never slept there. B Abe, because Beth obtained the key by fraud.Incorrect. The manner in which Beth obtained the key is irrelevant. True, Beth acted in bad faith, but so do many adverse possessors. C Abe, because Connie cannot tack on Beth's period of possession.Correct. The privity required for tacking usually arises from a deed; here, the only arrangement between Beth and Connie was a brief conversation. Even under the liberal standard that you saw in Howard, privity is not present. There is no "reasonable connection" between Beth and Connie which raises Connie's claim "above the status of the wrongdoer or the trespasser." D Connie, because she meets the elements for adverse possession.Incorrect. Connie cannot tack on Beth's use of the unit because there is no "reasonable connection" between Beth and Connie which raises Connie's claim "above the status of the wrongdoer or the trespasser," even under the generous Howard standard.

Mia lived in a very remote area. Walking home, Mia took a shortcut over the back ten acres of Liam's farm. Mia noticed a wild turkey next to a small stream that flowed through Liam's property. Throwing a rock, she stunned the turkey, brought it home, and placed it in a cage in her backyard. A few days later, Aiden, a lost and hungry hiker, wandered onto Mia's land, saw the turkey, grabbed it from its cage, and wandered back into the woods. When Mia noticed that the turkey had been taken, she traced the steps of Aiden and found him with the turkey. Between Mia and Aiden, who has better claim to the turkey?

A Aiden, because Mia never had any right to the turkey since she had gained the turkey while trespassing on Liam's land.Incorrect. Even though Liam has a better claim to the turkey than Mia, Mia has better property rights to the turkey than Aiden. Mere possession, even if unlawful, can provide the possessor with certain rights against others. Title is relative. While Mia's possessory rights may be less than Liam's, Mia's claim to the turkey is better than Aiden's. B Mia, because Aiden trespassed on her land to gain the turkey.Correct. Mia's possession is protected against the unlawful acts of Aiden, even though Mia may have gained possession of the turkey unlawfully. The law protects Mia's possessory rights against a later wrongdoer in order to avert an unending series of trespasses. C Mia, because she was first-in-time to gain the turkey.Incorrect. While this statement is true and vests Mia with certain possessory rights, it is not the best answer. For example, an individual could be first-in-time and lose her property rights if the wild animal escapes and returns to its natural habitat. Mia has a better claim than Aiden primarily because he was a trespasser. D Aiden, because he secured certain control of the animal while he was unaware that he was trespassing on Mia's land.Incorrect. Aiden took a turkey that was already in Mia's possession (it was caged on her land). The rule of capture applies to unowned wild animals.

Ashley found a wonderful antique brooch at her local flea market. She treasured the brooch and kept it in a jewelry box beside her bed. She often put it on while she was in her house and would wear it as she did her daily chores. Sometimes when Ashley attended a social function, she would proudly wear her brooch. Over a two year span, Ashley wore the brooch to several functions. Shortly thereafter, while at the opera, Emily noticed the brooch on Ashley's blouse and thought it looked strangely similar to one her grandmother had given her. It turned out that Emily's sister had taken the brooch out of Emily's bureau and sold it to Ashley at the flea market. Both Ashley and Emily now claim the brooch. The statutory period for the adverse possession of chattels is two years. Emily argues that, at minimum, Ashley has not met the open and notorious requirement for adverse possession of personal property. Which of the following is correct?

A Ashley met the open and notorious requirement because she used the brooch as an ordinary owner would.Correct. Although ordinary use may provide little notice to the owner, it makes sense not to require the adverse possessor to go above and beyond what a normal user of the property would do in the circumstances. Ashley used the brooch as the normal owner would and therefore has met the open and notorious requirement. B Ashley did not meet the open and notorious requirement because she did not display the brooch regularly in a public forum.Incorrect. While public display would certainly meet the requirement, it is not necessary. Many items of personal property (such as brooches) are not suitable for regular public display. C Ashley did not meet the open and notorious requirement because she did not use the brooch in a way that would notify Emily of its location.Incorrect. If Ashley used the brooch in a manner that would notify Emily of its location, she would definitely satisfy the open and notorious requirement. However, in the context of personal property, such a rule would be unreasonable and extremely burdensome; and therefore such acts are not necessary. D Ashley did not meet the open and notorious requirement because she did not wear it regularly for the two year period.Incorrect. No jurisdiction requires chattel to be continually used for the statutory period.

Allen owned Pearacre, a 500-acre tract of pear trees which included a small house in its southwest corner. When Allen died, his only living relatives were his daughter Betty and his niece Carrie. When Betty and Carrie looked through Allen's records, they discovered a document which appeared to be Allen's will; it provided in part: "I devise Pearacre to Carrie." The document was signed by Allen. Betty and Carrie both assumed that this paper was a valid will, but under state law it was invalid because no one witnessed Allen's signature; indeed, under state law Betty had automatically received title to Pearacre when her father died without a valid will. Carrie moved into the Pearacre house. Because she disliked pears, however, she never visited or used the rest of the Pearacre property. Two days after Carrie occupied the house, Betty drove a small green travel trailer onto the northeast corner of Pearacre and built a foundation around it; she occupied it each weekend as a vacation retreat from her city job. When Carrie drove down the public road in front of Betty's trailer on weekends, Betty shouted at her: "You robbed me of my inheritance!" Carrie and Betty continued the conduct set forth above for 15 years. The period for adverse possession in this jurisdiction is 12 years. Which of the following statements is correct in most jurisdictions?

A Betty owns all of Pearacre.Incorrect. Carrie satisfied the adverse possession elements for all of Pearacre except the northeast corner. Carrie has actual possession of the Pearacre house because she used it as her residence; her use of the house was exclusive because Betty never interfered with it; it was open and notorious because Carrie lived in the home, and Betty knew about Carrie's occupancy as evidenced by her shouts; Carrie's possession was adverse and hostile in almost all jurisdictions, because she meets the objective test by satisfying the other adverse possession elements and she meets the good faith test because she believed that the will was valid; Carrie's occupation was continuous; and it continued for 15 years, three years longer than required. In most jurisdictions, because Carrie had color of title to all of Pearacre due to the invalid will and satisfied the adverse possession requirements for part of the land, she obtains title to all of Pearacre―except the northeast corner, which was actually occupied by Betty, the true owner. B Carrie owns only the Pearacre house.Incorrect. It is true that Carrie owns the Pearacre house because she satisfied the elements for adverse possession by her 15 years of continuous occupancy. However, because Carrie had color of title to all of Pearacre pursuant to the invalid will, she obtained title to all of Pearacre except for the northeast corner, which was actually occupied by Betty, the true owner. C Carrie owns all of Pearacre except for the northeast corner.Correct. Carrie owns the Pearacre house because she satisfied the elements for adverse possession by her 15 years of continuous occupancy. In addition, because Carrie had color of title to all of Pearacre pursuant to the invalid will, she obtained title to all of Pearacre except for the northeast corner, which was actually occupied by Betty, the true owner. D Carrie owns all of Pearacre.Incorrect. Carrie satisfied all elements of adverse possession as to the Pearacre house; and because she had color of title to all of Pearacre pursuant to the invalid will, she obtains title to all of the rest of the land, except for the northeast corner. Color of title normally gives a claimant constructive possession of all land covered by the instrument. However, here Betty had actual possession of the northeast corner, so Carrie's constructive possession of that corner was not exclusive.

Bill owned a 2,000-acre tract of desert land. In 2005, Cara drove by the land and noticed that rare cactus plants grew there. Cara dug up four cacti, each time leaving a hole that was 18" wide and 12" deep; she sold the cacti for $50. Cara did exactly the same thing on a visit in 2008, another in 2012, and yet another in 2015. Cara also drove past the land without stopping during trips in 2006, 2007, 2009, 2011 and 2014. Between 2005 and 2015, Bill drove past the land on two occasions without stopping. Assume that the period for adverse possession in the state is 10 years. Cara now sues Bill for a declaratory judgment that she owns the land. Who will win the lawsuit?

A Bill, because Cara did not have continuous possession.Correct. Cara's possession was not continuous, assuming that Cara had possession at all. C entered the land on only four occasions, presumably just long enough to steal cacti—perhaps 30 minutes each time. Over slightly more than 10 years, she spent about two hours on the land, not enough for continuous possession even of desert land. B Bill, because Cara did not have exclusive possession.Incorrect. Cara's arguable possession was exclusive because Bill never used the land at all; merely driving past it on the highway is not a use of the land. It is not even a visit. C Bill, because Cara did not have open and notorious possession.Incorrect. Under the Gurwit standard, Cara's possession might be seen as open and notorious, although the argument is weak. Like Gurwit's conduct, Cara's activity was "in the sight of passers by;" and the 16 holes that Cara left on the land are perhaps akin to the signs that Gurwit posted. But it seems unlikely that 16 holes would have obvious to an inspecting owner, given the large size of the parcel. D Cara, because she meets the elements for adverse possession.Incorrect. Cara's possession was not continuous, assuming that Cara had possession at all. Cara entered the land on only four occasions, presumably just long enough to steal cacti—perhaps 30 minutes each time. Over slightly more than 10 years, she spent about two hours on the land, not enough for continuous possession even of desert land.

Anna owns a 100-acre tract of farmland. In which of the following situations did a trespass not occur?

A Bob walks onto Anna's land, believing in good faith that he owns it.Incorrect. Bob is liable for trespass because he voluntarily entered land which Anna owned. Because trespass is a strict liability tort, Bob's good faith is irrelevant. B Carol pushes Dave onto Anna's land as a joke.Incorrect. Carol is liable for trespass because she voluntarily caused Dave to enter Anna's land. Dave is not liable because his entry was inadvertent. C Elmer has to land his hot air balloon on Anna's land.Correct. The best answer is that Elmer did not trespass because he entered Anna's under a privilege arising from necessity. A hot air balloon can only remain aloft for a short period of time and cannot be steered in any particular direction. Thus, the pilot of a hot air balloon has little choice about when or where to land. The facts here indicate that Elmer "has to" land on Anna's land, which implies that the landing was due to necessity rather than voluntary choice. D Fran runs onto Anna's land to recover her $20 bill which a strong wind has carried away.Incorrect. Fran is liable for trespass because she entered Anna's land voluntarily. Fran did not have a privilege to enter since there was no necessity for her to recover the money.

Bob, who owned a bar, created a new cocktail which he dubbed "Bob's Blue Blend." Bob wrote down the ingredients and the technique for making the cocktail on a piece of paper; he added the word "Copyright" and the date at the bottom of the page. In order to publicize his bar, Bob paid for ads on internet sites which proudly proclaimed that the bar was "the home of Bob's Blue Blend." The ads also allowed a viewer to click on a link which took him to a copy of the recipe for making the cocktail. Irma, who owned a newspaper, clicked on the link and learned how to make the cocktail. She then published a story in her paper which explained in her own words how readers could make the "famous Blue Blend" cocktail in their own homes. After this, revenue from Bob's bar fell sharply. Bob then sued Irma. Who will win the lawsuit?

A Bob, because Irma infringed his copyright.Incorrect. Copyright protects the expression of an idea, not the idea itself. The manner in which Bob described how to make the cocktail on the piece of paper or on the website might qualify for copyright protection, but not facts about how to make it. Irma used her own words in describing the process, not Bob's words, so she did not infringe his copyright. B Bob, because Irma infringed his patent.Incorrect. Patent protection arises only when the federal Patent and Trademark Office issues a patent. There is no indication on these facts that Bob received a patent. C Bob, because Irma infringed his right of publicity.Incorrect. The right of publicity relates to the situation where a person uses the name, likeness, or other component of a person's identity to advertise or sell a product. Irma did not use Bob's name, image, or any other part of his identity in connection with her story. The cocktail recipe is not part of Bob's identity. D Irma, because her conduct was legal.Correct. The traditional rule is that the law does not recognize intellectual property rights. Information and ideas are, in general, not subject to private ownership; accordingly, they may be used freely by anyone. None of the exceptions to this general rule apply on the facts of this problem.

Carla, a biochemist, discovered that certain cells in Dan's body contained a liquid which could cure brain cancer. Carla begged Dan to let her remove some of his cells so that thousands of patients could be cured each year, but Dan refused because Carla was unwilling to pay him what he thought was a fair price. On a dark night, Carla saw Dan walking in a park, stunned him with a punch in the jaw, quickly cut away a small bit of Dan's skin that contained some of the valuable cells, and ran away. Carla then sold the cells to Elsa, a scientist who plans to use them to create a cell line that will generate the life-saving medicine. Which of the following is correct?

A Carla owns the cells because she excised them from Dan's body.Incorrect. True, arguably the court in Moore v. Regents of the University of California held that cells excised from a patient's body were no longer owned by the patient (if they ever had been), but rather could be used by the surgeon and others to create a cell line. But Carla's conduct here violates a fundamental public policy that was not at issue in Moore: the prohibition of criminal conduct. Carla committed a crime by removing Dan's cells without his consent. Because of this policy difference, Moore can be distinguished. B Dan owns the cells even though they were excised from his body.Correct. The best answer is that Dan owns the cells even though they were removed from his body, because the removal was a wrongful act. Moore v. Regents of the University of California might suggest a contrary conclusion. But Carla's conduct here violates a fundamental public policy that was not at issue in Moore: the prohibition of criminal conduct. Carla committed a crime by removing Dan's cells without his consent. Because of this policy difference, Moore can be distinguished. C Elsa does not own the cells because human cells can never be bought or sold.Incorrect. Human cells can sometimes be bought or sold. Blood, hair, and sperm, for example, are often bought and sold. D No one owns the cells because human cells cannot be owned.Incorrect. The law recognizes ownership of human cells under some circumstances. A person who sells his blood, for example, transfers ownership of the blood to the buyer. Moreover, many courts conclude that a person owns the cells which are still part of his or her body.

Ella owned Swampacre, a wild and undeveloped tract of wetlands spanning 1,500 acres, which she used for hunting wild ducks a few weeks each year with her son Fred. When Ella died, Fred assumed that he had inherited Swampacre, because Ella had always promised to leave it to him. But instead her valid will transferred Swampacre to a local church. Fred never read the will; nor did anyone tell him that the church owned Swampacre. For the next 10 years, no one visited Swampacre except for Fred, who hunted ducks on every part of the land during duck season for two weeks each year. Fred built three small "duck blinds" on the land; each one consisted of a wooden floor surrounded by reed-like camouflage to hide Fred from passing ducks. Fred's hunting on the land was quite successful; each year, he was able to kill the maximum number of ducks allowed by law, though many of his shots missed their intended targets. Last week, a representative of the church visited Swampacre for the first time, saw Fred hunting there, and shouted: "Get off the church's land!" The period for adverse possession in the jurisdiction is 10 years. Who owns Swampacre in most jurisdictions?

A Church, because Fred's possession was not adverse and hostile.Incorrect. It is not necessary that Fred have the bad faith intent to wrest title from the church, the true owner. In many jurisdictions, the adverse and hostile element is measured by the objective conduct of the claimant; if he satisfies the other adverse possession elements, this element is satisfied---unless he used the land with permission of the true owner. Here there is no indication that the church consented to Fred's use of the land. Indeed, the reaction of the church representative upon seeing Fred on the property was to demand that he leave, not to invite him to stay, which indicates that the church did not consent to the use. In other jurisdictions, the adverse and hostile element is met if the adverse claimant believes in good faith that he is the owner, which is the case here; Fred believed that he had inherited Swampacre. B Church, because Fred's possession was not continuous.Incorrect. The extent of continuity required is measured by the nature, character, and location of the land. Based on these factors, it appears that the only reasonable use for Swampacre is hunting ducks during duck season, which is when Fred hunted. His actions were sufficiently continuous. C Church, because Fred's possession was not open and notorious.Incorrect. It is true that Fred sought to hide his hunting activities from ducks, which is why he built (and presumably used) the duck blinds; thus, if a church representative had inspected the land he might not have seen evidence of Frank's activities. Moreover, given the size and topography of Swampacre, it unlikely that a church representative would come across the duck blinds if he had inspected the land. Although Fred's activities were not readily visible, they were readily audible. Duck hunting produces loud shotgun blasts. If a church representative had inspected the land during duck season, he probably would have heard Fred hunting. D Fred, because the adverse possession elements are satisfied.Correct. The best answer is that all adverse possession elements are satisfied here. Fred had actual possession of Swampacre, in that he used it as an ordinary owner would; his use was exclusive because nobody else used the land; it was open and notorious because the frequent shotgun blasts would be easily heard by an inspecting owner; it was adverse and hostile under both the objective standard and the good faith standard; Fred's use was sufficiently continuous given the nature, character, and location of the land; and the use continued for 10 years, the required statutory period.

When considering the effect of disabilities on the statutory period for adverse possession, which of the following is not correct?

A Death ends all disabilities.Incorrect. This is a correct statement. Any disability of the owner that exists when an adverse possessor enters the property is deemed to end upon the owner's death. Suppose a jurisdiction specifies that the age of majority is twenty-one. If an owner was five years old when the adverse possessor entered and the owner later died at age ten, the disability ended at the owner's death, rather than ending upon the hypothetical date when the owner would have gained majority. B Disabilities may be tacked.Correct. This is an incorrect statement. Disabilities cannot be tacked. Only disabilities of the owner that exist when the adverse possessor enters the property can be taken into account. If a "disabled" owner conveys her property to a "disabled" successor during the statutory period, the disability of the successor is not considered. C A disability cannot shorten the statutory period for adverse possession.Incorrect. This is a correct statement. While disabilities can lengthen the statutory period, they cannot shorten it. Even though a disability is removed shortly after adverse possession begins, the adverse possessor must, at minimum, meet all the required elements for the full statutory period. D A disability may lengthen the statutory period for adverse possession.Incorrect. This is a correct statement. Disabilities can affect the statutory period and potentially lengthen it. Generally, because owners are "disabled" from protecting their property rights during the term of their disability, the law provides an added, limited period of time after the disability ends within which the owner can bring suit. If this "window" period extends beyond the established statutory period, the statutory period will be lengthened.

Bob holds title to Redacre, a 100-year-old mansion. Over the objection of his family and his neighbors, Bob plans to demolish Redacre in a week. A group of Bob's neighbors file suit, seeking a court order to prevent the demolition. What is the most likely basis for obtaining such an order?

A Demolition of Redacre would lower the value of the neighbors' houses.Incorrect. While this factor was present in Eyerman, that case involved destruction after the owner's death; the court stressed that a living person has the power to dispose of his property "with fewer restraints than a decedent by will." B Redacre is listed on the National Register of Historic Places.Incorrect. A listing on the Register does not restrict an owner's right to destroy. C Demolition of Redacre would leave Bob penniless.Incorrect. U.S. law generally respects the right of an owner to destroy his property during his lifetime as he wishes, as long as he is mentally competent. D Demolition of Redacre would release asbestos particles that endanger public health.Correct. As a general rule, a landowner has a broad right to destroy property during his lifetime, absent direct harm to others. But here the demolition will be done in a manner that releases dangerous asbestos particles into the air, so the court would probably enjoin it as a nuisance.

Consider the ability to protect the color green as a trademark for chewable vitamins for adults (that is, the vitamin tablets are themselves are green in color). The vitamins have also been flavored with mint to make them more palatable. The application to protect the green color as a mark for chewable vitamins that are mint flavored would likely be considered:

A Descriptive, and protectable only upon a showing of secondary meaningIncorrect. It is not protected. B Suggestive, and protectable without a showing of secondary meaningIncorrect. It is not protected. Color also always requires secondary meaning. C Arbitrary, and protectable without a showing of secondary meaningIncorrect. It is not protected. Color also always requires secondary meaning. D Unprotectable on grounds of functionality (i.e., aesthetically functional to allow patients to tell the pills apart)Correct. The attempt to protect the color green for chewable vitamins that are also mint flavored presents a challenge in trademark law. This type of trademark application would most likely be unprotectable on grounds of functionality (i.e., aesthetically functional) because the green color can help consumers easily identify the flavor they can expect when they chew the vitamin tablets, given that in the United States, the consuming public often expects mint flavor and green color to correspond with one another. With the mint flavor itself remaining free for other producers to use under these facts, other producers should remain free to use the color green to communicate to consumers the flavor of the vitamin tablet in the non-reputation-related way that consumers have come to expect for vitamin products. In other words, barring the use of green to color a chewable mint-flavored vitamin would put the other producers at a significant non-reputation-related disadvantage in the vitamin marketplace. This is the essence of "functionality" in trademark, although here it is an appearance-based functionality rather than functionality based on whether the product works better or fulfills its intended purpose (as a vitamin product). In Qualitex Co. v. Jacobson Products Co., 514 U.S. 159, 165 (1995), the Court added that a feature is functional if its exclusive use by one enterprise "would put competitors at a significant non-reputation-related disadvantage." In other words, a feature is functional, and thus unprotectable, if it is one of a limited number of efficient options available to competitors, and therefore free competition would be unduly hindered by giving the design or feature trademark protection. The functionality doctrine helps prevent the stifling of competition by the exhaustion of a limited number of options for product packing or design, or by preventing a competitor from replicating an important non-reputation-related product feature.

Kate just celebrated her ninetieth birthday. She is extremely frail, takes copious amounts of prescription drugs, and is often very sickly. Yesterday, Kate felt terrible. She phoned Eduardo and asked him to come by her house. When he arrived, Kate called him to her bed and softly whispered, "I'm feeling bad today. God only knows how much time I have left on this earth. I'm not sure when I will see you again, so here, take this amulet. It's my most prized object and it will bring you good luck." Kate handed the amulet to Eduardo. He thanked her profusely, put the amulet in his pocket, and left. Two weeks later, Kate regained her strength. She called Eduardo, explained that she needed her good luck charm back, and demanded that he return the amulet. Who has the better claim to the amulet?

A Eduardo, because Kate made a valid inter vivos gift.Correct. A donor must fear imminent death from a specific cause in order to make a valid gift causa mortis. Even though Kate is extremely old, sickly, and feeling bad, there is no indication that she feared death was imminent. Taken as a whole, all of her statements to Eduardo regarding her health do not demonstrate that she feared her death was at hand. Here, Kate made a valid inter vivos gift, not a gift causa mortis. Generally, inter vivos gifts are irrevocable. Eduardo owns the watch. B Eduardo, because Kate waited too long to revoke her gift causa mortis.Incorrect. The gift was not a gift causa mortis, but an inter vivos gift. In addition, even if it were a gift causa mortis, Kate could revoke it at any time before she died. A two week period would not make the gift irrevocable. C Kate, because a gift causa mortis is revocable at any time before she dies.Incorrect. Although a gift causa mortis is revocable at any time before the donor dies, Kate made an inter vivos gift, not a gift causa mortis. Nothing indicates that she feared imminent death. D Kate, because Kate was sick when she attempted to make the gift.Incorrect. Even though Kate was sick when she made the gift, nothing indicates that she was mentally impaired or confused. She intended to gift her amulet to Eduardo, she delivered the amulet, and Eduardo accepted it. It was a valid inter vivos gift and the amulet is Eduardo's.

In 2000, Roy conveyed Greenacre and Meadowacre (two five-acre contiguous parcels) to Farrah. Farrah paid $140,000 for both parcels and received a general warranty deed from Roy. Farrah immediately moved onto Greenacre and constructed a two-story house. She cleared most of the rocky terrain immediately surrounding the house and planted a large garden next to the backdoor. Farrah rarely wandered anywhere on Greenacre outside of her house or garden. Farrah also never ventured onto Meadowacre. She was an environmentalist and wanted to preserve the pristine nature of the parcel. In 2012, Jaxen appeared and instructed Farrah that he owned both parcels, having validly purchased Greenacre and Meadowacre in 1990. After some investigation, the following facts were established: (i) Roy never owned either parcel; (ii) the deed from Roy to Farrah was invalid; and (iii) Farrah had no knowledge of Roy's fraud and had always acted in good faith. The statutory period for adverse possession in this jurisdiction is ten years. Which of the following is correct?

A Farrah owns both Greenacre and Meadowacre.Incorrect. Color of title can provide enhanced protection for the adverse possessor. An adverse claimant with color of title is deemed to be in constructive possession of the entire parcel even when she occupies only a portion of it. However, where a deed covers more than a single parcel, the adverse possessor's claim only extends to the parcels she has actually occupied. Here Farrah never ventured onto Meadowacre. Even with color of title, Farrah is not deemed to have constructively possessed Meadowacre. B Jaxen owns both Greenacre and Meadowacre.Incorrect. Farrah has met the requirements to adversely possess Greenacre. Color of title vests her with title to the entire parcel of Greenacre, even though she may not have actively possessed each inch of it. C Farrah owns Greenacre and Jaxen owns Meadowacre.Correct. Regarding Greenacre, Farrah has met the elements of adverse possession for the statutory period. Color of title provides her with title to the entire parcel even though she may not have occupied large portions of it. However, Farrah never occupied Meadowacre, a separate parcel. Color of title will not extend the fiction of constructive possession to a parcel the adverse possessor has not even partially possessed. D Jaxen owns Greenacre and Farrah owns Meadowacre.Incorrect. Farrah owns Greenacre. She has met the required elements of adverse possession for at least a portion of the parcel. Because her claim is based on an invalid deed that was gained in good faith, color of title allows her to extend her possessory rights to the entire parcel. However, Farrah has not met the elements required for adverse possession of Meadowacre. Because she has never occupied Meadowacre, color of title is of no help. Jaxen owns Meadowacre.

Oscar owned a Stradivarius violin worth $1.5 million. Dean, a dealer who sells and restores stringed instruments, stole the violin after one of Oscar's concerts. Four months later, Dean sold the Stradivarius for $2 million to Fay, who had no idea of the violin's history. A few weeks later, Oscar discovered that Dean had sold his violin to Fay and demanded its return. Fay claims the violin is hers. Who has the best title to the violin?

A Fay, because she purchased the violin from Dean, a merchant, for valuable consideration and without notice of any adverse claims.Incorrect. U.C.C. § 2-403 does not apply since Dean, even though a merchant, stole the violin. He was not entrusted with it. B Fay, because she purchased the violin from Dean, who had acquired title by adverse possession.Incorrect. No statute of limitations is as short as a few months, and therefore Dean had not acquired title by adverse possession. C Oscar, because he never transferred any interest to Dean.Correct. A thief gains no title and cannot pass valid title to another (unless the thief gains title by adverse possession). Even though Fay was a good faith purchaser, Dean had no title to pass to her. D Fay and Oscar have equal title and it is up to a court of equity to determine the most equitable distribution.Incorrect. Because Dean stole the violin, he had no title to transfer to Fay. Oscar holds sole title to the violin.

The theory of property which best explains why the farmworkers in State v. Shack were entitled to receive visitors over the landowner's objection is:

A First possession theory.Incorrect. Tedesco's possession was first-in-time. B Labor theory.Incorrect. Although the farmworkers labored on the farm, Tedesco's labors actually created the farm in the first place. C Utilitarian theory.Correct. The sentiment in Shack that property rights "serve human values" and "are recognized toward that end" reflects classic utilitarian theory. D Civil republican theory.Incorrect. An argument might be made that opening the farm to visitors would facilitate political participation by the farmworkers, but this is not the best answer. The sentiment in Shack that property rights "serve human values" and "are recognized toward that end" reflects classic utilitarian theory.

Fred, a huge fan of the Harry Potter books, decides to write a book which uses a boy wizard called "Charlie Cotter" as the main protagonist. Charlie Cotter has the same traits and characteristics as Harry Potter, but Fred's story is set in Australia in 1900, where Charlie Cotter must use his magic talents to help poor sheepherders contend with evil forces. BBB, which owns the copyrights to the Harry Potter books, sues Fred. Who will win?

A Fred, because he did not copy any of the protected expression in the Harry Potter books.Incorrect. The Harry Potter character is sufficiently unique to qualify for copyright protection, separate and apart from the copyrights in the text of the novels. B Fred, because he is telling an entirely different and original story.Incorrect. The Harry Potter character is sufficiently unique to qualify for copyright protection, separate and apart from the copyrights in the text of the novels. C Fred, because of the fair use defense.Incorrect. Fred cannot establish the defense because his novel was written for profit; it takes all of the Harry Potter character; and it impairs the market for future sequels about the same character which BBB might issue. D BBB, because Fred infringed its copyrights.Correct. The Harry Potter character is sufficiently unique to qualify for copyright protection, separate and apart from the copyrights in the text of the novels.

Fred finds a diamond ring in a field and places it in his pocket. As Fred walks home, the ring falls out of his pocket and onto the street. Kaya, a passerby, finds the ring. As Kaya admiringly holds it up to the sun, Loni grabs the ring from Kaya's hand and runs away. Fred, Kaya, and Loni bring an action to determine who has the

A Fred.Correct. Fred has the better property right because he is the prior possessor of the ring. B Kaya.Incorrect. Kaya, as a finder, has better property rights in the ring than the whole world, except for a prior possessor—which Fred is. C Loni.Incorrect. Loni, as a thief, cannot gain better rights than a prior possessor. Both Fred and Kaya have better rights than Loni. D Not for Fred, or Loni, because the ring belongs to the true owner.Incorrect. A court determines the property rights of the parties before it. Because the true owner is not before the court, the court will only determine who among Fred, Kaya, and Loni has the best title to the ring.

Zoe purchased an electric guitar from Bella. Bella had stolen the guitar one week earlier. Zoe kept the guitar in her home, only taking it out when she went to her guitar instructor's studio for lessons. After one year, Zoe sold the guitar to her cousin Freda, a well-known musician. For the next year, Freda traveled around the state with the guitar, giving public concerts in over twenty cities. Assuming a two year statutory period for adverse possession of chattels, the weight of authority would conclude:

A Freda has no title to the guitar because Zoe acquired possession from a thief.Incorrect. Although Bella did not have any title to transfer, Freda can gain title by her own actions and the actions of her predecessor, Zoe --as adverse possessors. B Freda has no title to the guitar because she has not met the required statutory period.Incorrect. Although Freda did not meet the required two year statutory period herself, she is able to tack the additional year of Zoe's possession. C Freda has no title to the guitar because she cannot tack Zoe's possessory period onto hers.Incorrect. Even for personal property, most jurisdictions allow tacking if there is privity between the adverse possessors. Here privity exists because there was a voluntary conveyance between Zoe (seller) and Freda (buyer). D Freda has title to the guitar.Correct. Freda owns the guitar since she can tack Zoe's period of possession to her own period of possession. The weight of authority allows tacking as long as there is privity between the possessors. Freda purchased the guitar from Zoe and therefore privity exists.

Freda provides housing on her farm for farmworkers during the harvest season. George, a neighboring farmer, seeks to enter Freda's land in order to induce them to quit their jobs on Freda's farm and instead work on George's farm. Harry, the cousin of a farmworker living on Freda's farm, also wants to enter to visit his rel

A Freda may only exclude George.Correct. Freda has a legitimate interest in excluding a competitor like George who seeks to interfere with her farming activities by luring away her employees. B Freda may only exclude Harry.Incorrect. Freda has no legitimate reason for excluding Harry, the farmworker's cousin. The Shack court stresses that a farmworker is entitled to enjoy "the associations common among our citizens," and these would include normal visits by family members. C Freda may exclude both George and Harry.Incorrect. Freda has no legitimate reason for excluding Harry, the farmworker's cousin. The Shack court stresses that a farmworker is entitled to enjoy "the associations common among our citizens," and these would include normal visits by family members. D Freda may not exclude George or Harry.Incorrect. Freda has a legitimate interest in excluding a competitor like George who seeks to interfere with her farming activities by luring away her employees.

Jay owned an expensive gold pen. When Jay became seriously ill, he wrote a will leaving all his property to his wife, Gene. Jay then summoned Aya to his bedside and handed her his pen, saying: "Take this. I want you to have it to remember me by." Aya took the pen and Jay died shortly thereafter. Who has the better claim to the pen?

A Gene because Jay left all of his property to her.Incorrect. Jay's will only controls property in his estate at his death. Since Jay gave away his pen before he died, the pen is no longer a part of his probate estate. B Gene because Jay did not have a present intent to give the pen to Aya.Incorrect. As Jay's statement evidences ("Take this. I want you to have it..."), Jay had a present intent to make the gift. C Aya because she has present possession of the pen.Incorrect. While Aya does have present possession of the pen, this fact alone will not enable Aya to gain priority over Gene. Aya needs to demonstrate that there was a valid gift of the pen to her (otherwise Gene would have the better claim as the devisee under Jay's will). A valid gift requires other elements, such as intent and delivery. D Aya because Jay made a valid gift causa mortis.Correct. Jay satisfied all the requirements for making a gift causa mortis. Jay made the gift in contemplation of death, had the necessary donative intent, manually delivered the pen, and Aya accepted his gift.

In 2003, Jasmine purchased an older, furnished house. Although she lived in a neighboring state, Jasmine purchased the house with the intent to move there when she retired. To help her make her mortgage payments on the house, Jasmine immediately rented it to her friend Hanna. Hanna had lived there for over ten years when she decided, with Jasmine's permission, to repaint the interior. When moving a dresser in the upstairs bedroom, Hana found a dusty antique ring resting on the floorboard. Hanna was joyous about her find. She immediately called Jasmine to tell her the good news. To Hanna's amazement, Jasmine was not supportive and claimed that the ring should be hers since it was found in her house. Who has the best right to the ring?

A Hanna, because she labored to move the dresser.Incorrect. While property rights may be based on labor theory, finders law focuses on a different policy -- rewarding the finder. It promotes the important social goal of placing lost items back into the economy. Finders receive property rights regardless of the extent of their labor in making the find. B Hanna, because she found the ring and Jasmine did not reside in the house.Correct. Finders usually have a better right to the lost item against all except prior possessors. However, where the find is made in a residence, other policies must be considered. If the owner resides in the house, the owner's expectations regarding found objects are high. Generally, these expectations are deemed to outweigh any interests in the finder. However here, because Jasmine was an absentee owner, her expectations are low. The policies of rewarding the finder and encouraging honesty outweigh Jasmine's expectations and consequently Hanna has the better right to the ring. C Jasmine, because she owned the house.Incorrect. Jasmine does not have better rights to the ring merely because she owned the house. Since Jasmine did not exercise dominion over the house (she did not live there), her expectations regarding found items in the house should be minimal. The policies of rewarding the finder and encouraging honesty outweigh the owner's expectations in this instance. D Jasmine, because the ring was attached to the house.Incorrect. The ring was not attached to the house. It was merely resting on the floor board. However, if it were attached, the ring would go to Jasmine. As owner, Jasmine has better rights to all objects attached to her property.

Irma, an inventor, purchased a two-acre parcel of country land so that she could it use while developing new inventions. Because many of Irma's inventions could be tested only under conditions that minimized the effect of various types of solar radiation, she decided to have a cave dug on her land so that she could conduct underground testing. Irma hired Cal to dig a cave located 30 feet below the surface of her property, which ran parallel to the surface for 100 feet. Cal inadvertently built the cave in the wrong place, so it extended 40 feet into the soil beneath the house owned by Nancy, Irma's neighbor. When Cal learned about the error, he explained the situation to Nancy, apologized to her, and told her that Irma would be using the tunnel for testing inventions; but Cal never told Irma about his mistake. Irma used the portion of the tunnel beneath Nancy's house once a month for the next six years. There was no visible indication on the surface of Nancy's land that there was a tunnel below it; nor did Irma's testing in the tunnel produce any noise or odors which a person on Nancy's land would have noticed. Irma and Nancy never discussed the tunnel. The period for adverse possession in this jurisdiction is five years. Who owns the portion of the tunnel beneath Nancy's land in most jurisdictions?

A Irma, because the adverse possession elements are satisfied.Correct. The best answer is that all adverse possession elements are satisfied here. Irma use the tunnel area in a more intensive manner than a reasonable owner would do, which meets the actual possession element; her possession was exclusive because Nancy never used the tunnel; her possession was open and notorious because Nancy had actual knowledge of it; her possession was adverse and hostile under the good faith and objective standards; her possession was sufficiently continuous because she used the tunnel every month, more often than a reasonable owner would do; and her use continued for six years, longer than the required five-year period. B Nancy, because Irma did not have adverse and hostile possession.Incorrect. Irma's use was adverse and hostile in almost all jurisdictions. She believed that the tunnel was on her land, which satisfies the good faith standard. The objective standard is also met because the other adverse possession requirements are satisfied. There is no indication that Nancy consented to Irma's use of the land. C Nancy, because Irma did not have exclusive possession.Incorrect. Irma's use was exclusive because neither Nancy nor anyone else ever used the tunnel. True, Nancy was in possession of the land above the tunnel, where her house was located; but this does not constitute possession of the subsurface. D Nancy, because Irma did not have open and notorious possession.Incorrect. Irma's use was not open and notorious in the usual sense, in that an owner making a reasonable inspection of Nancy's land would not discover her use. But where the true owner has actual knowledge of the claimant's use, as Nancy does here, this satisfies the requirement of openness and notoriety.

Jacob tracked a small wild boar on unowned land. Jacob won a gold medal in shooting at the most recent summer Olympics and has never missed his target. Surprisingly, the boar came within three feet of Jacob. Jacob aimed his rifle, but just as he was about to press the trigger, Sophia jumped out of the bushes, screaming loudly. She immediately hit the boar with a large stick and then picked it up, walking away with the unconscious boar in her arms. Who has best rights to the boar?

A Jacob, because he invested time and energy in the capture.Incorrect. While labor theory rewards the investment of resources, the rule of capture requires an individual to gain certain control over the wild animal in order to gain ownership rights. B Jacob, because he had a reasonable prospect of killing the boar.Incorrect. The rule of capture demands an individual gain certain control over the wild animal. A reasonable prospect of killing is not certain control. Certain control requires actual killing, entrapment, bodily seizure, or mortal wounding plus pursuit. C Jacob, because he was first-in-time to find and track the boar.Incorrect. The rule of capture does not reward an individual for being first-in-time to discover or track a wild animal. It demands the individual be first-in-time to gain certain control over the animal. D Sophia, because she was the first to gain certain control of the boar.Correct. Sophie owns the boar as long as she maintains her bodily seizure of it. Under the rule of capture, an individual owns a wild animal once she kills, entraps, bodily seizes, or "mortally wounds and pursues" the wild animal.

Ron wrote his son Jeb, who was studying in Brazil, the following letter: "Son, Happy Birthday! As your present, I give you my Picasso painting, which is in my Chicago bedroom. Love, Dad." Jeb never took possession of the painting and Ron kept it in his home. Ron died last week, survived by Carla, his wife, and Jeb. A court will most likely find:

A Jeb has the best claim to the painting because Ron made a valid inter vivos gift.Correct. Ron made an immediate transfer of a property right—a present transfer of a future interest. Ron is not required to manually deliver the painting if manual delivery is impossible or impracticable—as it is when Ron is conveying a future interest and Jeb lives 7,000 miles away in Brazil. Ron's letter is an acceptable form of symbolic delivery. B Jeb has the best claim to the painting because Ron made a valid holographic will.Incorrect. Ron's letter was a valid inter vivos conveyance. There was no testamentary intent, and therefore the letter cannot be viewed as a holographic will. C Carla has the best claim to the painting because Ron's attempt to make a future gift was legally inoperative.Incorrect. A present gift of a future interest is legally operative. Here Ron's intent was to convey an immediate interest in the painting. A future interest (here a vested remainder) is a present property interest (whose possession is delayed until some future time). D Carla has the best claim to the painting because Ron kept the painting, and therefore there was no delivery.Incorrect. Ron's letter served as symbolic delivery of Jeb's future interest. Ron reserved a life estate in the painting in himself and created a future interest in Jeb.

In 2000, Joe purchased a two bedroom condominium, intending to move into the unit when he retired. The condominium was located in a state far from his current residence. Unbeknownst to Joe, Skyla occupied Joe's condominium unit for three years, beginning in 2001. Skyla knew that someone else owned the unit, and often commented to her friends, "Hey, if the owner isn't going to use it, I might as well use it." When Skyla needed to leave the unit in 2004 because of a job transfer, she conveyed all interest she had in the condominium unit as well as all her furniture to a coworker, Danica, for $175,000. Danica occupied the unit for nine more years, at the end of which Joe appeared, having recently retired. The jurisdiction has a ten year statutory period for adverse possession and applies the modern rule re intent. In a suit to quiet title, how should a court rule?

A Joe owns the condominium unit because Skyla knew she did not own the unit.Incorrect. The modern rule is that subjective intent is irrelevant to a claim of adverse possession. The adverse and hostile element is focused on the claimant's actions, not on whether the claimant knew she occupied it in bad faith (or good faith). B Joe owns the condominium unit because neither Skyla nor Danica met the ten year statutory period.Incorrect. Although neither Skyla nor Danica individually met the statutory period, Danica's possession can be tacked onto Skyla's. C Danica owns the condominium unit because there was privity between Danica and Skyla.Correct. Most jurisdictions allow tacking if the successive possessors are in privity. The privity requirement is met when a prior occupant transfers all her rights in the property to her successor. Here Skyla transferred her rights in the condominium unit to Danica and therefore Danica could tack Skyla's three year occupancy. Three years plus nine years equal twelve years - consequently, the ten year statute of limitations has been met. D Danica owns the condominium unit because awarding her the title furthers adverse possession's goal of encouraging the productive use of land.Incorrect. While economic development is a goal of adverse possession, any claimant must meet all required elements for the statutory period. Title is not gained by merely forwarding a goal of the doctrine.

Joshua lives in a beautiful gated community. However, one day last month, Joshua noticed that a valuable Waterford vase he had brought back from one of his trips to Ireland was missing. Lily, a friend of Joshua's son, had stolen the vase. A few days later, Lily sold the vase to Sophia for $2000. Sophia took the vase to a local jeweler, Ryan, to have it appraised. Unfortunately, one of Ryan's salespersons mistakenly sold the vase to Ella, a customer and, serendipitously, a colleague of Joshua's. Last week, Joshua went to a housewarming at Ella's and discovered his vase sitting on Ella's fireplace mantle. Who has better right to the vase?

A Joshua, because he was the original owner of the stolen vase.Correct. Joshua maintains his title to the vase. Because Lily was a thief, no one purchasing from her (Sophia) or Sophia's bailee (Ryan) received any property rights that trumped Joshua's. And when Ella purchased the vase from Ryan, Ella received only the rights that Sophia had - and Sophia had none. B Sophia, because she paid valuable consideration for the vase.Incorrect. U.C.C. 2-403 (1) states that "a purchaser of goods acquires all title which his transferor had or had power to transfer." Since Lily was a thief, she had no title to transfer to Ryan. C Ella, because she was a good faith purchaser from a merchant who does business in that type of goods.Incorrect. U.C.C. 2-403 (2) states that "any entrusting of goods to a merchant who deals in goods of that kind gives him power to transfer all rights of the entruster to a buyer in the ordinary course of business." Ryan could transfer only the rights Sophia had. Sophia had no rights. Therefore, Ella received none. D Both Sophia and Ella as tenants in common, because they both were innocent parties, paid valuable consideration, and dealt with the vase in good faith.Incorrect. Both Sophia and Ella were innocent good faith purchasers. However, their status as good faith purchasers has little significance vis a vis Joshua because Lily (their predecessor) stole the vase.

Zane owns a ten acre tract of land (Forestacre), most of which is undeveloped forest. Zane only occasionally visits Forestacre, usually to fish in a small stream that runs through the southeastern portion of the parcel. Khole owns a small alpaca farm that abuts the northwest boundary of Forestacre. Because of a surveying error, Khole mistakenly thought that he also owned a 100 foot portion of Zane's land that bordered his farm. For the past ten years, Khole has entered this forested area and cut leaf-filled branches to feed his alpacas. Khole also regularly cleared the brush from the area and sometimes cut down trees for firewood. He even put up a small sign that read "Alpaca feeding zone." Zane recently discovered Khole's presence on Forestacre and instituted a quiet title action. Khole counterclaimed, seeking to have title to the 100 foot tract quieted in him. Which of the following is correct?

A Khole met the open and notorious element of adverse possession by his regular use of the 100 foot tract.Correct. Khole's actions taken as a whole provide sufficient notice of his claim of title. The nature and frequency of his use of the land (cutting branches, clearing brush, cutting timber) as well as the clearly posted sign should have made Zane aware of Khole's adverse claim if Zane had made a reasonable inspection of Forestacre. B Khole failed to meet the exclusive possession element of adverse possession because Zane occasionally visited Forestacre to fish.Incorrect. Khole did not share his possession of the 100 foot tract with Zane or the public in general. Zane's presence at the stream on the southeastern portion of Forestacre did not interfere with Khole's possession. C Khole failed to meet the continuous element of adverse possession because there were days, and even weeks, when he was not physically present on the tract.Incorrect. Khole meets the continuous element as long as his temporal presence mirrors that of other owners of like property. Since the tract was forested, unimproved and uncultivated, owners of similar land would likely be on the land at similar intervals. Remember continuous possession does not require continuous occupation or use. D Khole failed to meet the actual possession element of adverse possession because he physically used the tract only intermittently.Incorrect. Actual possession does not require that Khole be on every foot of the land at all times. Given the nature of the tract (unimproved forested land), Khole's acts of dominion and control were sufficient to establish his possession of the tract. [Remember, frequency of use relates to continuity, not to whether the possession is actual.]

Kira opens a new store that sells excellent fresh-baked cookies. She calls the business "Cookies Unlimited" and occasionally runs newspaper ads promoting her cookies. Five years later, Leo opens a competing cookie store across the street, which he calls "Cookies Limited." Experts conclude that the quality of Leo's cookies is low. Kira sues Leo. Who will win the lawsuit?

A Kira, because Leo's poor cookies have tarnished Kira's business name.Incorrect. Kira's mark "Cookies Unlimited" is not a famous mark. B Leo, because customers are unlikely to be confused by his business name.Incorrect. Customers are likely to be confused by the two names because they are quite similar. C Leo, if Kira's business name has not acquired a secondary meaning.Correct. Kira's trademark is descriptive, so it qualifies for protection only if it has acquired a secondary meaning, which did not happen here. D Leo, because Kira did not use her business name in trade or commerce.Incorrect. Kira did use her business name in trade and commerce because she operated under the name and used it in advertising.

Lance owned Greenacre in fee simple absolute. In 1998, Lance conveyed Greenacre to Julian by a valid deed which read "to Julian for life." In 2000, Kobe entered Greenacre and began to adversely possess it. Kobe occupied Greenacre for the next 13 years. In 2013, Julian died. Julian's will provided that his son, Rick, receive all his assets. In 2014, Kobe married Ada and made the following oral declaration: "I give Greenacre to my beloved wife Ada." The statutory period for adverse possession in this jurisdiction is ten years. In 2014, who owns Greenacre?

A KobeIncorrect. An adverse possessor gains only the estate that the owner had. Here Julian only owned a life estate. Therefore, Kobe perfected title to a life estate in Greenacre (measured by Julian's life) in 2010. When Julian died in 2013, Kobe lost the title (in the life estate) he had perfected as of 2010. At that time, Kobe would begin to adversely possess the fee simple held by Lance. And as of 2014, Kobe had not met the statutory period for adverse possession of that interest. B LanceCorrect. Lance regained fee simple title to Greenacre upon Julian's death in 2013. Remember, Lance only conveyed a life estate to Julian, retaining a reversion in himself. Kobe merely adversely possessed the life estate held by Julian. When Julian died, Kobe's title ended and Lance regained the fee simple. C RickIncorrect. Because Julian's adverse possession only established title to a life estate, he had no interest in Greenacre to transfer to Rick. When Julian died, Julian's interest in Greenacre ended. D AdaIncorrect. Kobe had no title to convey to Ada. He lost his life estate when Julian died. Additionally, an individual cannot orally convey an interest in real property. Kobe would have to comply with the Statute of Frauds' requirement that the interest be transferred by a written instrument.

Liana went to a nail salon to have a manicure. There she noticed a small purse resting on a side table by the entrance. Liana picked it up and discovered a large stack of $20 bills inside. She gave the purse and money to Ember, the salon's proprietor, telling Ember to give the items to the purse's owner if she should come back. After four months, Liana returned to the salon. When she discovered that no one had returned to claim the purse and money, Liana demanded that Ember hand over both items to her. Ember refused and Liana brought suit. In whose favor should a court rule?

A Liana, because as finder of lost property she has the best claim to the items against the entire world except the true owner.Incorrect. Given the nature of the items and the location where they were found, it is more likely that they were mislaid (voluntarily placed and accidentally left) rather than lost (involuntarily and unknowingly left). Liana was a finder of mislaid property, not lost property. As such, the owner of the locus in quo has the better property right to the purse and money. In addition, the correct statement of the rule for lost property is that the finder has the best claim against the entire world except a prior possessor (not, "except the true owner"). B Liana, because Ember was her bailee and owed Liana a duty to return the items.Incorrect. Under these facts, no bailment (for the benefit of Liana) was created. Ember has the better right to the items as the owner of the locus in quo in which the mislaid items were found. Liana gained no original right to the items by finding them. C Ember, because she gained title through adverse possession.Incorrect. Although title to personal property items can be gained by adverse possession, no jurisdiction has a statute of limitations as short as a few months. While the statutory period is usually shorter for personal property than for real property, most jurisdictions require the action for recovery be brought within two to six years. D Ember, because she was the owner of the salon in which an individual mislaid the items.Correct. Mislaid items belong to the owner of the locus in quo, not the finder. Ember was the owner of the salon and therefore has better property rights to the purse and money than Liana.

Margaret invents an innovative alarm clock in May 2019. For many months, she continues to work on her alarm clock in her home workshop tweaking minor parts of the invention. She finally finishes her clock, and files for a patent in April 2020. In January 2020, Marc begins working on a new invention and invents the same innovative alarm clock. He files for a patent in March 2020. Who will get the patent? Assume no other facts not stated.

A Margaret, as long as she can prove that she started designing the patent first (such as showing her journal with dated drawings and notes).Incorrect. Margaret's design are irrelevant under the USA's first to file system started in 2013. B Margaret, because she was the first to invent the alarm clock.Incorrect. This was the standard prior to the AIA, now we have a first to file system. C Marc, because he was the first to file for the patent.Correct. Answer C is correct because under the AIA, the first inventor to file for the patent has priority. It does not matter that Margaret was the first to invent, and the prompt does not state that Margaret publicly disclosed the invention prior to filing to bar Marc from receiving a patent. Thus, because Marc filed first, he will obtain the patent. Joint inventors is not an option because Margaret and Marc are two separate inventors and did not work together to create the alarm clock. D Both Marc and Margaret will be joint inventors because Margaret can show that she was the first to invent the alarm clock and Marc was the first to file.Incorrect, Joint inventorship is irrelevant here; they are not co-inventors as they did not work together.

Mary was a sickly individual and was regularly hospitalized. During one of her hospital stays, Mary wrote a valid will leaving all of her property to her boyfriend, Kyle. A few weeks later, Mary became seriously ill and was once again hospitalized. The attending doctors doubted that Mary would survive the night. Her good friend Ruby visited her in her hospital room. Mary called Ruby next to her, reached under her pillow, and placed a small object in Ruby's hand. Mary folded Ruby's fingers over the object and said, "Ruby, I'll likely die tonight and this is for you. It's the most precious and valuable thing I own. It's my great-grandmother's gold brooch. Remember me when you wear it." Observing what had happened, Kyle became upset. He dearly wanted the brooch since it was worth more than $20,000 and, besides, Mary had given it to him in her will. Before he could speak, Mary fell into a deep sleep. Who owns the brooch?

A MaryIncorrect. Mary effectively made a gift causa mortis to Ruby. Even though Mary can revoke the gift before her death (and the gift is revoked automatically if Mary does not die from the existing illness), Ruby received a valid gift causa mortis (Mary made it in contemplation of her imminent death). B RubyCorrect. A gift causa mortis is immediately effective when it is made. Even though it is revocable at any time before death, Mary made a valid gift causa mortis to Ruby. C KyleIncorrect. Kyle has no interest in any of Mary's assets until she dies. Wills are testamentary devices and are only legally operative when the testator dies. Also note, the will does not cover any assets that are no longer in Mary's estate when she dies - any inter vivos gifts and gifts causa mortis are not part of the probate estate. D Mary's heirsIncorrect. Mary's heirs have no interest in the brooch. Indeed, they are only ascertained once Mary dies, and would take assets that passed by intestacy. Here the brooch was validly gifted to Ruby and would not be part of the testate or intestate estates.

Igor opens a series of pretzel stands at airports around the country, calling his business "McPretzel." Each pretzel stand has a large yellow arch on the outside. The McDonald's Corporation holds valid trademarks on the name "McDonalds" and various other product names that begin with "Mc" (such as "McNuggets"). McDonald's restaurants have never sold pretzels, but two company researchers are exploring the idea of serving a hamburger between two pretzels. McDonald's sues Igor. Who will win the lawsuit?

A McDonald's, because its trademark was infringed.Correct. "McPretzel" is confusingly similar to the various "McDonald's" trademarks, especially given the yellow arch outside each McPretzel store. B McDonald's, because "McPretzel" is a functional mark.Incorrect. Whether Igor's mark is functional is irrelevant to liability for infringement of McDonald's trademarks. C Igor, if he has only used "McPretzel" in interstate commerce.Incorrect. There is no special exception for using an infringing trademark in interstate commerce. D Igor, because "McPretzel" is a fanciful mark.Incorrect. Whether Igor's mark is fanciful is irrelevant to liability for infringement of McDonald's trademarks.

Which of the following statements is not correct?

A Most states provide enhanced protection for the adverse possessor whose claim is based on color of title.Incorrect. This is a correct statement. Often the standards for gaining adverse possession are easier to meet if based on color of title. In many jurisdictions, the statutory period may be shortened or the adverse possessor may be able to gain more land than she actually has met the elements for. B Historically, adverse possession has been viewed as a legal tool for encouraging development.Incorrect. This is a correct statement. Adverse possession was a tool often used to promote the productive use of land, reallocating title from a lazy owner to an industrious squatter. C Adverse possession applies exclusively to real property.Correct. This is an incorrect statement. Adverse possession applies equally to both real and personal property. While traditionally real property was the more important category, title to personal property may be equally gained by adverse possession. D Modernly, the majority view is that an adverse claimant's intent is irrelevant.Incorrect. This is a correct statement. Most jurisdictions have abandoned trying to discern the adverse claimant's subjective state of mind. They have discarded both good faith and bad faith standards and focus on the adverse possessor's objective actions, rather than her subjective mental state.

For the past ten years, Ana rented a house she owned to Nia. The rental contract created a periodic tenancy, rent payable on January 1st of each year. From the beginning, Nia demonstrated great hostility toward Ana and often refused to speak with Ana or answer inquiries about the house. Each year, Nia would leave for Europe and spend several weeks visiting museums and touring the Alps. These vacations caused Ana some concern since the house was vacant and susceptible to waste. Shortly after Nia missed making her rental payment for the current year, Ana went to Nia to demand her rental monies. Nia asserted that she did not owe Ana any rent. Nia claimed that she now owned the house because she had adversely possessed it. The jurisdiction has a ten-year statutory period for adverse possession. Which of the following is correct?

A Nia did not gain title as an adverse possessor because her rental agreement was a periodic tenancy rather than a term of years.Incorrect. The type of rental agreement has no impact on the answer. Since Nia was a tenant for the first ten years, she failed to meet the adverse and hostile element during that period. She only begins her statutory period once she denies her tenant status. And this happened only recently. B Nia did not gain title as an adverse possessor because her possession was not adverse and hostile.Correct. In order to be an adverse possessor, all jurisdictions require that the possession be "adverse and hostile" for the statutory period. And all jurisdictions agree that when possession is authorized by the owner it is not "adverse and hostile." A tenant occupies with the consent of the landlord. Consequently, since Nia was a tenant in Ana's house for the previous ten years, she failed to meet this element. C Nia did not gain title as an adverse possessor because she did not meet the continuous element.Incorrect. Nia likely met the continuous element of adverse possession. Even though she took yearly vacations, her temporal connection with the house would almost certainly mirror the possession of other residential owners in the area. D Nia gained title to the house as an adverse possessor.Incorrect. Although Nia met most of the elements for adverse possession (actual possession, exclusive possession, open and notorious possession, and continuous possession), she failed to be "adverse and hostile." All elements must be met for the entire statutory period in order to gain title by adverse possession.

Tom rented a storage container at a commercial facility. Tom placed $8,000 in cash and a diamond necklace in the container. One day, Tom told his brother, Nick, "I want Sam to have the contents of my storage container." Tom gave Nick the key to the container and instructed Nick to give the key to Sam. A few days later, Tom died. In which of the following circumstances was there no effective gift?

A Nick immediately delivered the key (rather than the contents of the container) to Sam.Incorrect. Once Nick delivered the key to Sam, Tom made a valid inter vivos gift to Sam. Tom had donative intent, and constructive delivery was made; we can assume acceptance. B Nick immediately delivered the key to Sam. Tom's will provides: "To my wife, all the contents of my storage container."Incorrect. Once Nick delivered the key to Sam, Tom made a valid inter vivos gift to Sam. Tom had donative intent and constructive delivery was made; we can assume acceptance. In addition, will provisions cannot "undo" gifts already completed before the donor's death. A will controls only those assets remaining in the decedent's estate at the time of his death. C At Tom's request, Nick returned the key to Tom, rather than giving it to Sam.Correct. Even though Tom initially had donative intent, there was no effective delivery. Although the key was placed in Nick's hand, Nick was Tom's agent, not Sam's. Therefore, an element necessary for an effective gift (delivery) was never met. D All of the above.Incorrect. Answers (a) and (b) are both examples of a valid gift. Answer (c) is the only example of an invalid gift.

Amy owned Pearacre, a 100-acre tract of prize-winning pear trees; she lived in the home which was located in the southeast corner of the tract. In 2000, Amy was killed by a falling pear tree. Amy's son Sid quickly found what appeared to be Amy's valid will, which left Pearacre to Amy's niece Nina. Nina immediately moved into the home on Pearacre, and used it as her residence until the present; but Nina has never visited or used any other part of Pearacre. Last week, Sid discovered that the will was invalid due to a technicality and, accordingly, that he should have inherited Pearacre under state law as Amy's closest relative. Assume that the adverse possession period is ten years. Sid sued Amy for a declaratory judgment that he owned Pearacre. How should the court rule?

A Nina owns the home, and Sid owns the rest of Pearacre.Incorrect. Nina did satisfy all of the elements for adverse possession. Because she has color of title (the invalid will), she obtained title to all of Pearacre. B Nina owns all of Pearacre.Correct. By using the home as her residence for more than ten years, Nina has satisfied all adverse possession elements. The invalid will is a form of color of title, so Nina's possession of the Pearacre home gives her constructive possession of all of the property. C Sid owns all of Pearacre because Nina did not act in good faith.Incorrect. Most jurisdictions do not require good faith. Moreover, on these facts it appears that Nina believed in good faith that she owned Pearacre under Amy's will. D Sid owns all of Pearacre for another reason.Incorrect. By using the home as her residence for more than ten years, Nina has satisfied all adverse possession elements.

Mary owns a car wash "Ultimate Carh Wash." The car wash service is painted tiffany blue (same color blue as Tiffany jewelry). The store has a unique design inside and outside with a blue awning and the interior has a counter also shaded blue with a unique layout. Can Mary get trade dress protection?

A No, Mary could not get any trade dress protection for anything. because a car wash is functional.Incorrect. This answer is wrong because Mary could get protection for all aspects of the car wash except if it was functional. See correct answer C. B Yes, but Mary could only get trade dress protection for the use of the color and need not establish secondary meaning.Incorrect. Under Qualitex, color can be protectible if there is secondary meaning (identifying as a source identifier). C Yes, Mary could get protection for the color so long as there is not a likelihood of confusion as to the source and she establishes secondary meaning; she could also get protection for the unique trade dress of the store irrespective of secondary meaning.Correct. Color is protectible if there is secondary meaning. Trade dress of the store is protected if inherently distinctive, even apart from secondary meaning. It is unlikely that Tiffany would go into the car washing business so there would be little likelihood of consumer confusion, although Tiffany could argue that by using the color, Mary is diluting Tiffany's mark. D Yes, Mary could get protection for the color so long as there is not a likelihood of confusion as to the source and she establishes secondary meaning; she could not get trade dress protection for the layout of the store.Incorrect. Color can be protected if it has secondary meaning and is non-functional. Trade dress and design of the store can also be protected if inherently distinctive, there is no secondary meaning requirement for trade design of the store's layout.

Ted works at the county tax assessor's office. One day he notices that no one is paying taxes on a 50-acre parcel of forest land, so he decides to adversely possess the land. He nails four small signs to trees on the land; each sign states: "Property of Ted." Each summer, he camps out on the land for two weeks; and every Christmas he comes to the property to cut Christmas trees for his family and friends. Assume that these activities continue for the 10-year period required for adverse possession and that no one else visits the land during this time. In most states, has Ted acquired title by adverse possession?

A No, because Ted knew he did not own the land.Incorrect. In most states, the adverse claimant's state of mind is irrelevant. Only a minority of states require that the claimant have a good faith belief that he owns the land. B No, because Ted's activities were too infrequent and sporadic to be seen as continuous possession.Incorrect. Even minor and sporadic activities will meet the continuous possession requirement for wild and undeveloped land, as you saw in Gurwit. C No, because Ted's activities would not be seen as actual possession; a reasonable owner would use the land in a more intensive manner.Incorrect. Ted has used the land in a more intensive manner than most owners of forest land would. In particular, his activities were far greater than those which were found sufficient in Gurwit. D Yes, because Ted satisfied the requirements for adverse possession.Correct. The best answer is that Ted has acquired title by adverse possession, like the claimants in Gurwit. He had actual possession because he used the land for vacation purposes and harvesting trees. His possession was exclusive because no one else visited the land. The signs (and perhaps tree stumps) made his possession open and notorious, based on Gurwit. Ted's state of mind was irrelevant under the majority view, as long as he did not have the owner's permission to be on the land. Ted's use of the land was sufficiently continuous because owners of similar forest land would use it less frequently. And his possession continued long enough to meet the statutory period

Movie star Mina is a tenant in Oliver's apartment building. Seeking to sell the building, Oliver places an advertisement in the local newspaper which reads: "$2,500,000. Luxury 3-unit apartment complex. Home of famous movie star Mina. Call 333-3333." Has Oliver violated Mina's right of publicity?

A No, because it is reasonable for Oliver to tell prospective buyers who the tenants are.Incorrect. It is reasonable for the seller to provide the identities of tenants to a buyer at some point in the sales process, but there is no reason to do this in an advertisement. B Yes, because Oliver used Mina's name in order to gain a commercial advantage.Correct. As you saw in White, the common law right of publicity is violated when a person appropriates a celebrity's name or likeness to his advantage, unless the celebrity consents. Here, O seems to be using Mina's name in order to help sell his building, almost as if Mina were endorsing a product. C No, unless Mina objects to the advertisement within a reasonable time.Incorrect. A violation of the right of publicity occurs even if the celebrity does not object. D Yes, but only if Oliver also used Mina's likeness in the advertisement.Incorrect. The right of publicity can be violated by the use of a name alone.

The SSS Company builds a private spacecraft that takes its employee scientists to an asteroid which circles earth. The scientists locate a microscopic organism which is based on mercury, unlike the carbon-based life forms on earth. Research proves that releasing the organism on earth would endanger human life. SSS Company now applies to the PTO for a patent on the organism. Will the PTO grant the patent?

A No, because the organism would endanger human life.Incorrect. As Diamond indicates, the PTO is not responsible for assessing the potential harm that an invention may cause. B No, because the Framers of the Constitution never intended that patentable subject matter extend to things found in outer space.Incorrect. The subjective intent of the Framers is irrelevant; in fact, Congress intended patentable subject matter to extend to "anything under the sun that is made by man." C No, because the SSS scientists did not create the organism.Correct. Diamond notes that patentable subject matter does not extend to "manifestations of..." nature, such as "a new mineral discovered in the earth or a new plant found in the wild." Here, the SSS scientists merely discovered the organism; they did not invent it. D Yes, the patent will be granted.Incorrect. Diamond notes that patentable subject matter does not extend to "manifestations of..." nature, such as "a new mineral discovered in the earth or a new plant found in the wild." Here, the SSS scientists merely discovered the organism; they did not invent it.

Joe, a botanist, spent months in a tropical rain forest searching for plants that might have medicinal value. One day he discovered a red-leafed plant which was unknown to science. Upon his return to the United States, Joe fed leaves from the plant to laboratory animals suffering from malaria. To his amazement, they were immediately cured. Joe then applied to the Patent and Trademark Office for a patent on the plant. Will the PTO issue the patent?

A No, because the plant already existed and thus was not novel.Incorrect. True, the plant already existed, but this is not the best answer. The novelty requirement applies to inventions, that is, creations which are patentable subject matter. The plant is not patentable subject matter because it was discovered, not created. B No, because the benefits of the plant are obvious.Incorrect. The nonobviousness requirement applies to inventions, that is, creations which are patentable subject matter. The plant is not patentable subject matter because it was discovered, not created. C No, because the plant is not patentable subject matter.Correct. Manifestations of nature are not patentable subject matter. Accordingly, as the court noted in Diamond v. Chakrabarty, "a new plant found in the wild is not patentable subject matter." D Yes, because all patent requirements are satisfied.Incorrect. The plant is not patentable subject matter because it was discovered, not created.

Bob counts cards in order to win at casino games. The casino does not like this, so they kicked him out and told him he cannot return. The casino is in a jurisdiction that follows the majority rule. Can the casino exclude Bob from the casino?

A No, so long as Bob was not causing a disruption or intoxicated, the casino could not throw him out.Incorrect. This is the minority rule, as shown in Uston v. Resorts Hotel in New Jersey. B No, under no circumstances can the casino exclude him since the casino is open to the public.Incorrect. Just because the casino is open to the public does not mean that the casino can't throw him out. Even the minority New Jersey rule gives private owners the right to exclude if causing a disruption and so long as not afoul of discrimination statutes. C Yes, so long as the casino did not run afoul of federal or state antidiscrimination mandates, the casino can throw him out.Correct. This is the majority rule as stated in Donovan v. Grand Victoria Casino,which took place in Indiana. This gives a strong right of public proprietors to be able to kick people out. D Yes, but only if Bob was in private parts of the casino.Incorrect. It does not matter where in the casino Bob was.

Ollie is relaxing on a chair in his forested backyard, watching an enormous squirrel bury acorns. Paul, a squirrel hunter, accidentally wanders onto Ollie's property. Seeing this "trophy" squirrel, Paul silently sneaks behind it. Just as Paul is about to grab the squirrel, Ray, a dealer in wild animals, jumps from behind a tree and snatches it with his hands. Ray places the squirrel in a cage and runs from Ollie's backyard back to his shop. When one of Ray's favorite customers, Sean, arrives and marvels at the size of the animal, Ray makes Sean a gift of the squirrel. Who has the best right to the squirrel?

A Ollie.Correct. Even though Ollie never exerted certain control over the squirrel, he had constructive possession of the animal vis-à-vis any trespasser on his land. Both Paul and Ray were trespassers, and both will lose against Ollie. Even though Sean had no notice of Ray's trespass and now exerts certain control over the squirrel, Ollie is a prior possessor and therefore has the best right to the squirrel. B Paul.Incorrect. Paul is a trespasser. In order to discourage trespassing on private land, the law constructs "prior" possession on Ollie's behalf. As a prior possessor, Ollie has the best right to the squirrel. C Ray.Incorrect. Ray is a trespasser. In order to discourage trespassing on private land, the law constructs "prior" possession on Ollie's behalf. As a prior possessor, Ollie has the best right to the squirrel. D Sean.Incorrect. Although Sean had no notice of Ray's trespass, his rights in the squirrel are based on Ray's. Since Ray was a trespasser and thus has less rights in the squirrel than Ollie, Sean likewise has less rights than Ollie.

Carl owns a 10-acre tract of unfenced, rural land, where he lives in a small cabin. Don walks up to Carl's front door to try to sell him a set of encyclopedias. The next day, while researching an endangered species of frog, Ella walks across Carl's land, believing in good faith that it is public land. Who is liable for trespass?

A Only Don.Incorrect. A pedestrian like Don has an implied license to walk up to the front door of a person's home, unless the owner has posted a "no trespassing" sign or otherwise manifested an intention to bar such entry. B Only Ella.Correct. Good faith is not a defense to trespass liability. Ella intentionally entered land that, in fact, was privately owned; this is enough to subject her to liability. C Both Don and Ella.Incorrect. A pedestrian like Don has an implied license to walk up to the front door of a person's home, unless the owner has posted a "no trespassing" sign or otherwise manifested an intention to bar such entry. D Neither Don nor EllaIncorrect. Good faith is not a defense to trespass liability. Ella intentionally entered land that, in fact, was privately owned; this is enough to subject her to liability.

Molly just learned that her application to the Peace Corps was approved and that she would be leaving for Mongolia in two days. She jumped into her car and drove the four miles to her best friend's (Josie's) apartment. After telling Josie the good news, Molly exclaimed, "Josie, I want you to have the silver Couture watch I am wearing and the Xbox One console that is in my apartment." Molly took a paper towel from Josie's kitchen and wrote, "I, Molly, make an irrevocable gift of my watch and Xbox One console to my best friend Josie." When she had finished writing, Molly gave the paper towel to Josie. She also simultaneously shouted, "Josie, I also want you to have my car. Here are the keys. Catch!" Molly threw the keys to Josie (which landed next to Josie's feet). Molly made a valid gift of which item(s)?

A Only the watch and the Xbox One.Incorrect. Manual transfer of items that practicably can be handed over is required. Therefore, the watch must be physically handed to Josie for the gift to be completed. Molly did not hand her watch to Josie and therefore did not make a valid gift of it. (The Xbox One was validly gifted.) B Only the watch and the car.Incorrect. Manual transfer of items that practicably can be handed over is required. Therefore, the watch must be physically handed to Josie for the gift to be completed. Molly did not hand her watch to Josie and therefore did not make a valid gift of it. (The car was validly gifted.) C Only the Xbox One and the car.Correct. Symbolic delivery is allowed if manual delivery is impracticable or impossible. Since the Xbox One console was in Molly's distant apartment, manual transfer was impracticable. Consequently, the written note was an acceptable form of delivery. Likewise, constructive delivery is allowed where manual delivery is impracticable or impossible. Cars are too heavy to physically hand over. Providing Josie with the keys gave her access to the car and therefore satisfied the delivery requirement. Molly made a valid gift of both the Xbox One and the car. D The watch, the Xbox One, and the car.Incorrect. Manual transfer is required for any items that practicably can be handed over. Therefore, the watch must be physically handed to Josie for the gift to be completed. Molly did not hand her watch to Josie and therefore did not make a valid gift of it. (The Xbox One and the car were validly gifted.)

Oscar owned a 50-acre tract of desert, where the only viable economic use was grazing goats for six weeks each spring. Because Oscar was a committed environmentalist who wanted to preserve the natural vegetation on his land, he built a ten-foot-high fence around the parcel in order to exclude animals. Seven years ago, Fiona broke a large gap through the fence, and brought in her flock of 1,000 goats to feast on the spring vegetation for six weeks. Oscar first noticed the gap in July, and repaired it. This same pattern of fence breaking, grazing use, and fence repair occurred every year thereafter until the present. Assume that the period for adverse possession is five years. Fiona sued Oscar for a declaratory judgment that she had obtained title by adverse possession. Who will win the lawsuit?

A Oscar, because Fiona did not have open and notorious possession.Incorrect. Fiona's repeated destruction of the fence and the obvious damage to vegetation caused by grazing goats should have put Oscar on notice of Fiona's use. B Oscar, because Fiona did not have actual possession.Incorrect. The nature of the required use for actual possession is dictated by the character, location, and nature of the land; here, the only viable use is goat grazing, and Oscar's desire to preserve the environment is irrelevant. C Oscar, because Fiona did not have continuous possession.Incorrect. The required continuity is dictated by the character, location, and nature of the land, and here six weeks of grazing is all the land will permit. D Fiona because she meets all by adverse possession requirements.Correct. Fiona has met all the elements for adverse possession in most states.

Jack bought a one-of-a-kind gold carbon-framed Bianchi bicycle. Unfortunately, Owen immediately stole Jack's bicycle. Owen rides the bicycle to work each day, locks it outside of his work in plain view, and stores it in the front window of his house. If this jurisdiction applies the discovery rule to the adverse possession of chattels, the statutory period begins when:

A Owen stole the bicycle.Incorrect. Under the discovery rule, the statute of limitations will not begin to run as long as Jack diligently seeks the recovery of his bicycle. B Jack discovers, or by exercise of reasonable diligence should have discovered, the location of the bicycle or the identity of Owen.Correct. As long as Jack continues to use due diligence to recover his bicycle, the statutory period will not begin. The statute of limitations commences once Jack discovers where his bicycle is located or that Owen possesses it. C Owen begins to meet all the elements for adverse possession.Incorrect. The discovery rule shifts the focus from the adverse possessor's conduct (Owen's) to the owner's conduct (Jack's). The focus is not on whether Owen has met the requirements of adverse possession, but on whether Jack has acted with due diligence in pursing his bicycle. D Never, because Owen was a thief.Incorrect. Thieves can adversely possess personal property as long as they meet the requirements. In a discovery rule jurisdiction, the statutory period begins when Owen discovers, or by exercise of reasonable diligence should have discovered, the location of the bicycle or its current possession by Owen.

Zena, a first-year law student, develops the concept of a new reality television show called "Living the Law," which involves following six students through their first year of law school as they learn the law, go to class, and socialize with other students. During the final 15 minutes of each episode, the students would compete against each other in answering "Socratic" questions on legal issues posed by the show's host. Zena calls her friend Paul, a television show producer, and discusses the concept. Paul shows no interest. However, a few month later Zena learns that Paul had produced a new reality show called "Learning the Law," which is almost identical to Zena's concept. Zena confronts Paul, who admits that the new show is based on Zena's concept. Zena sues Paul. Who will win the lawsuit?

A Paul, because he did not mviolate any of Zena's intellectual property rights.Correct. The general rule is that ideas are not protected, and are thereby "as free as the air to common use," unless some special exception applies. No exception applies here, so Zena has no property rights in the idea of the show. Because Zena never used the name for the show in trade or commerce, she has no trademark in the name, even if the names were confusing similar, which they are not. The idea for the show is not patentable subject matter. B Zena, because Paul infringed Zena's trademark in the name of the show.Incorrect. Zena has not used the title of the show in trade or commerce, so no trademark exists. C Zena, because Paul's show violated Zena's copyright in her show.Incorrect. Copyright law protects the manner in which an idea is expressed, not the idea itself. D Zena, if she was able to obtain a patent for the show.Incorrect. Even if the PTO issued a patent, it would be invalid due to the lack of patentable subject matter.

Pedro was a successful professional baseball pitcher due to his unusually-shaped right hand. During a game, Pedro's right hand was struck by a hard-hit baseball, which almost severed it from his arm, leaving it hanging by only a bit of skin. Pedro was taken to the hospital unconscious, where doctor Darla removed the hand. One hour later, Darla transplanted the hand onto the arm of another patient. Pedro sued Darla for conversion. Under Moore v. Regents of the University of California, who will win the lawsuit?

A Pedro, if the hand could have been successfully reattached to his arm.Correct. This is the best answer because of the key factual differences—and thus policy differences—between this situation and Moore. Notice that Pedro was unconscious after being struck by the ball, so he had no idea that he was even having an operation, unlike Moore who knew his spleen was to be removed. Most importantly, imposing conversion liability here does not impair medical research (there is no indication that Darla is doing such research), so the key policy concern that underpins Moore is absent. B Pedro, because his hand is unique.Incorrect. Although the Moore majority discussed whether Moore's cells are unique, this point was not central to the result. C Darla, because liability would impair important scientific research.Incorrect. There is no evidence that Darla was engaged in important scientific research, a situation quite different from Moore. D Darla, if she acted in good faith.Incorrect. Conversion is a strict liability tort, so Darla's good faith is irrelevant.

Barry purchased a fee simple absolute in an oceanfront home on a large lot. The deed Barry received from the seller specified that his lot extended seaward all the way to the "low tide line." In checking title records, Barry learned that the state had originally owned his lot and the surrounding region, and that when the state first conveyed the lot to a private owner in 1872, the deed specified that the lot extended to the "low tide line." Barry posted "no trespassing" signs along the seaward side of his lot. Last Saturday, Rita and Terry walked onto the portion of Barry's lot between the high tide line and the low tide line; they spent the entire day there, picnicking, sunbathing, and swimming. Which of the following is most likely?

A Rita and Terry are liable for trespass because they intentionally entered Barry's land.Incorrect. In most ocean states, the public trust doctrine entitles members of the public to use portions of ocean beaches below the mean high tide line, even if the state has conveyed title to these areas to private owners. B Rita and Terry are liable for trespass unless Barry's signs were underwater at the time and could not be seen.Incorrect. Trespass is a strict liability tort, so whether Rita and Terry had notice is irrelevant. The key here is that they were entitled to use the beach under the public trust doctrine. C Rita and Terry are not liable for trespass because they did not damage Barry's land.Incorrect. Trespass liability attaches even if the trespasser does not cause damage to the owner's property. D Rita and Terry are not liable for trespass because they had a privilege to enter Barry's land.Correct. In most ocean states, the public trust doctrine entitles members of the public to use portions of ocean beaches below the mean high tide line, even if the state has conveyed title to these areas to private owners. Rita and Terry were entitled to use this area.

A scenic five-mile stretch of the Bolana River passes right through the middle of a 20,000-acre ranch owned by Opal. Opal posts large "NO TRESPASSING. NO RIVER RAFTING" signs on the river shore where her property line begins. Rita now wants to float down the river in her raft, passing through Opal's ranch. Which of the following is correct?

A Rita has the right to raft through the ranch due to the public trust doctrine.Correct. National Audubon Society cites Marks for the proposition that the public has a right "to use for boating...the navigable waters of the state." Rafting is a form of boating, and the river is presumably navigable if Rita can raft down it. (Of course, the precise scope of the public trust doctrine differs somewhat from state to state.) B Rita has the right to raft through the ranch if a state agency gives permission.Incorrect. The river is protected under the public trust doctrine. The state acts as the trustee of the trust for the public without any special use permission required. C Opal has the right to prevent Rita from rafting because Opal owns the ranch.Incorrect. The river is protected under the public trust doctrine. The trust supersedes Opal's private property rights. D Opal has the right to prevent Rita from rafting because this may damage fragile wetlands.Incorrect. The river is protected under the public trust doctrine. The state is obligated to protect the trust, not property owners whose lands adjoin public trust property.

Rita notices a rare and valuable butterfly sitting on a leaf in a public park. As Rita moves forward with her hands outstretched to grab the butterfly, it begins to fly away and a brisk wind quickly pushes it 100 feet away from Rita. Susan springs up from behind a bush, nets the butterfly, and refuses to give it to Rita. Under which circumstances below would Rita own the butterfly?

A Rita injured the butterfly when trying to grab it.Incorrect. Under Pierson, a person who merely wounds a wild animal does not obtain title to the animal. B Rita would have caught the butterfly without Susan's intervention.Incorrect. The Pierson majority holds that property rights in a wild animal are acquired only through capturing or killing the animal. The butterfly is wild. Rita did not capture or kill it, so Susan's intervention is irrelevant. C Susan acted in bad faith, to prevent Rita from catching the butterfly.Incorrect. The Pierson majority holds that property rights in a wild animal are acquired only through capturing or killing the animal. The butterfly is wild. Rita did not capture or kill it, so Susan's bad faith is irrelevant. D Rita threw a rock that mortally wounded the butterfly before Susan netted it.Correct. The Pierson majority holds that property rights in a wild animal are acquired only through capturing or killing the animal, which seems to include mortal wounding. The butterfly is wild; Rita mortally wounded it, thus acquiring title, before Susan captured it.

In 1999, Rosa entered Greenacre, a five acre parcel owned by Dana. Rosa met all the elements for adverse possession for the next 15 years. Dana was mentally incapacitated during the first three years of Rosa's occupancy. In 2014, Dana entered Greenacre and told Rosa to leave. Rosa brought suit to quiet title. The statute of limitations for this jurisdiction is 15 years. A state statute also suspends the running of the statutory period until the disability is removed. Which of the following is correct?

A Rosa is the owner of Greenacre because she met all required elements for the statutory period.Incorrect. If an owner is suffering under a disability when an adverse possessor enters, the disability may affect the running of the statutory period. Here the statute was tolled during the first three years. Rosa could only gain title in 2017, at the earliest (2002 + 15 years). B Rosa is the owner of Greenacre because Dana had regained mental capacity by the time the statutory period ended in 2014.Incorrect. The fact that Dana was mentally competent in 2014 and had been mentally competent for the past 12 years will not negate the statutory provision that suspends the running of the statutory period. Here the 15 year statute of limitations only began once Dana became mentally competent in 2002. C Dana is the owner of Greenacre because Rosa did not meet all the required elements for the requisite statutory period.Correct. In this jurisdiction, the statute of limitations is suspended until the disability is removed. Because Dana regained mental capacity in 2002, the 15 year statute of limitations only began at that time. Therefore, Rosa had not gained title in 2014. The earliest she could gain title would be 2017. D Dana is the owner of Greenacre because title based on adverse possession can never be gained if the owner was disabled when the adverse claimant entered the property.Incorrect. Even if an owner is disabled when an adverse possessor enters, an adverse possessor can still gain title. Existing disabilities may result in the lengthening of the statutory period, but do not permanently negate an adverse claimant's ability to obtain title.

Leonard owned a house which he rented to Tyler for a one-year term. In which of the following situations did a trespass occur?

A Sarah, a salesperson, walks from the public sidewalk across the front yard and up to the door to the house.Incorrect. A salesperson has an implied privilege to walk up to the front door of a residence unless a sign specifies otherwise. B Earsplitting noise from Fred's adjacent factory fills the house.Incorrect. Trespass liability extends only to persons or physical things that enter the land of another. Noise has no physical existence; it consists of waves, not particles. C Leonard enters the house in Tyler's absence to show it to a prospective buyer.Correct. As a tenant, Tyler is normally entitled to exclusive possession of the house. Therefore, Leonard's entry into the house without Tyler's permission is a trespass. D Tyler's nephew Ned walks into the house without advance notice.Incorrect. A family member normally has an implied privilege to enter property occupied by another family member.

Ethan threw his large fishing net into unowned waters of the Pacific Ocean. After several hours, four large fish swam into the net. While it was not absolutely impossible for fish to escape Ethan's net, under normal circumstances only a few, if any, fish escape. Emma also was fishing in the area on her own boat. Just as Ethan was about to raise his net and bring the fish onto his boat, Emma reached over the net and with a large spear (never touching the net) stabbed two of the more prized fish and lifted them into her boat. In a suit to regain the fish, Ethan will be...

A Successful, because the fish were safely secured in Ethan's net.Correct. The law does not require absolute certainty against the possibility of escape. Given that the probability of escape was minimal, Ethan demonstrated the degree of certain control required by the rule of capture. B Successful, because Emma's spear entered waters controlled by Ethan.Incorrect. Ethan's introduction of his net into the Pacific Ocean did not give him any property rights to the waters. They remained unowned. C Unsuccessful, because Ethan could only own the fish if he brought them onto his boat.Incorrect. The rule of capture provides that an individual acquires a property right in a wild animal immediately when he brings it under his certain control. By confining the fish in his own private enclosure, Ethan gained ownership of the fish. He did not need to bring them onto his boat. D Unsuccessful, because the fish could escape Ethan's net.Incorrect. Certain control does not mean absolute control. Even though there is a small possibility that a fish might escape Ethan's net, the rule of capture is satisfied even when there exists some (minimal) risk of escape.

In which of these examples has Wendy successfully given her pen to Vic, her partner? Wendy calls Vic into Wendy's bedroom.

A Taking the pen from her heavy desk, Wendy hands the pen to Vic and declares, "I want you to have my pen when I die. Keep it safe."Incorrect. Wendy has no present intent to make an immediate transfer. Wendy wants to transfer the pen only upon her death. B Pointing to her heavy desk, Wendy declares, "I want you to have my desk and everything in it." Inside the desk is the pen.Incorrect. Here there is no delivery. The pen, which is capable of manual transfer, stays in the desk. Manual delivery is required if it is possible and practicable. C Pointing to her heavy desk, Wendy declares, "I want you to have my desk and everything in it." Wendy gives Vic a key that unlocks the desk. Inside the desk is the pen.Incorrect. Here there is no delivery. Wendy has only made a valid delivery of the desk. Since it is impracticable to manually transfer a heavy desk, constructive delivery (giving Vic the key that unlocks the desk) is acceptable. However, the pen (which is capable of being manually delivered) must be manually transferred to Vic. D Distracted by the television, Vic does not see Wendy place the pen in Vic's back pocket, although he does hear Wendy declare, "I want you to have my pen."Correct. Wendy manually transferred the pen to Vic and had the simultaneous intent to gift the pen to him. Acceptance of a valuable item such as a gold pen is presumed.

Caroline, a musician, is working on a new song. She writes out the lyrics and the piano sheet music for her song. She only works on this song in her studio located in the basement of her house. Because of the expensive equipment down there, Caroline locks the studio whenever she is not in there. She has never moved the sheet music or lyrics away from the locked studio. Before Caroline releases her song, she notices a music artist, Dylan, posted a video of himself performing a song on Instagram. Dylan claims his song is an original. Dylan's song has a lot of the same musical characteristics as Caroline's song, such as the piano chords featured during the chorus and lyrics. Caroline sues Dylan for copyright infringement. What is the likely outcome?

A The court will find that there was not copyright infringement because there is no evidence that Dylan had access to Caroline's work. Correct. Answer A is correct because in order to establish copyright infringement, Caroline must show direct evidence or circumstantial evidence of infringement. Here, there is not direct evidence of infringement, so we must look at circumstantial evidence of (1) access to the work; and (2) the similarity between the two works. Though the works might be similar, there is not enough evidence to show that Dylan had access to Caroline's work. The prompt mentions that Caroline works solely in her basement studio that is locked. It also mentions she has never brought materials of her sheet music or lyrics outside of the basement. Thus, it is nearly impossible to establish that Dylan, a random artist on Instagram, has access to her work. Also, it does not matter that Caroline did not file for copyright protection, because she did not need to (B). Similarities are not enough to establish circumstantial evidence of copyrighted work (C). And, Dylan does not have to check to make sure others did not write a song similar before releasing his original song (D). B The court will find that there was not copyright infringement because Caroline never filed for a copyright.Incorrect. You stil have copyright protection even if you don't file for a copyright. C The court will find that there was copyright infringement because of the similarities between the two works.Incorrect. You need to show both access and substantial similiarity. D The court will find that there was copyright infringement because though Dylan did not have access to Caroline's song, he should have checked to see if other artists created a similar song before releasing his own song.Incorrect. Dylan does not have to do this.

Which of the following is not true in most states about a gift causa mortis?

A The gift must be made in contemplation of imminent death.Incorrect. This statement is correct. A gift causa mortis is a gift of personal property made by a living person in contemplation of death. B If the donor recovers, the gift is automatically revoked.Incorrect. This statement is correct. While a donor may revoke a gift causa mortis at any time before her death, in most states it is revoked automatically if the donor does not die. C The gift can be made for any kind of property.Correct. This statement is not correct. A gift causa mortis cannot be made of real property. It applies only to personal property. D The gift becomes irrevocable upon the donor's death.Incorrect. This statement is correct. While a gift causa mortis is revocable at any time before death, once the donor dies it becomes irrevocable.

Mark Twain published his novel The Adventures of Tom Sawyer in 1876. Last year, Alma created a new book called Tom Sawyer and the Zombies. Alma's book copies about 150 pages of text from the original Twain book, but adds 50 pages of new material, inserted at different places into the Twain text, to add a zombie story line to the classic story. Twain's heirs sue Alma. Who will win the lawsuit?

A The heirs, because Alma infringed the copyright in Twain's novel.Incorrect. Any work which was "published" in the U.S. before 1923 is in the public domain, and anyone may use it freely. The copyright in Twain's novel expired long ago. B Alma, because she added her own material to Twain's novel.Incorrect. Even a copier who adds some new material to a copyrighted work is still liable for infringing that work if a substantial amount of the work was taken, as is the case here. C Alma, because she can successfully assert the fair use defense.Incorrect. Twain's novel is in the public domain, so there is no need for Alma to raise the fair use defense. Even if the work were still copyrighted it seems unlikely that Alma could win a fair use defense. Alma's novel was written for profit; and she took a substantial amount of the original work. D Alma, because Twain's novel is in the public domain.Correct. Any work which was "published" in the U.S. before 1923 is in the public domain, and anyone may use it freely. The copyright in Twain's novel expired long ago.

Lisa, a poet, was invited to participate in a poetry competition sponsored by the American Poetry Society ("APS"). She spent three weeks working out the text of a new four-line poem in her head, until the poem was complete and she could recite it from memory. Lisa's recitation of her poem won the first prize at the APS competition. Two days later, however, Lisa was startled to see a copy of her poem posted on the APS website. Upon inquiring, she learned that an APS employee had videotaped her recitation of the poem without her consent and transcribed the poem from that source. If Lisa sues APS for copyright infringement, which is the most likely reason that Lisa will lose?

A The poem was not fixed.Correct. One of the copyright requirements is that the work must be fixed in a tangible medium of expression. Here Lisa never fixed her poem by writing it down on a piece of paper or otherwise. Either the videotape or the website posting would satisfy the fixation requirement if done with her consent; but neither is sufficient because she did not consent. B The poem was not sufficiently creative.Incorrect. A four-line poem, while short, would probably be viewed as sufficiently creative because the applicable threshold is quite low. Only a "creative spark" is needed, even if the work is "crude, humble, or obvious," according to Feist Publications v. Rural Telephone Service. C The poem was not a work of authorship.Incorrect. A poem is a literary work, which is listed as a type of work of authorship in the Copyright Act. D Lisa failed to give proper notice that the poem was copyrighted.Incorrect. Notice is not required for copyright protection to arise. If Lisa had fixed her poem in a tangible medium of expression she would have obtained a copyright.

In which of the following situations has Thomas made a valid gift of his tools (two screwdrivers, a wrench, and a hammer) to Adam?

A Thomas was in his garage working on his car that would not start. Everything he tried did not work. Adam, Thomas' neighbor, came over to help. Adam was a mechanic for a major car dealership. Adam grabbed a pair of pliers and quickly fixed the problem. Frustrated, Thomas pointed to his large heavy tool box and exclaimed, "I hate this! I'm no mechanic and never should try to be one. Adam, here is the key to my tool box. It's yours and also every tool inside it!" Thomas tossed the key to Adam. The tool box contained two screwdrivers, a wrench, and a hammer.Incorrect. While constructive delivery (handing the key) may be permitted for a large heavy object (the tool box is impracticable for manual delivery), the tools require manual delivery since they can be easily physically handed over. B Thomas called Adam on the phone, asking Adam to come over to his garage and saying he wanted to give Adam some of his tools. When Adam arrived, Thomas pointed to a group of his tools (two screwdrivers, a wrench, and a hammer) lying on his workbench and said, "My friend, those tools are yours. I gladly give them to you."Incorrect. Merely pointing to the tools is not sufficient. They must be manually transferred to Adam. C Yesterday Adam asked Thomas if he could borrow a few of his tools. Thomas happily loaned Adam two screwdrivers, a wrench, and a hammer. Today was Thomas's birthday and his husband gave Thomas a complete set of new tools. Delighted by this present, Thomas called Adam and said, "Keep the tools. They're yours. I just got a brand new set of tools for my birthday."Correct. Delivery is deemed complete if the gifted items are in the possession of the donee. The law does not require the donee to briefly return the items to the donor for the mere purpose of requiring him to redeliver them. Therefore since Adam was already in possession of the tools, the gift was complete when Thomas told him to "keep them." D Adam frequently asked Thomas if he could borrow a few of Thomas's tools. After lending his tools to Adam for on numerous occasions, Thomas exclaimed, "Adam I think you need my tools more than I do! I want you to have my wrench, hammer, and these two screwdrivers when I die." Thomas placed the tools in Adam's hands and said, "Take good care of them!"Incorrect. There is no intent to make an inter vivos gift. Although the tools were placed into Adam's hands, Thomas's intent was to transfer ownership only when he died. Thomas would need to write a will if he wanted to make a testamentary gift.

Tripp was walking along a trail on unowned land when he noticed a shiny object on the ground. He picked up the object, which turned out to be a lost Rolex watch. Noticing that the Rolex was not working, Tripp took it to a local jeweler to be repaired and explained to the jeweler that he found the lost watch while on his daily walk. The next day Tripp went to the jeweler to retrieve the watch. The jeweler refused to return the Rolex, simply saying "Hey, it isn't your watch." Tripp brings an action in replevin. Which statement is correct?

A Tripp is the owner of the watch because he was first to find the lost item.Incorrect. As a finder, Tripp is a bailee of the watch, not the owner. B Tripp will be successful in his suit although he is not the owner of the item.Correct. The basic rule of finders law is that a finder of a lost item has better possessory rights to the item against everyone but a prior possessor. Here Tripp possessed the watch before he gave it to the jeweler. Tripp's rights are therefore paramount to the later-possessing jeweler. Additionally, finders are bailees, not owners. C Tripp incorrectly sued in replevin. He should have sued in trover if he wanted the watch returned.Incorrect. Replevin is a lawsuit to recover possession of personal property (which is exactly what Tripp wants). Trover is a common law action that seeks monetary damages for the wrongful taking of personal property. D The jeweler has better possessory rights to the watch since Tripp was not the true owner and the watch is now in the jeweler's possession.Incorrect. Property rights are relative. Even though Tripp was not the true owner of the Rolex, he had better property rights to the watch than the jeweler because Tripp was a prior possessor of the lost item.

After 10 years of extensive research, Wilma discovered that Shakespeare was actually a woman. Wilma wrote and published a 550-page book called The Truth About Shakespeare that documented her conclusion. Six years later, Mike writes a screenplay for a fictional movie with the same theme; he incorporated some of the historical incidents set forth in Wilma's book into the screenplay. Mike's screenplay is used to produce the award-winning movie Shakespeare Revealed. Wilma sues Mike. Who will win the lawsuit?

A Wilma, because her copyright was infringed.Incorrect. Copyright protects the expression of an idea, not the idea itself. A fact (such as Shakespeare's gender) is viewed as an unprotected idea. B Mike, because Wilma's book was not sufficiently original.Incorrect. The expression in Wilma's book is presumably original enough to merit copyright protection. But Mike used only the unprotected facts in the book, not Wilma's expression of those facts. C Mike, under the fair use defense.Incorrect. Copyright protects the expression of an idea, not the idea itself. A fact (such as Shakespeare's gender) is viewed as an unprotected idea. Mike will win without asserting the fair use defense. D Mike, because his screenplay did not infringe Wilma's copyright.Correct. Copyright protects the expression of an idea, not the idea itself. A fact (such as Shakespeare's gender) is viewed as an unprotected idea.

Xao owns a herd of domesticated horses that roam free on his ranch. One day, one of Xao's horses wandered onto the adjoining property owned by Yarin. Yarin decided he wanted to ride the horse and began to pursue it with a rope. Just as Yarin was about to lasso the horse, Zeb, who was trespassing on Yarin's land, jumped out and wrestled the horse onto the ground. Hearing the commotion, Xao ran over and demanded the horse's return. However, both Yarin and Zeb also claim they own the horse. Who owns the horse?

A Xao owns the horse because it is a domesticated animal and belongs to Xao, even though it wandered onto Yarin's land and even though a subsequent party exercised control over it.Correct. A prior owner retains title to her domestic animals (who have animus revertendi) even if they roam at large and are captured by a subsequent party. B Yarin owns the horse because it was captured on his property, and Yarin is considered to have constructive possession over it.Incorrect. While Yarin could own the horse if the horse were a wild animal (ferae naturae), horses are domesticated animals and thus not controlled by the rule of capture. C Yarin owns the horse because he gained certain control over the horse by pursuing it.Incorrect. Even if the horse were a wild animal and its ownership determined by the rule of capture (it is not), Yarin would not gain certain control by merely pursuing the horse. Certain control is gained by "pursuit + mortal wounding," not by pursuit alone. D Zeb owns the horse because he gained certain control over the horse by capturing it.Incorrect. While Zeb may have a better right to the horse than Xao if the horse were a wild animal (ferae naturae), horses domesticated animals and thus not controlled by the rule of capture. Additionally, since Zeb trespassed on Yarin's land, Yarin would have better rights to the horse than Zeb.

Xin, Yara, and Zayne were roommates in law school. After the summer of their first year, Xin and Zayne stayed on campus while Yara interned at a foreign law firm in Australia. On Yara's birthday, Xin wrote to Yara: "Dear Yara: I want you to have my painting, which has been delivered to your parent's home and awaits your return. Xin." Yara wrote back saying: "Dear Xin: Thank you for such a generous gift. I accept! Yara." A few days before Yara returned from her internship, she learned that Zayne objected to Xin's action, convinced Xin that Yara had no rights to the painting, and counseled Xin to get the painting back. Who owns the painting?

A Xin, because Xin never manually transferred the painting to Yara.Incorrect. The requirement of manual delivery is completed when the donor manually transfers the painting to the donee's agent (here, Yara's parents). B Xin, because he revoked his gift before Yara gained possession of the painting.Incorrect. Generally, inter vivos gifts are not revocable. Once Xin met the elements for a valid inter vivos gift (intent, delivery, and acceptance), Xin could not revoke it. C Yara, because the chain of correspondence between Xin and Yara serves as an enforceable contract.Incorrect. Xin intended to make a gift of his painting. A gift is not a contract. A gift, by definition, has no consideration, and without consideration there is no contract. D Yara, because Xin parted with dominion and control when the painting was delivered to Yara's parents, who acted as Yara's agents.Correct. Xin made a valid inter vivos gift to Yara: Xin's letter demonstrates donative intent, delivery is accomplished when Xin parts with dominion and control and gives the painting to Yara's parents (Yara's agents), and Yara expressly accepts. Generally, inter vivos gifts are irrevocable.

Adam creates a western movie Cowboys of the West. The setting takes place in a Western style town in the 1850s; the protaganist is a cowboy, the antagonist is an evil villain; the cowboy rides fast horses, wears a pistol on the hip, protects the town from villans, and has a sidekick who helps fight crime. The villain wears a dark mask. Does Adam have a copyright in the elements of Cowboys of the West?

A Yes, Adam could get a copyright for the storyline and the characters.Incorrect. It is not original enough. B Yes, Adam could get a copyright so long as he wrote the story down.Incorrect. It is likely he could not get a copyright at all, so his writing it down is irrelevant. C No, Adam could not get a copyiright because the ideas he describes are typical of all cowboy movies.Correct. the scenes a faire doctrine allows copying of incidents, characters, or settings which as a practical matter are typical of a treatment of a given topic. Here, the things mentioned are typical of all cowboy movies and it would be hard to write a cowboy movie without them. D No, Adam could not get a copyright because he copied his ideas from John Wayne movies.Incorrect. This may be true, but it is irrelevant to the question. There is no suggestion of copying, and regardless, the scenes a fair doctrine woudl mean that many of the elements are not copyrightable anyway, by Adam or whoever created other western cowboy movies.

Lucy writes a biography about a famous artist Billy BonaFide. In writing the book, she quotes from six unpublished journal entries of Billy. The diaries were never for sale. The total quotations constitute 1% of the letters of the journal, and Lucy uses them to demonstrate factual occurrences. Lucy publishes her biography and makes a profit. Did Lucy likely commit copyright infringement?

A Yes, Lucy copied the work, and the fact that she made money is irrelevant.Incorrect. Lucy has a strong fair use defense. B Yes, Lucy copied the work with intent to profit.Incorrect. Lucy has a strong fair use defense. C No, Lucy likely did not commit copyright infringement because she only published direct quotes.Incorrect. However, the fact that she only published direct quotes is not the reason for why no copyright offense. It is because she has a good fair use defense. D No, Lucy likely did not commit copyright infringement because she likely can mount a strong fair use defense.Correct. Lucy has a strong fair use defense. Factor 1: Purpose is educational; 2) Closer question, quotes may be personal thoughts but quotations indicate factual nature; the fact that they are unpublished weighs in favor of infringement; 3) Lucy uses only a small amount and entries are not substantial to the whole; and 4) given small amount Lucy used and that the diaries were never used for profit indicates a fair use because she is not trying to preempt publication of the diaries for herself.

Ever since the invention of the solar panel, engineers have been trying to develop a building material that captures the sun's energy. They have been seeking to make the entire outer structure of a building from material that will absorb the sun's energy much like a solar panel.One engineer named Byron makes a great discovery: the solar panel itself can serve as the outer structure of a building. That is, Byron discovers that solar panels can be used for the outside walls and roofing of buildings. As a result of this discovery, Byron seeks a patent on the method of making a building from solar panels. While waiting for the issuance of the patent, Byron's method of constructing buildings from solar panels achieves immediate commercial success. Is Bryon's invention obvious for purposes of determining whether he can get a patent?

A Yes, it would be obvious.Incorrect. It is nonobvious. B No, it would be nonobvious because there is no prior art concerning his inventon that could anticipate his invention.Incorrect. The fact pattern describes the prior art, so there is prior art. Also, the answer confuses anticipation and obviousness. Anticipation is when one prior art reference is the same as your invention wheras obviousness combines two or more prior art references. C No, the invention would be nonobvious because the combination of the prior art references would not be obvious to a solar panel building constructor and secondary considerations weigh in favor of nonobviousness.Correct. The combination of prior art references would not be obvious to the PHOSITA, here the PHOSITA would be a person of ordinary skill in the art, and the art here is solar panels. Also, secondary factors weigh in favor of nonobviousness. Engineers have been trying for years without success so if so easy to make why didn't they succeed? Also fact that Bryon's invention was a commercial success weighs in favor of nonobviousness. D Not clear, the invention would be nonobvious because the combination of the prior art references would not be obvious to a lay person and secondary considerations weigh against a finding of nonobviousness. Incorrect. Obviousness is determined by the PHOSITA, which is an ordinary skill of the art, not a lay person. See correct answers as secondary factors - the fact that others failed and commercial success - weight in favor of the invention being nonobvious.

Mike has obtained Kathy's trade secret, a process for manufacturing widgets. He obtained it by bribing an employee of Kathy's company to disclose the process. Mike then posts the info online on hits website about innovative techniques for running businesses. Big Business Inc. sees the info and begins to use the process. Can Kathy sue both Big Business, Inc. and Mike for misappropriation of the trade secret?

A Yes, she can sue both; Mike because he misappropriated it through improper means and Big Business learned of the secret through improper means.Incorrect. She can sue Mike but not Big Business. Big Business learned of it through the improper means of Mike but they themselves did not do anything wrong. B Yes, but she can only sue Mike.Correct. They can only sue Mike. Mike got it through improper means, but Big Business was innocent; Mike disclosed it publicly, and Big Business got it through a public process. C No, she cannot sue either Mike nor Big Business.Incorrect. Mike is liable because he got it through improper means. D Yes, she can sue Big Business but not Mike because Mike did not actually use the trade secret and was not an employee of Kathy's company.Incorrect. Mike is liable because he got the secret through improper means. It does not matter he was not an employee. Big Business is not liable here because they got it from a public disclosure.

Lily creates an advertisement that shows pictures of Tom Hanks and Julie Roberts over clothes that Lily sells. Her store is called Lily's Labels. The ad states: "Wouldn't they look much better in Lily's Labels?" The ad states taht neither Hanks nor Roberts agreed to appear in the ad. Can Lily be sued? Assume California law.

A Yes, she has arguably violated the actors' right of publicity.Correct. Lily has possibly violated the right of publicity in the actors image. See Vanna White case. B No, Lily has not violated the right of publicity since she made clear that neither actor consented to be in the ad so there would be no confusion among consumers.Incorrect. The fact that the ad has a caveat is irrelevant. Right of publiclity could still be violated. Also, this intermixes trademark law and rights of publicity. C No, Lily has not violated the actors' right to publicity because use of the name was a fair use.Incorrect. Fair use is a defense to trademark and copyright disputes, not rights of publicity. D No, Lily has not violated the rights of publicity because there is no such claim as a right of publicity for famous actors. Incorrect. The Vanna White case suggests that there is a right of publicitiy recognized, depending on what the state's statute is.

Jane is a member of the Girl Scouts. She would like to be a member of the Boy Scouts since her best friend John is a Boy Scout. However, the New Jersey chapter of the Boy Scouts says that only boys may be Boy Scouts since girls are not tall enough to climb. Jane and her parents bring suit against the Boy Scouts. Will Jane succeed?

A Yes, under the federal Civil Rights Act as well as New Jersey antidiscrimination laws.Incorrect. Sex is not a protected category under the Civil Rights act. B Yes, but only under the New Jersey antidiscrimination acts; she cannot bring a claim under the federal Civil Rights Act.Correct. The CRA does not have "sex" as a protected category while under the Now v. Little League Baseball, we see that the New Jersey antidiscrimination statute protects discrimination in public accommdations on the basis of sex. C No, becuse the Boy Scotts do not count as a "public accommodation" under either federal or state law.Incorrect. The NOW v. Little League Baseball case establishes that Jane would have a claim in New Jersey. D No, because there is no discrimination on the basis of sex.Incorrect. The federal CRA does not protect sex in public accomodation, but the New Jersey statute does.

Silas was a wealthy entrepreneur and owned several parcels of land around the country. He was particularly fond of Wateracre, a parcel that encompassed four small lakes and several streams. Silas was divorced but maintained close ties with his adult son, Luke. Luke has a daughter, Cora. Silas also was very close to his lifelong friend, Kaleb. Last month, Silas executed a valid deed. The instrument provided that Silas granted Wateracre "to Luke for life, then to my grandchildren for life, then to Kaleb and heirs." Silas died last week. When he died, Silas was 108, Luke was 91, and Cora was 75. This jurisdiction applies the Rule in Shelley's Case and the common law Rule Against Perpetuities. Which of the following correctly identifies the interests created by Silas's will?

A Luke received a life estate, Cora received indefeasibly vested remainder in life estate, and Kaleb received a vested remainder in fee simple.Incorrect. Even if you concluded (incorrectly) that Cora received a remainder, it would not be indefeasibly vested, but vested subject to open (more grandchildren could be added to the class). B Luke received a life estate, Silas's grandchildren received a vested remainder subject to open in life estate, and Kaleb received a vested remainder in fee simple.Incorrect. While this choice identifies the interests initially created, application of the Rule Against Perpetuities makes the grant to Silas's grandchildren void ab initio. Because Silas could have another child before his death (at his age, he would either utilize modern reproductive technologies or adopt!), and that child could have a child (Silas's grandchild), the remainder in that grandchild is void. It might vest more than 21 years after the lives in being at creation (Silas, Luke, and Cora) all die. C Luke received a life estate and Kaleb received a vested remainder in fee simple.Correct. The interest in Silas's grandchildren is void under the Rule Against Perpetuities. There is a possibility that the vested remainder subject to open in Silas's grandchildren will vest more than 21 years after all lives in being (Silas, Luke, and Cora) die: suppose Silas has another child, Lyla. Then Silas, Luke, and Cora die in a plane disaster. Thirty years later, Lyla (not a life in being for purposes of the Rule) has a child, Amy (a grandchild of Silas and not a life in being for purposes of the Rule). Amy's interest will then vest, at a point more than 21 years after the lives in being have died. Because of this possibility of remote vesting, the Rule Against Perpetuities is violated, and the interest (the grandchildren's vested remainder subject to open) is struck. This leaves Luke with a life estate and Kaleb with a vested remainder in fee simple. D Luke received a fee simple absolute.Incorrect. Luke received a life estate. Neither the Rule in Shelley's Case nor the Rule Against Perpetuities void the vested remainder held by Kaleb.

Ron and Jed are joint tenants in Redacre. Ron transfers his interest to Sue. While Jed is on sabbatical in France, Sue takes up exclusive occupation of Redacre. Which of the following is true in most jurisdictions?

A Sue must account to Jed for the fair rental value of Greenacre.Incorrect. The prevailing rule is that a cotenant in possession does not owe any rent to a cotenant out of possession, absent an ouster. When Ron transfers his interest to Sue, Sue and Jed become tenants in common. B Sue must account to Jed for ½ of the fair rental value of Greenacre.Incorrect. The prevailing rule is that a cotenant in possession does not owe any rent to a cotenant out of possession, absent an ouster. When Ron transfers his interest to Sue, Sue and Jed become tenants in common. C Sue must make a contribution to Jed for ½ of the fair rental value of Greenacre.Incorrect. The prevailing rule is that a cotenant in possession does not owe any rent to a cotenant out of possession, absent an ouster. When Ron transfers his interest to Sue, Sue and Jed become tenants in common. D Sue is not liable for any part of the rental value of Greenacre.Correct. The prevailing rule is that a cotenant in possession does not owe any rent to a cotenant out of possession, absent an ouster. When Ron transfers his interest to Sue, Sue and Jed become tenants in common. Since Sue never prevents Jed from using or occupying the premises, there is no ouster.

Oren holds a fee simple absolute. Oren conveys "to Ted for life, then to Ted's first child to graduate from high school and his heirs." (Ted has a 15-year-old son, Ken, presently in the 11th grade.) This jurisdiction applies the common law Rule Against Perpetuities. Which of the following is correct?

A The first child to graduate from high school has a vested remainder in fee simple.Incorrect. While an initial parsing of the conveyance would conclude that the first child to graduate from high school has a remainder, the remainder would be contingent, not vested. It is contingent because, at the time of the conveyance, the child is unascertainable. Only when Ken (or another child of Ted) graduates from high school will the remainder vest. [Note: the contingent remainder is void ab initio because it violates the common law Rule Against Perpetuities.] B The first child to graduate from high school has a contingent remainder in fee simple.Incorrect. An initial parsing of the conveyance would conclude that the first child to graduate from high school would have a contingent remainder. The interest is a remainder because (i) it is capable of becoming possessory immediately upon the expiration of the preceding estate and (ii) it cannot divest any other interest. It is contingent because, at the time of the conveyance, the child is unascertainable. If and when a child of Ted graduates from high school, the remainder will vest. However, because the contingent remainder violates the common law Rule Against Perpetuities, the contingent remainder is void ab initio. C Ted has a fee simple.Incorrect. Ted has a life estate, identified by the words of limitation "for life." While the future interest in Ted's first child to graduate is void, Ted's interest is good. All present possessory estates and future interests in a grantor are deemed vested for purposes of the Rule. D Oren has a reversion.Correct. The words of purchase "to Ted" denote Ted as the grantee of the present possessory interest. The words of limitation "for life" identify the estate as a life estate. The words of purchase "to Ted's first child to graduate from high school" denote the child as the grantee of a future interest. The words of limitation "and his heirs" identify the estate (that will exist if the future interest becomes possessory) as a fee simple. The interest is a remainder because (i) it is capable of becoming possessory immediately upon the expiration of the preceding estate and (ii) it cannot divest any other interest. It is contingent because, at the time of the conveyance, the child is unascertainable. If and when a child of Ted graduates from high school, the remainder will vest. Oren retains a future interest known as a reversion. A reversion will always follow a contingent remainder because someone must hold seisin in the estate at all times. Since a contingent remainder is not certain to vest, the grantor must retain a reversion. Applying the common law Rule Against Perpetuities:(a) Identify the contingent interests: because the conveyance creates a contingent remainder, one must consider whether or not the interest violates RAP.(b) List the lives in being: Ted, Ken, and Oren are alive at the time of the conveyance. Therefore, they are the lives in being.(c) Give birth to any potential afterborns: Ted may have a child, Ned, after the conveyance. Ned is not a life in being for purposes of the rule.(d) Kill off the lives in being and add 21 years: suppose that in 2020, Ted, Ken (who is then 16), and Oren all die. The lives in being are now dead. The "RAP line" becomes 2041.(e) Is there any possibility that the contingent remainder may vest more than 21 years after all lives in being (Ted, Ken and Oren) die (past the 2041 "RAP line")? One possibility: Ted has a child, Ned, who is born in early 2020 (and therefore is not a life in being at creation). Ken, a weak student, never graduated from high school. One day after the birth of Ned, Ted, in his excitement over the news, slips in the snow, falls into Ken and Oren, and the three of them meet an untimely end. The lives in being are now dead. Twenty-two years later (in 2042), Ned, a poor student, graduates from high school, satisfying the condition precedent. His contingent remainder vests more than 21 years after the deaths of the lives in being, and RAP is violated.When a conveyance violates the Rule Against Perpetuities, it does so when it is made, not 21 years later. In the absence of reform tools, the offending interest is deleted. Hence, the contingent remainder in Ted's first child to graduate from high school is deleted. The conveyance now reads: Oren conveys "to Ted for life." Ted has a life estate. Oren retains a reversion (the future interest remaining in the transferor when he grants a vested estate of lesser quantum).

Hal, Sam, and Ted own Greenacre as joint tenants. Hal sells his interest to Xin without notifying either Sam or Ted. One day later, Sam dies intestate as a result of a tragic chain saw mishap, leaving Yap as his only heir. The next day, Ted dies. Ted's will leaves Ted's entire estate to Zoe. Who has what interest in Greenacre?

A Xin and Zoe own Greenacre as joint tenants.Incorrect. Hal's sale of his interest to Xin severs Hal's joint tenancy with Sam and Ted; Xin holds as a tenant in common with Sam and Ted (who still hold as joint tenants vis-à-vis each other). B Xin, Yap, and Zoe own Greenacre as joint tenants.Incorrect. Hal's sale of his interest to Xin severs Hal's joint tenancy with Sam and Ted; Xin holds as a tenant in common with Sam and Ted (who still hold as joint tenants vis-à-vis each other). Additionally, Sam's interest is not descendible to Yap. When Sam dies, his participation interest is removed and Ted gains the entire 2/3 interest by right of survivorship. C Xin and Zoe own Greenacre as tenants in common, in equal shares.Incorrect. When Hal sells his interest to Xin, Hal severs the joint tenancy and Xin becomes a tenant in common with Sam and Ted (who retain their survivorship rights vis-à-vis each other). Xin has an undivided 1/3 share; Sam and Ted have an undivided 2/3 share. When Sam dies, his participation interest is removed, and Ted owns the 2/3 share as a tenant in common with Xin. When Ted dies, Ted's 2/3 interest passes to Zoe. Xin and Zoe are tenants in common, owning fractional shares of 1/3 and 2/3 respectively. D Xin and Zoe own Greenacre as tenants in common, in fractional shares of 1/3 and 2/3.Correct. When Hal sells his interest to Xin, Hal severs the joint tenancy and Xin becomes a tenant in common with Sam and Ted (who retain their survivorship rights vis-à-vis each other). Xin has an undivided 1/3 share; Sam and Ted have an undivided 2/3 share. When Sam dies, his participation interest is removed, and Ted owns the 2/3 share as a tenant in common with Xin. When Ted dies, T's 2/3 interest passes to Zoe. Xin and Zoe are tenants in common, owning fractional shares of 1/3 and 2/3 respectively.

When Bess retired, she purchased a single room school building ("Readacre") in fee simple absolute. There Bess held free reading classes for illiterate adults. Last month, Bess fell and broke her hip. Unable to continue teaching, Bess conveyed Readacre to Dee. The valid deed read, "...to Dee and heirs provided that Readacre is used for teaching reading to illiterate adults, then to my daughter Lucy." Last month, Dee cancelled all classes and converted Readacre into a commercial winery. This jurisdiction applies the common law Rule Against Perpetuities. Which of the following statements is correct?

A Dee has a fee simple absolute in Readacre.Correct. Dee received a fee simple subject to an executory limitation (a fee simple defeasible whose future interest is held by a transferee, rather than by the transferor) and Lucy a shifting executory interest. Since Readacre could be used for teaching reading to illiterate adults for the next several hundred years (and only then the condition breached), the executory interest violates the Rule Against Perpetuities (there is a possibility of remote vesting beyond the lives in being of Bess, Dee, and Lucy). Therefore, it is void ab initio. Because the condition subsequent ("provided that....") is deemed a part of the executory interest (that is void ab initio), Dee is left with a fee simple absolute. B Lucy has a fee simple absolute in Readacre.Incorrect. Lucy has no interest in Readacre. What appeared to be a shifting executory interest violated the Rule Against Perpetuities and therefore was void ab initio. C Lucy's interest violated the Rule Against Perpetuities and therefore Bess has a vested remainder in fee simple absolute.Incorrect. Although Lucy's interest violated the Rule Against Perpetuities, Bess did not gain a vested remainder. No future interest was created. Dee simply received a fee simple absolute. D Lucy's interest violated the Rule Against Perpetuities and therefore Bess regained possession of Readacre when Dee violated the condition.Incorrect. Dee received a fee simple absolute. The condition ("provided that Readacre is used for teaching reading to illiterate adults") is deemed to be part of the executory interest that was void under the Rule Against Perpetuities.

Harriet owned Desertacre, an unfenced and unimproved tract of desert land that covered four square miles, located 20 miles from the nearest town; the land was covered with wild cactus plants. The west side of Desertacre adjoined Highway 47 for one mile. Each year, Harriet visited the land once a year to hike for a few hours. When Harriet died, her will devised Desertacre to her son Ivan. Two years after Ivan received title, Julie, a cactus expert driving past Desertacre, noticed that some of the plants growing there were rare and valuable. She stopped and dug up two cactus plants, each about 10 inches high, which she subsequently sold to a collector. Over the next 11 years, Julie visited Desertacre on nine more occasions. Each time she removed between two and four cactus plants from different portions of the property; the plants she took ranged in height from 7 to 15 inches. Julie sold the plants to collectors. Ivan never walked on Desertacre. But once a year for 14 years after receiving title, Ivan drove past the land slowly on the highway, making a visual inspection. During these inspections, he never noticed any evidence of Julie's activities. The period for adverse possession in this jurisdiction is 10 years. Who owns Desertacre?

A Ivan, because Julie did not have actual possession.Incorrect. A claimant holds actual possession when she uses the land in the same manner that a reasonable owner would, given its character, location, and nature. Because Desertacre is an unimproved tract of desert land far from the nearest town, a reasonable owner would have few uses for it. Most desert land is not used at all. A reasonable owner might use the land for recreation. Here, however, Julie is essentially using the land for cactus farming, which is a more intensive use than a reasonable owner would adopt; she is able to pursue this use because of her expert knowledge about cactus plants, knowledge that most landowners would not have. Because Julie is using the land for a more intensive use than would be expected of a reasonable owner, this element is satisfied. B Ivan, because Julie did not have continuous possession.Incorrect. Continuity is measured by how frequently a reasonable owner would use the land, given its character, location, and nature. A reasonable owner would use remote desert land like Desertacre only on rare occasions; Harriet, for example, visited only once a year. Julie used the land on 10 occasions in 12 years which would probably be viewed as continuous. C Ivan, because Julie did not have open and notorious possession.Correct. A use must be visible and obvious, such that an owner who made a reasonable inspection of the land would become aware of the adverse claim. Ivan did not make a reasonable inspection because he could only see a small part of the land from the highway. However, even if he had made a careful inspection, it is unlikely that he would have noticed any evidence of Julie's activities. Over 12 years, Julie removed fewer than 40 small cactus plants from four square miles of land. Given the huge size of the tract, a reasonable owner would not have noticed that the plants were missing. Although Julie's actions would have left a small hole when each plant was removed, these holes would not have put an inspecting owner on notice due to size of the tract. D Julie, because the adverse possession elements are satisfied.Incorrect. The use was not open and notorious, in that an owner who made a reasonable inspection of the land would not have noticed any evidence of Julie's activities, given the huge size of the tract and the small number of plants she removed.

Twenty years ago, Sam constructed a five-story apartment building on a city lot. Part of the building roof protruded six inches into the airspace over the adjacent lot owned by Lila. Lila now discovers this problem and demands that Sam remove the protrusion. What should Sam do?

A Refuse because he has acquired title to the airspace by adverse possession.Correct. Sam has met all elements for adverse possession in most states. B Refuse because Lila's title does not extend five stories above the ground surface.Incorrect. The surface owner's title does extend upward beyond five stories; it is certainly possible for the owner of a city lot like Lila to "occupy or use" this airspace, just as Sam did on his own lot. C Refuse because he is not in privity with Lila.Incorrect. Privity is irrelevant here; Sam is not trying to tack on anyone's prior possession. D Agree to remove the protrusion.Incorrect. Sam has met all elements for adverse possession in most states.

In most jurisdictions, which of the following is not a correct statement about gifts causa mortis?

A A gift causa mortis must be made in contemplation of imminent death.Incorrect. This is a correct statement regarding gifts causa mortis. B A gift causa mortis may be made for both real and personal property.Correct. This is an incorrect statement. A gift causa mortis is a gift of personal property made by a living person in contemplation of death. A gift causa mortis cannot be a gift of real property. C A gift causa mortis becomes irrevocable upon the donor's death.Incorrect. This is a correct statement regarding gifts causa mortis. D A gift causa mortis is automatically revoked if the donor recovers.Incorrect. This is a correct statement regarding gifts causa mortis.

While sitting in his backyard, Noah noticed a squirrel (whose nest was in a neighboring yard) slowly moving over his grass looking for nuts. Suddenly, a stranger, Ava, jumped over Noah's fence, grabbed the squirrel, and ran away. If Noah sues Ava to recover the squirrel, who will win?

A Ava, because she was the first to gain certain control of the squirrel.Incorrect. Although Ava was the first to gain certain control of the squirrel, she trespassed on Noah's land. The law abhors trespassers. Consequently, it "constructs" possession on Noah's behalf in order to discourage trespassing. Noah is deemed to be in constructive possession of the squirrel while it is on his land against all trespassers. B Ava, because the squirrel did not live on Noah's property.Incorrect. Whether the squirrel nested on Noah's parcel or not is irrelevant. The important fact is the location of the squirrel when it was grabbed by Ava. It was on Noah's property and therefore in Noah's constructive possession. C Noah, because Ava was a trespasser.Correct. Noah has best rights to the squirrel even though he never gained certain control. In order to discourage trespassing, the law gives Noah constructive possession of wild animals on his property against any trespasser. It is important to note that Noah did not gain any ownership rights in the squirrel while it was simply moving over his grass. However, when a trespasser grabs the wild animal, the law will protect the landowner against the unlawful acts of a trespasser. D Noah, because the rule of capture requires the animal be on unowned property.Incorrect. The rule of capture applies to both owned and unowned property. If a wild coyote wanders onto an individual's family owned farm, she gains ownership rights under the rule of capture as soon as she secures certain control over the animal. Likewise, if an individual is on unowned high seas, any fish she successfully entraps belong to her under the rule of capture.

Oren holds a fee simple absolute. Oren conveys "to City, its successors and assigns, provided that if the land is not used as a park, then to Karl and his heirs." (City is the City of New York.) This jurisdiction applies the common law Rule Against Perpetuities. Which of the following is correct?

A City has a fee simple absolute.Correct. The words of purchase "to City" denote City as the grantee of the present possessory interest. The words of limitation identify the estate as a fee simple. However, the words of limitation "provided that" create a defeasible fee. Since the defeasible fee is followed by an interest in a third party, City has a fee simple subject to an executory limitation. The words of purchase "to Karl" denote Karl as the grantee of a future interest. The words of limitation "and his heirs" identify the estate (that will exist if the future interest becomes possessory) as a fee simple. Since the defeasible fee is followed by an interest in a third party, Karl's interest is an executory interest. It is a shifting interest because Karl, a transferee, will divest City, another transferee, of the estate. Applying the common law Rule Against Perpetuities:(a) Identify the contingent interests: the rule must be tested against Karl's shifting executory interest. City's fee simple subject to an executory limitation is a present possessory interest and therefore is deemed to be vested for purposes of the rule.(b) List the lives in being: Oren and Karl are alive at the time of conveyance. Therefore, they are both lives in being. Remember, a life in being must be a real person. Thus, City, a city, is not considered a life in being.(c) Consider whether anyone can be born who might affect vesting: no children, issue, heirs, or widows are referred to in the conveyance. Karl could have a son, Ben, in 2020.(d) Kill off the lives in being and add 21 years: suppose that in 2021 Oren and Karl both die in a common disaster. The lives in being are now dead. The "RAP line" becomes 2042.(e) Ask: is there any possibility that the executory interest may vest more than 21 years after all lives in being (Oren and Karl) die (past the 2042 "RAP line")? If you said no, think more creatively. Imagine this possibility: 50 years after the deaths of Oren and Karl, City decides the land would make a much nicer parking garage than a park, and converts the land into this different use. According to the terms of the conveyance, the executory interest becomes possessory at that moment. The interest would vest in 2071, far beyond the 2042 "RAP line." RAP is violated. It is important to keep in mind that you are asking whether the future interest itself is capable of violating RAP, not whether the current holder of the interest will be alive when the interest vests. Again, the focus is on the interest and the conditions of its vesting, not on the identity of any current holders of the interest and whether they will vest in possession. The vesting of the executory interest is not dependent on Karl being alive.When a conveyance violates the Rule Against Perpetuities, it does so when it is made, not 21 years later. In the absence of reform tools, the offending interest (Karl's executory interest) is deleted. The conveyance therefore reads: Oren conveys "to City, its successors and assigns." Remember, the phrase "provided that..." is deemed to be part of the executory interest (and therefore is also penned out). City has a fee simple absolute. B City has a fee simple subject to an executory limitation.Incorrect. While initially it appears that City has a fee simple subject to an executory interest (City's defeasible fee is followed by an interest in a third party), Karl's executory interest violates the common law Rule Against Perpetuities. When Karl's interest is cancelled, City has a fee simple absolute. C Karl has an executory interest.Incorrect. Because the executory interest violates the common law Rule Against Perpetuities, it is void ab initio. Karl has nothing. D Oren retains his fee simple absolute.Incorrect. Oren retained nothing. Oren validly conveyed his fee simple to City.

Louise has carved an ornate wooden sculpture. She sells it on the open market. Mark copies the sculpture by manufacturing thousands of them and sells them without Louise's permission. Will Louise succeed on her claim for misappropriation of trade secrets?

A No, the sculpture is not a trade secret.Correct. The sculpture is not a trade secret. It does not constitute info that is not generally known and Louise has made no effort to keep it secret. B No, the sculpture is a trade secret but Mark did not misappropriate it.Incorrect. It's not a trade secret. C No, Mark did not misappropriate the trade secret if he proves he reverse engineered it.Incorrect. It's not a trade secret so his independent creation is irrelevant. D Yes, Mark misappropriated her trade secret.Incorrect. It's not a trade secret.

Justin was a renowned movie star. Because Justin's fame and high visibility made him and his home a common target of overzealous fans, Justin employed Brent as his private security guard. One day while making his daily security sweep around Justin's estate, Brent found a small paper bag in the middle of a patch of tall grass. Brent picked up the bag and discovered a gold pendant inside. Both Brent and Justin now claim the pendant. Both have stipulated that the pendant was a lost item. Who has the better property right to the pendant, Brent or Justin?

A Brent, because he found the pendant.Incorrect. While the general rule is that finders have the best property right to the found item except for prior possessors, employees in the course of their employment usually are deemed to find on behalf of their employers. Here Brent found the pendant in the course of his employment as a security guard. B Brent, because the item was lost property, not mislaid property.Incorrect. Lost items often go to the finder, but not necessarily so. There are a number of exceptions. When an employee finds an object during the course of his employment (as Brent did here), courts usually award the item to the employer on the theory that the employee's actions were performed on behalf of the employer. C Justin, because the pendant was found on his property.Incorrect. This answer needs more developed analysis. An owner of the locus in quo usually gains better rights to a mislaid item than a finder. However, lost items can go to the finder rather than the owner of the locus in quo depending on the status of the finder (invitee or trespasser), the status of the owner (absentee or residential owner), and whether the public policies of rewarding the find and encouraging honesty outweigh the expectations of the owner. D Justin, because Brent found the pendant in the course of his employment.Correct. Because Brent discovered the pendant while he was performing his daily security sweep, he made the find during the course of his employment. Therefore, most jurisdictions would award the pendant to Justin on the theory that Bent's actions were performed on behalf of Justin.

Max develops a new type of bed and sells the bed under the name MAXITRON BRAND BEDS. Can he get a trademark?

A No, because the mark uses his name in the mark.Incorrect. It is irrelevant that Max uses his name in the mark. B Yes, because the mark is arbitrary or fanciful.Correct. Arbitrary or fanciful marks receive the highest level of protection under Abercrombie. C Yes, because the mark is descriptive.Incorrect. Descriptive marks can be protected but only if it acquires secondary meaning. So this is not the best answer because it would be incomplete. D No, because the mark is descriptive.Incorrect. Descriptive marks can be protected but only if it acquires secondary meaning. So this is not the best answer because it would be incomplete.

Rhonda owned an undeveloped tract of farm land, located 200 feet from a large river. Tim's barren and undeveloped 10-acre parcel was located immediately between Rhonda's land and the river. Three years ago, Rhonda installed a pipeline across the surface of Tim's land which Rhonda uses to bring water to irrigate Rhonda's crops during the summer months. Which of the following is correct?

A Rhonda has acquired title to the land underneath the pipeline.Incorrect. Rhonda used the pipeline for only three years, which is not long enough for adverse possession in any state. B Rhonda has not acquired title to the land underneath the pipeline because her use was not continuous.Incorrect. The pipeline was always in place, so its existence was continuous, even though water flowed through it only in the summer months. C Rhonda has acquired a right to take water from the river under the riparian approach.Incorrect. This answer is incorrect because Rhonda's land does not adjoin a river. D Rhonda has acquired a right to take water from the river under the prior appropriation approach.Correct. The best answer is that Rhonda was the first person to put the water to beneficial use, here farming, and thereby obtained a right to the water under the prior appropriation approach.

Xiao, Yasir, and Zed own Greenacre as cotenants. Xiang builds a swimming pool in the backyard and incurs all the costs herself. Absent an agreement, which of the following is true in most jurisdictions?

A Yasir and Zed are obligated to pay their proportionate share of the cost because they each have an undivided right to use and possess Greenacre.Incorrect. In most jurisdictions, cotenants are not responsible for paying a proportionate share for improvements made by another cotenant, even though all cotenants have the right to use and possess all of the property (although they may be responsible for paying a proportionate share of the carrying costs, such as mortgage payments, insurance, and taxes). B Yasir and Zed are obligated to pay their proportionate share of the cost only if they are cotenants in possession.Incorrect. In most jurisdictions, cotenants are not responsible for paying a proportionate share for improvements made by another cotenant, even if they are cotenants in possession (although they may be responsible for paying a proportionate share of the carrying costs, such as mortgage payments, insurance, and taxes). C Yasir and Zed are obligated to pay their proportionate share of the cost if they refuse to let Xiao occupy Greenacre.Incorrect. While Yasir and Zed's refusal to let Xiao occupy the property would be an ouster, this does not create an obligation to pay for costs of improvements (although it would require Yasir and Zed to pay Xiao her pro rata share of the fair rental value of their occupancy). D Yasir and Zed are not obligated to pay their proportionate share of the cost; however, upon partition, Xiao will receive a credit equal to the increased market value produced by the improvement.Correct. Upon partition, a cotenant who improves property is entitled to a credit for the added property value produced by the improvement. If there is a partition in kind, the court will usually assign the improved portion of the property to the improving cotenant.

Aunt Betty loved her nephew, Louis. On Louis's twenty-first birthday, the family had a huge party, which Aunt Betty attended. Aunt Betty handed Louis a check for $2100, saying "You're my favorite nephew. This is for you. Have a great time." The next morning, Louis came home drunk. Aunt Betty was not overly understanding and immediately stopped payment on her uncashed check. If this jurisdiction applies the majority rule, which of the following statements is correct?

A Aunt Betty made an irrevocable gift of $2100 to Louis and owes him that amount.Incorrect. A check is merely an instrument authorizing the bank to pay the holder from a specified account. No gift occurs until the check is cashed. Under the facts, Louis merely gained Aunt Betty's unenforceable promise to make a gift of $2100. B Aunt Betty made a revocable gift of $2100 to Louis and therefore had the right to retract the same.Incorrect. Generally, inter vivos gifts are not revocable. Therefore, the initial statement that Aunt Betty made a revocable gift is incorrect. [Note, some jurisdictions view the gift of an engagement ring as a revocable gift.] C Louis can successfully bring an action in trover to gain the $2100.Incorrect. Louis did not cash the check and therefore no gift had yet occurred. The gift is only "completed" when Aunt Betty's bank accepts the check. D No gift occurred.Correct. The majority rule is that no gift occurs until the check is cashed because until then the donor retains dominion and control of the funds. Since Louis had not yet cashed the check, Aunt Betty simply made an unenforceable promise to make a gift.

Driving home from work, Ben noticed a chrome hubcap laying the middle of the street. Ben stopped, grabbed the hubcap, and threw it in the back bed of his pickup. Ben's route home included a dirt road along which were a number of potholes. Hitting one pothole extremely hard, the pickup bounced violently and the hubcap fell out of the truck's bed onto the dirt road. A few minutes later, Cale came upon the hubcap while on his evening jog. Cale picked up the hubcap and carried it with him until he reached home. That weekend, Ben went to the local swap meet and discovered the hubcap sitting on a vendor's (Cale's) table. When Ben asked the Cale where he got the hubcap, Cale replied that he had found it on a local dirt road. Ben immediately screamed that the hubcap was his and that he had lost it on that very road when it had fallen out of his pickup. Who has the best property right to the hubcap, Ben or Cale?

A Ben, because he was a prior possessor of the hubcapCorrect. Title is relative. All finders have better rights to items they find except for any prior possessors of the items. Both Ben and Cale were finders. Ben was a prior possessor of the hubcap and as such has better property rights in it than Cale, a later finder. B Ben, because he was not knowingly or willingly separated from the hubcap.Incorrect. Ben lost the hubcap he found. But Ben's priority over Cale is not based upon his losing the hubcap, but upon his being a prior possessor. Even if Ben had misplaced the hubcap, he still would maintain his priority. C Cale, because he was the last one to possess the hubcap and Ben was not the true owner of the hubcap.Incorrect. Neither rationale is correct. Later possessors lose to earlier possessors. In addition, Ben's priority is protected even though as finder he is a bailee, rather than the true owner. D Cale, because he had no notice of Ben's possession and Ben lost the hubcap through his own ineptitude.Incorrect. Cale's good faith status and Ben's ineptitude have no significance in the prioritization of their claims. Finders law protects the earlier possessor period.

In 1995, Mason and Adam moved in together and are still very much in love. They have one child, Bruno, who is in his last semester of law school. Mason is writing his will and wants to leave a large nature preserve that he owns (Greenacre) to his partner and son. He is very insistent that Greenacre be preserved in its current state and not be developed. He would like Adam to become the owner when he dies, but wants Bruno to automatically gain the property if the land is developed. This jurisdiction applies the common law Rule Against Perpetuities. Which of the following provisions, if placed in his will, will best forward Mason's intentions?

A "Greenacre to Adam provided that it is only used as a nature preserve, then to Bruno."Incorrect. Initially, the state of title would seem to be: Adam has a fee simple subject to an executory limitation (the future interest is held by a transferee, rather than the transferor), and Bruno has a shifting executory interest. Applying the Rule Against Perpetuities to the executory interest, it is void ab initio. Greenacre might cease be used as a nature preserve only after several hundred years, far beyond a life in being (Mason, Adam, and Bruno) plus 21 years. The condition "provided that..." is deemed a part of the void executory interest. Therefore, this grant results in Adam receiving a fee simple absolute, with no condition attached. B "Greenacre to Adam so long as it is only used as a nature preserve, then to Bruno and heirs."Incorrect. Initially, the state of title would seem to be: Adam has a fee simple subject to an executory limitation (the future interest is held by a transferee, rather than the transferor), and Bruno has a shifting executory interest. Applying the Rule Against Perpetuities to the executory interest, it is void ab initio. Greenacre might cease be used as a nature preserve only after several hundred years, far beyond a life in being (Mason, Adam, and Bruno) plus 21 years. Discarding the executory interest, Adam is left with a fee simple determinable ("so long as....") and Mason retains a possibility of reverter. This does not fulfill Mason's intention of creating a future interest in Bruno. C "Greenacre to Adam until it ceases to be used as a nature preserve, then to Bruno if living."Correct. Adam has a fee simple subject to an executory limitation (the future interest is held by a transferee, rather than the transferor), and Bruno has a shifting executory interest. Applying the Rule Against Perpetuities to the executory interest, it is valid. The executory interest will only vest if Bruno (a life in being at creation) is alive and, therefore, there is no possibility of remote vesting. The words of limitation ("until....") denote that Adam's defeasible fee will automatically end if and when Greenacre ceases to be used as a nature preserve. At that moment, Bruno's executory interest will vest in possession. D "Greenacre to Adam on the condition that if it ceases to be used as a nature preserve, then to Bruno and heirs."Incorrect. Initially, the state of title would seem to be: Adam has a fee simple subject to an executory limitation (the future interest is held by a transferee, rather than the transferor), and Bruno has a shifting executory interest. Applying the Rule Against Perpetuities to the executory interest, it is void ab initio. Greenacre might cease be used as a nature preserve only after several hundred years, far beyond a life in being (Mason, Adam, and Bruno) plus 21 years. Since the condition "on the condition that..." is deemed a part of the executory interest, it also is void. Therefore, this grant results in Adam receiving a fee simple absolute, with no condition attached.

Eli and Xi are married and still deeply in love. They have one child, Lucas, who is 25. Eli wants to convey his most prized possession, a 100 acre nature preserve (Meadowacre), to Xi and Lucas. Eli wants to maintain his dominion and control while he is alive, but upon his death he wants Xi to have a life estate in Meadowacre followed by a possessory fee simple in Lucas. This jurisdiction applies the traditional Rule Against Perpetuities. Which of the following provisions, if placed in his deed, will best forward Eli's intentions?

A "Meadowacre to Eli for life, then to Xi for life, then to my first son who reaches 30."Incorrect. The contingent remainder in "my first son who reaches 30" is void. Pursuant to the Rule Against Perpetuities, Eli could have another son three years later, all the lives in being at the time of the grant (Eli, Xi, and Lucas) could then die, and this second child would reach 30 more than 21 years later. The grant only conveyed Eli a life estate, Xi a vested remainder in life estate, and Eli a reversion. Lucas gains no interest. B "Meadowacre to Eli for life, then to Xi for life, then to my first son when he reaches 30."Correct. The conveyance grants Eli a life estate, Xi a vested remainder in life estate, and Lucas a contingent remainder in fee simple. Lucas's contingent remainder is valid under the Rule Against Perpetuities because it will vest, if at all, during Lucas's life. Lucas can only reach 30 while he is alive; he cannot reach 30 after he has died. Consequently, there is no possibility of remote vesting and the contingent remainder in Lucas is good. Note the difference between "my first son who reaches 30" and "my first son when he reaches 30." The first phrase defines a single individual who may or may not be Lucas (Lucas could die the following year, never having reached 30, and Eli could have another child who does reach 30). The second phrase only refers to Lucas since no matter what occurs no one other than Lucas can be Eli's first child. Out of the four choices, this grant will best forward Eli's intentions. C "Meadowacre to Eli for life, then to Xi for life, then to my heirs."Incorrect. The contingent remainder in Eli's heirs does not guarantee that Lucas will gain an interest. Lucas is an heir apparent. Eli's heirs will only be determined at his death - and Lucas could predecease his father. D "Meadowacre to Xi for life, then to Lucas."Incorrect. This grant fails to continue Eli's dominion and control while he is alive.

Which of the following statements is not correct?

A Ben's oral promise "to provide for Clare financially for the rest of her life" may form the basis of an enforceable palimony contract.Incorrect. This statement is correct. Courts have held that one cohabitant can have enforceable rights in the property of the other based on an oral promise. The palimony contract may be express or implied. Undertaking of a way of life in which two people commit to each other provides the consideration. B In most jurisdictions, graduate degrees and professional licenses are considered marital property.Correct. This statement is incorrect. Courts often view professional degrees and licenses as intellectual achievements that only potentially assist in the future acquisition of property. They themselves are not considered marital property. One practical basis for this view is the problem of trying to place a dollar value on the degree or license and the value of the supporting spouse's contribution. C In a modern separate property jurisdiction, property gained during marriage is separately owned by the spouse who earned it; while in a community property jurisdiction, all earnings during the marriage and all assets acquired from those earning are owned by both spouses equally.Incorrect. This is a correct statement. In a separate property jurisdiction, earnings during marriage belong to the earner; in a community property jurisdiction, earnings during marriage belong to both spouses equally. D In all jurisdictions, same-sex couples can freely enter into express contracts about how their property is to be shared.Incorrect. This is a correct statement. However, it remains unclear whether courts will extend palimony rights or rights based upon implied contracts to same-sex couples.

Obe conveyed Greenacre to "Ben so long as it is used as a school, and if such use ever ceases, then to Carl, if living." At the time of the grant, Ben is 90 years old and Carl is five years old. In a jurisdiction that applies the common law Rule Against Perpetuities, which of the following is correct?

A Carl has no interest because what would have been a contingent remainder violates the Rule Against Perpetuities.Incorrect. Carl's future interest will not violate the Rule Against Perpetuities because the interest must vest, if at all, during Carl's (a life in being's) life; the conveyance reads, "...to Carl, if living". B Carl has a contingent remainder.Incorrect. Carl's interest is an executory interest (it is a future interest in a transferee that follows a defeasible fee simple), not a contingent remainder; a remainder will not follow a vested fee simple. C Carl has no interest because what would have been an executory interest violates the Rule Against Perpetuities.Incorrect. Carl's executory interest does not violate the Rule Against Perpetuities because the interest must vest, if at all, during Carl's (a life in being's) life; the conveyance reads, "...to Carl, if living". D Carl has an executory interest.Correct. Carl has an executory interest because it is an interest in a third party that follows a fee simple defeasible. A remainder cannot follow a vested fee simple. The Rule Against Perpetuities is not violated because the interest must vest, if at all, during Carl's life; the conveyance reads "...to Carl, if living".

Mark conveys his house to "Chuck for life, then to Quinn's widow for life, then to Chuck's children living at the time of Quinn's widow's death." Chuck is 21 years old, unmarried with no children. Quinn is 84 years old, happily married to Reda who is 91 years old, with four adult children. The common law Rule Against Perpetuities applies in this jurisdiction. Who has what interest in the house?

A Chuck has a life estate; Quinn's widow has a contingent remainder in life estate; Chuck's children have a contingent remainder in fee simple; Mark has a reversion.Incorrect. The contingent remainder in Chuck's children violates the Rule Against Perpetuities and is therefore void. Quinn could marry Xi, who is not alive at the time of the grant (and therefore is not a life in being). Chuck could have a son, Zin, who is not alive at the time of the grant (and therefore is not a life in being). Thirty years after all lives in being (Mark, Chuck, Quinn, and Reda) die, Xi could die. At that moment, Zin's interest would vest—which is at a point beyond a life in being plus 21 years. This provides one instance of remote vesting; and therefore, the contingent remainder in Chuck's children is stricken from the conveyance. B Chuck has a life estate; Quinn's widow has a life estate; Chuck's children have a contingent remainder in fee simple; Mark has a reversion.Incorrect. Quinn's widow does not have a life estate (which is a present possessory interest); she has a future interest (she has a remainder in life estate). C Chuck has a life estate; Quinn's widow has a vested remainder in life estate; Chuck's children have a contingent remainder in fee simple; Mark has a reversion.Incorrect. Quinn's widow has a contingent remainder, not a vested remainder. Since Quinn is alive at the time of the grant, Quinn's widow is not ascertainable (even though Quinn is happily married to Reda, Reda could die before Quinn or Reda and Quinn could divorce - and Quinn could remarry someone else). Quinn's widow is only ascertained at this death. D Chuck has a life estate; Quinn's widow has a contingent remainder in life estate; Mark has a reversion.Correct. (i) Chuck has a life estate because of the words of limitation "for life." (ii) Quinn's widow has a contingent remainder in a life estate because she is unascertainable (Quinn is alive at the time of the grant; therefore, his widow will only be ascertained at his death). This contingent remainder does not violate the Rule Against Perpetuities because it must vest, if at all, immediately upon a life in being's (Quinn's) death. (iii) Chuck's children have a contingent remainder because they are unascertainable since Chuck has no children. This contingent remainder violates the Rule Against Perpetuities and therefore is void ab initio. Quinn could marry Xi, who is not alive at the time of the grant (and therefore is not a life in being). Chuck could have a son, Zin, who is not alive at the time of the grant (and therefore is not a life in being). Thirty years after all lives in being (Mark, Chuck, Quinn, and Reda) die, Xi could die. At that moment, Zin's interest would vest—which is at a point beyond a life in being plus 21 years. This provides one instance of remote vesting; and therefore, the contingent remainder in Chuck's children is stricken from the conveyance. (iv) Mark's reversion follows Quinn's widow's interest (since Quinn's widow's interest is a contingent remainder in life estate, it will end and possession then reverts to Mark).

Daisy and Jorge married. As a wedding present, Daisy took money from her separate property trust fund and purchased Greenacre. The deed read, "to Daisy and Jorge as tenants by the entirety." Jorge often gambled and soon needed money. He received a loan from his friend Ezra, who required a mortgage (which listed Greenacre as security for the loan). One day, coming back from a bar, Daisy negligently injured Alan when she ran a red light. Daisy was also intoxicated. Jorge has defaulted on his loan to Ezra and Daisy is being sued by Alan. Greenacre is their sole asset. Fearing the worse, Daisy and Jorge convey Greenacre "to our son Trevor and his heirs." Which of the following statements is correct in a majority of jurisdictions that recognize the tenancy by the entirety?

A Daisy and Jorge are liable to Ezra and Alan for the fraudulent transfer of Greenacre.Incorrect. Because Greenacre was held in tenancy by the entirety, it could not be attached to satisfy any creditor of an individual spouse. Greenacre would have been unavailable to Ezra or Alan whether or not Daisy and Jorge had conveyed it to Trevor. B A tenancy by the entirety did not arise because Daisy purchased Greenacre with her own separate property funds. Ezra may be able to gain Jorge's interest in Greenacre; and Alan may be able to gain Daisy's interest.Incorrect. A tenancy by the entirety arises when parties intend to create a tenancy by the entirety and the five unities (time, title, interest, possession + marriage) are met. The source of the funds is irrelevant. Here the elements of the tenancy are met and Daisy and Jorge own Greenacre as tenants by the entirety, not as tenants in common. C Alan (a tort creditor) can gain Daisy's interest in Greenacre; but Ezra (a contract creditor) cannot gain Jorge's interest.Incorrect. The majority position does not distinguish between contract creditors and tort creditors. Both Daisy and Alan's interests in the tenancy by the entirety are shielded from the individual creditors of each spouse. D Neither Ezra nor Alan has any interest in Greenacre.Correct. In most jurisdictions that recognize tenancies by the entirety, one spouse cannot unilaterally encumber his interest. Neither spouse has a separate divisible interest that can be reached by creditors or conveyed to third parties. Consequently, neither Ezra nor Alan has any interest in Greenacre.

When Bess retired, she purchased a single room school building ("Readacre") in fee simple absolute. There Bess held free reading classes for illiterate adults. Last month, Bess fell and broke her hip. Unable to continue teaching, Bess conveyed Readacre to Dee. The valid deed read, "...to Dee and heirs provided that Readacre is used for teaching reading to illiterate adults." The next week Dee cancelled all classes and converted Readacre into a commercial winery. Last month, Bess died. Her will provided that any interest she may have in Readacre passes to Lucy. This jurisdiction applies the common law Rule Against Perpetuities. Which of the following statements is correct?

A Dee has a fee simple absolute in Readacre.Incorrect. Dee has a fee simple subject to a condition subsequent, not a fee simple absolute. The breach of the condition did not change Dee's interest. B Lucy has a fee simple absolute in Readacre.Incorrect. Lucy inherited Bess's right of entry. Although traditionally a right of entry could not be assigned or devised (but only transferred through intestacy), modernly most jurisdictions allow free alienability of the interest. Once the condition was breached, Lucy could exercise her right of entry and gain present possession in fee simple absolute. But until she does this (and she has not), Dee's fee simple subject to a condition subsequent continues. C Dee has a fee simple subject to a condition subsequent in Readacre.Correct. The words of limitation ("provided that...") create a fee simple subject to a condition subsequent. Although the condition was breached, the estate does not automatically terminate (unlike a fee simple determinable). It ends only when Lucy exercises her right of entry by re-entering the property and divesting Dee of possession. Lucy has not done this and therefore Dee's defeasible fee continues despite breach of the condition. D Lucy has a vested remainder in Readacre.Incorrect. Lucy gained Bess's right of entry. This was the interest Bess retained when she conveyed a fee simple subject to a condition subsequent to Dee. When Bess died, the right of entry descended to Lucy.

At common law, a wife had dower in all land which her husband had been seized of an inheritable interest. Which of these is not a correct statement about common law dower?

A Dower attaches to all land which the husband possesses during marriage, even if the land was owned before marriage.Incorrect. This is a correct statement. A wife gains inchoate dower rights to all land her husband owns when they marry as well as all land the husband gains subsequent to their marriage. B A bona fide purchaser of the husband's fee can cut off the wife's dower interest.Correct. This is an incorrect statement. Bona fide purchasers gain no special rights. A wife's inchoate dower rights will only be extinguished if she expressly consents to their termination. C Dower only becomes possessory upon the husband's death; it is an inchoate right during the marriage.Incorrect. This is a correct statement. A wife's inchoate dower rights will vest only if she survives her husband. If she predeceases him, dower rights are extinguished. D Once dower attaches, the land cannot be conveyed free of his wife's dower interest.Incorrect. This is a correct statement. If a husband sells any land to which his wife's inchoate dower rights have attached, the purchaser gains the property with her dower rights still attached. For example, if Hal owns Greenacre during his marriage and sells Greenacre to Bee, Bee's fee is "burdened" by Hal's wife's dower rights. Upon Hal's death, his wife would have a life estate in one-third of Greenacre.

Today Tristan made the following inter vivos conveyance of his residence: "to Elizabeth and her heirs in two weeks." What interests are created by this conveyance?

A Elizabeth receives fee simple absolute and Tristan retains a remainder.Incorrect. Elizabeth does not have present possession. Her possession is delayed until some future date; therefore she has a future interest. She does not hold a present possessory estate in fee simple. Likewise Tristan retains present possession in fee simple defeasible and consequently does not have a future interest. B Elizabeth receives a fee simple defeasible and Tristan retains a possibility of reverter.Incorrect. Elizabeth does not have present possession. Her possession is delayed until some future date; therefore she has a future interest. She does not hold a present possessory estate in fee simple defeasible. Likewise Tristan retains present possession in fee simple defeasible and consequently does not have a future interest. C Elizabeth receives a vested remainder and Tristan retains a fee simple defeasible.Incorrect. A remainder must (i) be capable of becoming possessory immediately upon the termination of the prior interest, and (ii) not divest the prior estate. Here, in two weeks, Elizabeth will cut short Tristan's possession and therefore she does not have a remainder. She received an executory interest. D Elizabeth receives an executory interest and Tristan retains a fee simple subject to an executory limitation.Correct. Tristan retains the present possessory interest, labeled a fee simple subject to an executory interest since it is followed by a future interest in a transferee. Elizabeth has an executory interest because she will cut short Tristan's fee simple after two weeks (it is a springing executory interest because her possession will follow the transferor's, Tristan's).

Erik and Drake were brothers and owned a two-bedroom house as tenants in common. The house was located on a large lot containing a number of mature trees (excellent commercial timber). For the past 15 years, Erik occupied the house and paid all of the operating and maintenance expenses during that period. He built a greenhouse in the backyard at a cost of $20,000 (although it only increased the market value of the property by $15,000). Erik also cut down approximately one third of the trees on the property and sold the timber to a paper mill. Erik and Drake recently sold the house and now are arguing about how to divide the proceeds of the sale. This jurisdiction applies all existent majority rules when dealing with the rights and obligations of concurrent tenants. Which of the following is correct?

A Erik is not entitled to reimbursement for one-half of the real estate taxes.Incorrect. As a general rule, each cotenant is responsible for payment of his pro-rata share of operating expenses, such as mortgage payments, taxes, and insurance. Therefore, Drake is responsible for one-half of the real estate taxes paid. B Erik is entitled to reimbursement for the one-half of his cost of the greenhouse.Incorrect. The majority rule is that a cotenant who improves the property receives a credit equal to the increased market value produced by the improvement. Since the greenhouse only produced a net gain in value of $15,000, Erik can receive reimbursement for one-half of this $15,000, not his $20,000 investment cost. C Drake is entitled to a payment for one-half of the fair market value of Erik's occupancy.Incorrect. The majority rule is that a cotenant in exclusive possession does not owe any rent to a cotenant out of possession, absent an ouster. Erik never ousted Drake and therefore is not responsible for any rental value. D Drake is entitled to a proportionate share of the proceed from the cut timber.Correct. A cotenant may remove natural resources on the property without the consent of the other cotenants, but must account to his cotenants for a proportionate share of any profits derived from the extraction.

Oren holds a fee simple absolute. Oren conveys "to Gail and her heirs, but if the land is used as a bar, then Oren shall have the right to re-enter and retake the estate." Which of the following is correct?

A Gail has a fee simple determinable.Incorrect. The words of limitation ("but if...") create a fee simple subject to a condition subsequent. Remember, words of limitation that characterize a fee simple determinable are: so long as, while, during, and until. B Oren has an executory interest.Incorrect. An executory interest follows a fee simple subject to an executory limitation (a defeasible fee simple created in a transferee that is followed by a future interest in another transferee). Here Gail gains a fee simple subject to a condition subsequent; consequently, Oren has a right of entry. C Oren has a right of entry.Correct. The words of purchase "to Gail" denote Gail as the grantee of the present possessory interest. The words of limitation "and her heirs" create a fee simple. The additional words of limitation "but if" make the fee simple a fee simple subject to a condition subsequent, which is a defeasible fee. The estate can be cut short if the express condition is broken. The conveyance also contains words of reentry, which further manifest the grantor's intent to create a fee simple subject to a condition subsequent. Gail (or her grantee, devisee or heir) can lose the estate if the land is used as a bar. Oren, the grantor, retains a right of entry. Upon the happening of the stated event, Oren has the right to re-enter and reclaim the estate. Oren must take affirmative action and exercise this right to regain possession of the estate. Gail's estate continues until Oren exercises his right of entry. D Oren retains his fee simple absolute.Incorrect. Oren has made a valid grant to Gail and therefore no longer retains his fee simple absolute.

Hafez and Wanda are happily married. They buy a small restaurant and take title as tenants by the entirety. Hafez spends most of his time working at the restaurant while Wanda stays at home raising her son from a prior marriage. Wanda is independently wealthy. The industrial stove in the restaurant breaks down and Hafez immediately goes to the bank to borrow money to purchase a replacement. The bank requires Hafez to sign a promissory note and a mortgage, which he does. The mortgage lists the restaurant as security for the loan. Unfortunately, the restaurant does not succeed and Hafez defaults on the payments to the bank. Which of the following is true in a majority of jurisdictions that recognize tenancies by the entirety?

A Hafez and Wanda are jointly and severally liable for the loan.Incorrect. In a majority of jurisdictions that recognize tenancies by the entirety, one spouse cannot unilaterally encumber his interest. As the Sawada court stated, neither spouse has a separate divisible interest in the property that can be conveyed to third parties or reached by creditors. Since the restaurant is held in tenancy by the entirety, neither Hafez nor Wanda is liable for the loan. B The bank can only foreclose on Hafez's survivorship right in the restaurant.Incorrect. In a majority of jurisdictions that recognize tenancies by the entirety, one spouse cannot unilaterally encumber his interest. Therefore, the bank cannot foreclose on Hafez's interest. C The mortgage severed the right of survivorship; the bank can foreclose on all of Hafez's interest in the restaurant.Incorrect. The tenancy by the entirety provides a right of survivorship which is indestructible (unlike a joint tenancy). Therefore, Hafez's attempt to mortgage the restaurant would not sever the tenancy. D The bank has no interest in the restaurant.Correct. In a majority of jurisdictions that recognize tenancies by the entirety, one spouse cannot unilaterally encumber his interest. As the Sawada court stated, neither spouse has a separate divisible interest in the property that can be conveyed to third parties or reached by creditors. Therefore, Hafez could not unilaterally convey any interest to the bank.

Farr inherited Greenacre from his father. The property has been handed down through generations and has great emotional significance to Farr's family members. Farr is in a non-traditional relationship with his partner, Hal. On his deathbed, Farr devises Greenacre to "my loving partner, Hal, and his heirs as long as Hal does not sell Greenacre." What interests are created by this grant?

A Hal has a fee simple absolute.Correct. An absolute restraint on a fee simple is void. Therefore, the defeasible language is legally inoperative, and Hal is left with a fee simple absolute ("to Hal and his heirs"). B Hal has a fee simple determinable; Farr has a possibility of reverter.Incorrect. Even though the words "as long as" indicate a fee simple determinable, language that completely prevents the holder of a fee simple from transferring his interest is invalid. The restriction "as long as Hal does not sell Greenacre" is a disabling restraint and thus invalid due to public policy. C Hal has a fee simple subject to a condition subsequent; Farr has a right of entry.Incorrect. Even disregarding the fact that the devise contains an invalid prohibition on sale, there are no words of limitation to create a fee simple subject to a condition subsequent (e.g., on the condition that; provided that; but if). D Hal has a fee simple subject to an executory limitation; Farr has a springing executory interest.Incorrect. Even disregarding the fact that the devise contains an invalid prohibition on sale, a fee simple subject to an executory limitation arises only when a defeasible fee in a transferee is followed by a future interest in another transferee. Here no interest is created in any transferee other than Hal.

Hal, Sam, and Ted own Greenacre as tenants in common. Hal sells his interest to Xin without notifying either Sam or Ted. One day later, Sam dies intestate as a result of a tragic chain saw mishap, leaving Yap as his only heir. The next day, Ted dies. Ted's will leaves Ted's entire estate to Zoe. Who has what interest in Greenacre?

A Hal, Yap, and Zoe own Greenacre as tenants in common, in equal shares.Incorrect. As a tenant in common, Hal's interest is freely alienable. There is no requirement that Hal notify his other tenants in common of the sale. Xin owns Hal's interest. B Xin, Yap, and Zoe own Greenacre as tenants in common, in equal shares.Correct. Each undivided, fractional interest in a tenancy in common is alienable, devisable, and descendible. When Hal sells his interest to Xin, Xin, Sam, and Ted own Greenacre as tenants in common, in equal shares. When Sam dies, leaving Yap as his only heir, Xin, Yap, and Ted own Greenacre as tenants in common, in equal shares. And when Ted dies, leaving his entire estate to Zoe, Xin, Yap, and Zoe own Greenacre as tenants in common, in equal shares. C Xin, Yap, and Zoe own Greenacre as tenants in common, in fractional shares of ½, ¼, and ¼.Incorrect. Xin, Yap, and Zoe own Greenacre in equal shares (an undivided 1/3). The respective fractional interest of each original party was transferred to his transferee. D Yap and Zoe own Greenacre as tenants in common, in equal shares.Incorrect. Hal sold his interest to Xin and therefore Xin owns an undivided 1/3 interest in Greenacre. As a tenant in common, Hal's interest is freely alienable. There is no requirement that Hal notify the other tenants in common of the sale.

In 2000, Haru and Wyla were married. One year later, Haru purchased Greenacre, using the bonus Haru received that year from his employer. In 2005, Haru inherited Redacre and the family business located on it. In 2010, Haru's rich uncle gave him $500,000 as a birthday gift. Haru died last week and left a will that devised all his property to his mistress, Matilda. Haru and Wyla have always lived and worked in a community property state, and all of their real estate is located there. As Haru's surviving spouse, Wyla has rights to:

A Haru's $500,000 gift.Incorrect. In community property states, gifts remain the separate property of the individual spouse, even if gained during marriage. As such, Haru has the right to will the $500,000 birthday gift to Matilda. B Half of Redacre.Incorrect. In community property states, inheritances remain the separate property of the individual spouse, even if gained during marriage. Since Haru inherited Redacre, he has the right to will Redacre to Matilda. C Half of Greenacre.Correct. In community property states, both spouses own an equal and undivided share in all earnings acquired during marriage. Because Greenacre was acquired from Haru's earnings during marriage (Haru's bonus), it is a community asset and Wyla has an undivided ½ interest in the property. Haru's will controls only his separate interest in the community property assets; Matilda will receive his undivided ½ share of Greenacre. D Dower rights in Greenacre and Redacre.Incorrect. There is no dower in community property states.

Wendy and Harvey were married. Before the marriage, Harvey acquired a valuable antique car; when the wedding ceremony was over, Harvey handed Wendy a key to the car and said: "We now own it together!" After the marriage, Wendy established a successful business on the internet, while Harvey managed the family home and supervised their two children. Wendy used some of the earnings from the business to purchase a large house, taking title only in her own name, where she and Harvey lived with their children. Soon after the second child was born, Wendy's aunt died intestate, leaving Wendy an ocean-front cabin. Which of the following is not correct in a state which follows the community property approach to marital property?

A Harvey owns a one-half share of the car.Incorrect. Because Harry did not acquire the car during marriage, it was not community property. However, it appears that Harry made a valid inter vivos gift of a half-interest in the car to Wendy, which left him with a one-half interest. B Harvey owns a one-half share of the business.Incorrect. Property acquired through the labor of either spouse during the marriage is community property in which each spouse has a half-share. Because Wendy established the business during the marriage, Harvey owns a one-half interest. C Harvey owns a one-half share of the house.Incorrect. The earnings from the business were community property, so all assets purchased with those earnings are also community property, including the house. D Harvey owns a one-half share of the cabin.Correct. Property acquired by devise or descent during the marriage is the separate property of the donee spouse. Because Wendy acquired the cabin by descent, Harry has no interest in it.

Kane and Len own Greenacre as tenants in common. Kane lives on the property, while Len lives with his parents. One day, Len decides to leave his parents' house and tries to move into Greenacre. Kane refuses to allow Len to enter, claiming that he needs perfect quiet to study for law school. Kane continues to refuse to allow Len to occupy the property for the next 12 months. The fair market rental value of Greenacre is $12,000 per year. In a majority of jurisdictions:

A Kane is liable to Len for the full fair market rental value of $12,000.Incorrect. Kane's refusal to allow Len occupy Greenacre constitutes an ouster. However, Kane is only liable to Len for Len's pro rata share of the rental value of Kane's occupancy (here $6,000). B Kane is liable to Len for $6,000.Correct. Kane's refusal to allow Len occupy Greenacre constitutes an ouster. Kane is liable to Len for Len's pro rata share of the rental value of Kane's occupancy (here $6,000). C Kane is not liable to Len for any amount because as a cotenant he is entitled to the full use and enjoyment of Greenacre.Incorrect. Kane is entitled to the full use and enjoyment of Greenacre. However, because Kane prevented Len from using the property, Kane ousted Len and is liable to Len for Len's pro rata share for the rental value of Kane's occupancy. D Kane is not liable to Len for any amount because Len's occupancy would disturb Kane's use of the premises.Incorrect. All cotenants have an equal right to use and enjoy the property. While this shared right of possession can lead to conflicts about use, no cotenant may deny the other cotenants access to the property.

Seth inherited Greenacre in fee simple absolute from his uncle when Seth was a teenager. Seth became a successful entrepreneur and soon was independently wealthy. At 30, Seth married Leilani and they had two children, Elliot and Paxton. At 55, Seth purchased a small island (Floatacre); the habendum clause of the deed read, "to Seth for life, then to my son Elliot." Elliot is married to Fiona. They are incapable of having children and have agreed never to adopt. Seth recently died. His will left all his assets to his son Paxton. Seth and Leilani have always resided in a separate property jurisdiction that follows the rules of common law dower. Both Greenacre and Floatacre are located in this jurisdiction. Which of the following statements is correct?

A Leilani has a life estate in an undivided one-third of Greenacre.Correct. This is a correct statement. Dower provides the surviving spouse a life estate in one-third of all the freehold land which was possessed by the husband during the marriage and inheritable by his issue. Seth possessed Greenacre during his marriage with Leilani and his fee simple was an inheritable interest. Leilani gained inchoate dower rights immediately upon the marriage. When Seth died, her dower rights vested. She owns a life estate in an undivided one-third of Greenacre. B Fiona has a life estate in an undivided one-third of Floatacre.Incorrect. This is an incorrect statement. When Seth died, Elliot's vested remainder became possessory and Elliot gained a fee simple absolute in Floatacre. At that moment, Fiona gained inchoate dower rights in Floatacre (her husband now had possessory rights and the fee simple is an inheritable estate - note, it does not matter that Elliot and Fiona cannot have children; the focus is whether the estate itself is inheritable). C Paxton owns Greenacre in fee simple absolute, unencumbered by any dower rights.Incorrect. This is an incorrect statement. When Seth died, Leilani's inchoate dower rights in the estate vested. (Remember, even though Seth gained Greenacre before marriage, as long as he possessed the estate during the marriage, Leilani's inchoate dower rights attached.) She now holds a life estate in an undivided one-third of Greenacre. D Elliot owns Floatacre in fee simple absolute, unencumbered by any inchoate dower rights.Incorrect. This is an incorrect statement. When Seth died, Elliot's vested remainder became possessory and he gained present possession in fee simple absolute. Although Leilani had no dower rights in Floatacre (Seth never had an inheritable interest; he had a life estate), once Elliot gain a possessory interest, Fiona gained inchoate dower rights in the property. [Note, before Seth died, Elliot held a future interest; consequently, at that time, Fiona had no inchoate dower rights since Elliot did not have a possessory interest in Floatacre.]

Oren holds a fee simple absolute. Oren conveys "to Gail for life, then to Mel for life, then to Mel's firstborn child for life." Mel has no children. Which of the following is correct?

A Mel has a contingent remainder in life estate.Incorrect. Mel's future interest is a remainder because (i) it is capable of becoming possessory immediately upon the expiration of the preceding estate and (ii) it cannot divest any other interest. However, it is vested because (i) Mel is ascertainable at the time of the conveyance and (ii) there is no condition precedent to her receiving the estate. B Mel's firstborn has a vested remainder in life estate.Incorrect. Mel's firstborn has a remainder because (i) it is capable of becoming possessory immediately upon the expiration of the preceding estate and (ii) it cannot divest any other interest. However, it is a contingent remainder because, at the time of the conveyance, the firstborn child of Mel is unascertainable; the child is not yet born. C Gail has a vested remainder in life estate.Incorrect. The words of purchase "to Gail" denote Gail as the grantee of a present possessory interest, not a future interest. Gail has current possession. The words of limitation "for life" identify the estate as a life estate. D Oren has a reversion.Correct. The words of purchase "to Gail" denote Gail as the grantee of the present possessory interest. The words of limitation "for life" identify the estate as a life estate. The words of purchase "to Mel" denote Mel as the grantee of a future interest. The words of limitation "for life" identify the estate (that will exist when the future interest becomes possessory) as a life estate. Mel's future interest is a remainder because (i) it is capable of becoming possessory immediately upon the expiration of the preceding estate and (ii) it cannot divest any other interests. It is vested because (i) Mel is ascertainable at the time of the conveyance and (ii) there is no condition precedent to her receiving the estate. The words of purchase "to Mel's firstborn child" denote Mel's firstborn as the grantee of a future interest. The words of limitation "for life" identify the estate (that will exist when the future interest becomes possessory) as a life estate. Mel's firstborn's future interest is a remainder because (i) it is capable of becoming possessory immediately upon the expiration of the preceding estate and (ii) it cannot divest any other interest. It is a contingent remainder because, at the time of the conveyance, the firstborn child of Mel is unascertainable; the child is not yet born. Since Oren carved out a smaller estate (three successive life estates) from his larger estate (a fee simple), he retains a future interest known as a reversion, which will become possessory upon the expiration of the last life estate.

Fred is the owner of Forestacre and the father of twin daughters, Mia and Nada, and a son, Ben. Two years before Fred dies, Fred devises Forestacre to "Mia and Nada as joint tenants with right of survivorship." Two years after Fred dies, Mia dies in a car accident. Mia's will reads, "All my property goes to my brother, Ben." Who owns Forestacre?

A Nada is the sole owner of Forestacre.Correct. Upon Fred's death, Mia and Nada hold Forestacre as joint tenants. When Mia dies, her participation interest is removed, and Nada holds Forestacre in fee simple. At death, Mia has no interest in Forestacre to pass to Ben. B Nada and Ben own Forestacre as tenants in common.Incorrect. Because Mia and Nada held Forestacre as joint tenants, Nada automatically became the sole owner when Mia died. Mia's interest in the estate ceased upon her death; Mia had no interest to devise to Ben. C Nada and Ben own Forestacre as joint tenants.Incorrect. Because Mia and Nada held Forestacre as joint tenants, Nada automatically became the sole owner when Mia died. Mia's interest in the estate ceased upon her death; Mia had no interest to devise to Ben. D Nada owns a life estate in Forestacre, and Ben a vested remainder in fee simple.Incorrect. Joint tenants have the right of survivorship. When Mia died, Mia's interest in the estate was removed, and Nada automatically became the sole owner of Forestacre (as the fee simple absolute owner).

Wilma and Harry were married. Before the marriage, Wilma owned a valuable painting, while Harry owned nothing except his clothes. During the marriage, Harry earned a substantial salary as a professional baseball player. Harry used his earnings to purchase a single-family house, taking title only in his name, where he and Wilma lived with their four children. Wilma never worked outside of the home. Harry eventually died. His will provided, in part: "I bequeath and devise all of my property, including the house and my one-half share of Wilma's painting, to my nephew Nate." Which of the following is the most likely outcome in a state that follows the modern common law approach (aka the separate property approach) to marital property?

A Nate and Wilma will be tenants in common in the painting.Incorrect. Because Wilma acquired the painting before her marriage, it was her separate property. Harry has no right to bequeath any part of it to Nate. B Nate and Wilma will be tenants in common in the house.Correct. Harry's earnings during marriage were his separate property, and accordingly the house he bought with those earnings was his separate property. No facts suggest that he made an inter vivos gift of his interest to Wilma. Upon the death of a spouse, the surviving spouse may choose to either take under the decedent's will or receive a forced share (aka elective share) in the decedent's estate. Because Wilma receives nothing under Harry's will, the most likely outcome is that she will choose to receive a forced share in the house, equal to one-third or one-half, while Nate will receive the remaining share as a cotenant with Wilma. C Harry's estate will be subject to equitable distribution.Incorrect. Equitable distribution applies to the division of property at divorce, not death. D Wilma will disclaim the will and receive all of Harry's estate.Incorrect. As the surviving spouse, Wilma is entitled to a forced share (aka elective share) in Harry's estate. But this extends only to one-third or one-half of the estate, depending on the jurisdiction, not all of it.

Mel and Cassie are not married but have lived together for ten years. Mel gave up his career as a professional basketball player to help care for Cassie and manage the home. Mel could have made a substantial amount of money if he had continued with his career. Cassie gives Mel an allowance and pays for all his expenses. Throughout their relationship, Cassie has consistently assured Mel that he "is her deep love" and "could always count on her to take care of him." Unfortunately, Cassie has recently become enamored with a young business executive and has asked Mel to leave. In a jurisdiction applying the ruling in Roccamonte, does Mel have any rights to Cassie's property?

A No, there was no express agreement between Mel and Cassie.Incorrect. Even without an express contract, the Roccamonte court enforced an agreement that was implied from the parties' conduct. Here, Cassie's actions and oral statements are sufficient to give Mel at least some rights to Cassie's property. B No, housekeeping and sexual relations are the normal social experience of couples that live together and cannot form the basis of a contract for services rendered.Incorrect. While the older view was that contracts made in consideration of meretricious relationships were not enforceable, today most courts enforce agreements of support between unmarried individuals. C No, there was no legally recognized consideration for any promises Cassie made to Mel.Incorrect. Roccamonte held that one cohabitant can have enforceable rights in the property of the other, even without an express contract. Undertaking a way of life in which two people commit to each other provides the consideration. D Yes, a contract can be implied from oral statements made by Cassie within the context of a long marital-type relationship.Correct. Roccamonte held that one cohabitant can have enforceable rights in the property of the other, even without an express contract. Undertaking a way of life in which two people commit to each other provides the consideration.

Paul and Ivy are identical twins. Several years ago, they bought a small farm (Farmacre) and took title as joint tenants. Paul and Ivy moved onto Farmacre and slowly remodeled the farm house. Three years ago, Ivy left Farmacre, moving to a large city to begin her professional career as an accountant. Paul continued to live on Farmacre and started medical school. Ivy has recently returned, wanting to spend a six week vacation at Farmacre. Paul refuses to allow Ivy onto Farmacre, stating that he needs to be alone while studying for his medical boards. Paul told Ivy to come at any other time she desired; he would even leave for that period and give her exclusive occupancy. Ivy is insistent upon spending her vacation at Farmacre. The fair market rental value of Farmacre is $1500 per week. In a majority of jurisdictions, which statement is correct?

A Paul is not liable to Ivy for any amount because he is entitled to the full use and enjoyment of Farmacre.Incorrect. Concurrent tenants are entitled to the full use and enjoyment of the property; however, they cannot refuse other cotenants their equal rights to the full use and enjoyment of the property. B Paul is liable to Ivy for $9000.Incorrect. Ivy does not receive the full rental value of Farmacre for the six weeks she was prevented from occupying it. She only receives her pro rata share: here one-half of the $9000 (6 weeks x $1500), or $4500. C Paul is not liable to Ivy because her occupancy would have disturbed his reasonable use of the property and he was willing to grant her exclusive use for the same amount of time.Incorrect. All cotenants are entitled to the full use and enjoyment of the property. Paul's hypersensitivity does not elevate his rights. And merely because he offered her an exclusive period of residency does not mitigate what is a clear ouster.

Oren holds a fee simple absolute. Oren conveys "to Mary for life, then to Pip and his heirs so long as the land is used as a park." Which of the following is correct?

A Pip has a vested remainder in fee simple determinable.Correct. The words of purchase "to Mary" denote Mary as the grantee of the present possessory interest; thus, she receives the estate. The words of limitation "for life" identify the estate as a life estate. The words of purchase "to Pip" denote Pip as the grantee of a future interest. The words of limitation "and his heirs" identify the estate (that will exist when the future interest becomes possessory) as a fee simple. The interest is a remainder because (i) it is capable of becoming possessory immediately upon the expiration of the preceding estate and (ii) it cannot divest any other interest. It is vested because Pip is (i) ascertainable at the time of the conveyance and (ii) there is no condition precedent. However, the words "so long as" are words of limitation that create a defeasible fee. Pip has a vested remainder in fee simple determinable. B Pip has a contingent remainder in fee simple determinable.Incorrect. Pip's interest is a remainder because (i) it is capable of becoming possessory immediately upon the expiration of the preceding estate and (ii) it cannot divest any other interest. However, it is vested because Pip is (i) ascertainable at the time of the conveyance and (ii) there is no condition precedent. C Oren has a right of entry.Incorrect. Oren has a possibility of reverter. Always following a fee simple determinable will be a future interest in the transferor - a possibility of reverter. A right of entry follows a fee simple subject to a condition subsequent. D Oren has a reversion.Incorrect. Oren retained a possibility of reverter, following Pip's vested remainder in fee simple determinable.

A Harvey owns a one-half share of the car.Incorrect. Because Harry did not acquire the car during marriage, it was not community property. However, it appears that Harry made a valid inter vivos gift of a half-interest in the car to Wendy, which left him with a one-half interest. B Harvey owns a one-half share of the business.Incorrect. Property acquired through the labor of either spouse during the marriage is community property in which each spouse has a half-share. Because Wendy established the business during the marriage, Harvey owns a one-half interest. C Harvey owns a one-half share of the house.Incorrect. The earnings from the business were community property, so all assets purchased with those earnings are also community property, including the house. D Harvey owns a one-half share of the cabin.Correct. Property acquired by devise or descent during the marriage is the separate property of the donee spouse. Because Wendy acquired the cabin by descent, Harry has no interest in it.

A Ronald and Lela as joint tenants.Incorrect. Harry's inter vivos conveyance severed his joint tenancy interest and gave Matilda a tenancy in common with the remaining concurrent tenants. While Eve's death removed the joint tenancy interest she maintained with Ronald and Lela, Ronald and Lela still were concurrent tenants with Matilda. Also, note that Ronald and Lela cannot claim adverse possession because, as concurrent tenants, they cannot meet the "adverse and hostile" requirement, even though they were in exclusive possession for ten years (there was no ouster). B Matilda with an undivided ¼ interest as a tenant in common with Ronald and Lela; and Ronald and Lela with an undivided ¾ interest as tenants in common with Matilda and as joint tenants with themselves.Correct. Harry's inter vivos conveyance to Matilda severed his interest in the joint tenancy. Matilda became a tenant in common (of Harry's ¼ interest) with the other concurrent tenants because the unities of time and title were destroyed. Eve, Ronald, and Lela's joint tenancy with each other continued since their unities were not disturbed. When Eve died, her participatory interest in the joint tenancy with Ronald and Eve was removed; Ronald and Eve now hold a joint tenancy with each other. Therefore, Matilda has an undivided ¼ interest as a tenant in common with Ronald and Lela; and Ronald and Lela have an undivided ¾ tenancy in common with Matilda and an undivided a joint tenancy with themselves. C Matilda, Ronald, and John as tenants in common, each with an undivided ⅓ interest.Incorrect. When Lela mortgaged her interest to John, no severance occurred (in a lien jurisdiction, a mortgage is not viewed as an event of severance, but merely as a lien to secure repayment of a debt). Therefore, Lela maintained her joint tenancy with Ronald. D Matilda with an undivided ⅓ interest as a tenant in common with Ronald and Lela; and Ronald and Lela with an undivided ⅔ interest as tenants in common with Matilda and as joint tenants with themselves.Incorrect. Matilda succeeded to Harry's ¼ interest in Greenacre. Because of Harry's conveyance (destroying the unities of time and title), his joint tenancy interest was severed and became a tenancy in common. However, Matilda could not gain a greater participation interest than Harry had - and this was ¼.

Ten years ago, Harry, Eve, Ronald, and Lela became owners of Greenacre as joint tenants. Greenacre consisted of a four-bedroom house on a two acre lot on a hill overlooking the ocean. Harry and Eve were a married couple, as were Ronald and Lela. Harry and Eve gained their interests in Greenacre with the idea that it would be their retirement residence. They worked and lived in a neighboring state and have not visited Greenacre these past ten years. Ronald and Lela immediately took up exclusive possession of Greenacre and have remained there ever since. Five years ago, Harry conveyed all his interest in Greenacre to his mistress, Matilda. Early last year, Eve died in a car accident and Lela mortgaged her interest to her brother, John. This jurisdiction has a five year statutory period for adverse possession of land and follows the lien theory of mortgages. Who owns Greenacre?

A Ronald and Lela as joint tenants.Incorrect. Harry's inter vivos conveyance severed his joint tenancy interest and gave Matilda a tenancy in common with the remaining concurrent tenants. While Eve's death removed the joint tenancy interest she maintained with Ronald and Lela, Ronald and Lela still were concurrent tenants with Matilda. Also, note that Ronald and Lela cannot claim adverse possession because, as concurrent tenants, they cannot meet the "adverse and hostile" requirement, even though they were in exclusive possession for ten years (there was no ouster). B Matilda with an undivided ¼ interest as a tenant in common with Ronald and Lela; and Ronald and Lela with an undivided ¾ interest as tenants in common with Matilda and as joint tenants with themselves.Correct. Harry's inter vivos conveyance to Matilda severed his interest in the joint tenancy. Matilda became a tenant in common (of Harry's ¼ interest) with the other concurrent tenants because the unities of time and title were destroyed. Eve, Ronald, and Lela's joint tenancy with each other continued since their unities were not disturbed. When Eve died, her participatory interest in the joint tenancy with Ronald and Eve was removed; Ronald and Eve now hold a joint tenancy with each other. Therefore, Matilda has an undivided ¼ interest as a tenant in common with Ronald and Lela; and Ronald and Lela have an undivided ¾ tenancy in common with Matilda and an undivided a joint tenancy with themselves. C Matilda, Ronald, and John as tenants in common, each with an undivided ⅓ interest.Incorrect. When Lela mortgaged her interest to John, no severance occurred (in a lien jurisdiction, a mortgage is not viewed as an event of severance, but merely as a lien to secure repayment of a debt). Therefore, Lela maintained her joint tenancy with Ronald. D Matilda with an undivided ⅓ interest as a tenant in common with Ronald and Lela; and Ronald and Lela with an undivided ⅔ interest as tenants in common with Matilda and as joint tenants with themselves.Incorrect. Matilda succeeded to Harry's ¼ interest in Greenacre. Because of Harry's conveyance (destroying the unities of time and title), his joint tenancy interest was severed and became a tenancy in common. However, Matilda could not gain a greater participation interest than Harry had - and this was ¼.

Jayce makes the following conveyance: "I convey my fee simple absolute in Greenacre to Omar for life, then to Stella for life, then to the heirs of Omar." Which of the following statements is correct?

A Stella has a life estate.Incorrect. Stella has a future interest, not a present possessory interest. She has a vested remainder in life estate. B The heirs of Omar have a vested remainder in fee simple absolute.Incorrect. The heirs have a contingent remainder, not a vested remainder. It is contingent because the heirs are currently unascertainable - Omar is still alive. C The interest in the heirs of Omar violates the Rule Against Perpetuities.Incorrect. The contingent remainder does not violate the Rule Against Perpetuities because it will vest, if at all, within 21 years of a life in being at creation of the interest. Omar is a life in being and therefore his heirs will be ascertained immediately upon his death (no need to even wait the 21 years). At that point, the contingent remainder will vest; consequently, RAP is not violated. D Jayce has retained a reversion.Correct. Jayce retains a reversion because he has conveyed estates of lesser quantum than his initial fee. The contingent remainder in the heirs of Omar may never vest (Omar may not leave any heirs); in this event, a reversion is needed. Jayce's reversion could become possessory upon Stella's death (or upon Omar's death, if Stella dies first).

This assumes knowledge of Assignments 9 and 10. Jayce makes the following conveyance: "I convey my fee simple absolute in Greenacre to Omar for life, then to Stella for life, then to the heirs of Omar." Which of the following statements is correct?

A Stella has a life estate.Incorrect. Stella has a future interest, not a present possessory interest. She has a vested remainder in life estate. B The heirs of Omar have a vested remainder in fee simple absolute.Incorrect. The heirs have a contingent remainder, not a vested remainder. It is contingent because the heirs are currently unascertainable - Omar is still alive. C The interest in the heirs of Omar violates the Rule Against Perpetuities.Incorrect. The contingent remainder does not violate the Rule Against Perpetuities because it will vest, if at all, within 21 years of a life in being at creation of the interest. Omar is a life in being and therefore his heirs will be ascertained immediately upon his death (no need to even wait the 21 years). At that point, the contingent remainder will vest; consequently, RAP is not violated. D Jayce has retained a reversion.Correct. Jayce retains a reversion because he has conveyed estates of lesser quantum than his initial fee. The contingent remainder in the heirs of Omar may never vest (Omar may not leave any heirs); in this event, a reversion is needed. Jayce's reversion could become possessory upon Stella's death (or upon Omar's death, if Stella dies first).

Which of the following statements is not correct?

A Traditionally, a husband could not create a joint tenancy with his wife in a parcel he owned by executing a deed that read "to myself and my wife as joint tenants with right of survivorship" because the unities of time and title were missing.Incorrect. This is a true statement. Because the husband held title by a prior instrument, the unities of time and title are missing (the husband received his interest by a prior instrument at a prior time). The conveyance merely creates a tenancy in common between the spouses. To avoid this problem, the husband could convey to a third party "straw person." The straw person ( who as a result of the conveyance holds a fee simple) would then reconvey to "husband and wife as joint tenants with right of survivorship." The four unities are now present and a joint tenancy arises. B A tenancy by the entirety is different from a joint tenancy in that one spouse cannot unilaterally destroy the other's right of survivorship.Incorrect. This is a true statement. While a joint tenancy can be severed unilaterally by any joint tenant, a tenancy by the entirety can only be ended by divorce, death, or the agreement of both spouses. And unlike the joint tenancy, a tenancy by the entirety provides that the right of survivorship of one spouse is indestructible by the unilateral actions of the other spouse. C Generally, a joint tenant does not have the right to sue for partition of the property unless his interest would be diminished or harmed by continuing the concurrent tenancy.Correct. This statement is not correct. Any tenant in common or joint tenant has the right to sue for partition of the property (for any reason or for no reason at all). The law will not force these cotenants to remain in a concurrent tenancy. The partition action will end the cotenancy and distribute its assets. D The prevailing rule is that, absent an ouster, a cotenant in exclusive possession does not owe rent to a cotenant out of possession.Incorrect. This is a true statement. Each cotenant has the right to use and enjoy all of the property. Therefore, a cotenant in possession is not obtaining any greater benefit than she already has. Imposing liability merely because one cotenant has chosen to exercise his right of possession would be contrary to the notion of a concurrent tenancy. The occupying tenant is not exceeding her rights or denying the rights of her cotenants. [However, a few courts take the opposing view, and impose a rental obligation on the occupying cotenant.]

Vivian and Jesse were mother and daughter. They held title to Greenacre as joint tenants. Last year, Vivian secretly mortgaged her interest in Greenacre to Mila. A few months later, Vivian married Piper, Jesse's father. Recently, while on a trip to Brazil, Jesse died. Jesse's valid will left all of her assets to her father. Which of the following is correct?

A Under the title theory, Vivian and Piper are tenants in common in Greenacre.Correct. Under the title theory of mortgages, the mortgage is viewed as a conveyance of title to the mortgagee. Consequently, the joint tenancy is severed because the unities of time and title are destroyed. As a result, Vivian and Jesse became tenants in common. Tenancies in common are alienable, devisable, and descendible. Piper succeeded to Jesse's interest. B Under the lien theory, Vivian and Piper are tenants in common in Greenacre.Incorrect. Under the lien theory of mortgages, the mortgage is considered a lien to secure repayment of the debt. The joint tenancy continues since all the unities are preserved. When Jesse died, the joint tenancy's right of survivorship resulted in Vivian gaining a fee simple absolute in Greenacre. Jesse's interest was extinguished at her death. C Vivian and Piper are joint tenants in Greenacre.Incorrect. Vivian and Piper received their interests at different times and through different instruments. Because the unities of time and title are not present, no joint tenancy would arise. D Vivian and Piper are tenants by the entirety in Greenacre.Incorrect. Even though Vivian and Piper are husband and wife (and even if they are concurrent tenants because they are in a jurisdiction that applies the title theory of mortgages), no tenancy by the entirety is created since the unities of time and title are absent

Brianna and Lillian planned to attend the opening of a new exhibit at a local museum. Lillian drove to Brianna's to pick her up. While waiting for Brianna in Brianna's bedroom, Lillian accidently broke the necklace she was wearing. Lillian became very upset, but Brianna calmed her and said, "Don't worry my friend. I'm going to make you a gift of one of my necklaces." In which of the following situations has Brianna effectively delivered a necklace to Lillian?

Brianna and Lillian planned to attend the opening of a new exhibit at a local museum. Lillian drove to Brianna's to pick her up. While waiting for Brianna in Brianna's bedroom, Lillian accidently broke the necklace she was wearing. Lillian became very upset, but Brianna calmed her and said, "Don't worry my friend. I'm going to make you a gift of one of my necklaces." In which of the following situations has Brianna effectively delivered a necklace to Lillian?

Able owned a vacant lot located on a remote mountain lake. Each summer for many years, he pitched a tent on the lot for three months and lived there with his son Bob, enjoying the wilderness. Bob was only eight years old when this pattern began, and he grew to share his father's love of nature as time passed. During one of these trips, Able said to Bob: "I wrote my will two weeks ago. It says that this lot will go to you!" A few days after Bob reached age 21, Able was paralyzed in a car accident. When Bob told Able that he intended to camp on the lot during the coming summer, Able looked sad but said nothing. Bob spent the next six summers camping on the lot, just as he and Able had done in the past. Able then died. Bob discovered that Able's will devising the lot to him (Bob) was legally defective. Instead, under Able's last valid will, the lot was devised to the United Way, a national charity. Bob filed suit, seeking a declaratory judgment that he had obtained title to the lot by adverse possession. Assume that the adverse possession period in the jurisdiction is five years. Was Bob's possession of the lot adverse and hostile?

Yes, because the will gave Bob color of title.Incorrect. Bob occupied the lot before Able died; during this time, the will had no legal force or effect, and thus could not create color of title―which might only be relevant, in any event, in a jurisdiction following the good faith standard. More fundamentally, however, a court would most likely conclude that Able tacitly consented to Bob's use of the lot, and therefore Bob failed to prove that his possession was adverse and hostile. B Yes, if the jurisdiction uses the objective standard.Incorrect. A court would most likely conclude that Able tacitly consented to Bob's use of the lot, which prevents Bob from establishing adverse and hostile possession in any jurisdiction. C Yes, because Bob took possession in good faith.Incorrect. Bob did not have a good faith belief that he owned the lot; at most, he might have believed that he would inherit in the future, which is not enough to meet the good faith standard. More importantly, a court would most likely conclude that Able tacitly consented to Bob's use of the lot. D No, Bob's possession was not adverse and hostile.Correct. Bob was the son of Able, who owned the lot. Given the close family relationship between the two and their history of camping together on the lot, a court would most likely conclude that Able tacitly consented to Bob's use of the lot. Accordingly, Bob's use was not adverse and hostile.


Kaugnay na mga set ng pag-aaral

Business Final, BUS101 FINAL PT.2

View Set

Guided Reading Activity 2-3 "The Goals of the Nation"

View Set

Module 24: Forgetting, Memory Construction, and Improving Memory Practice Quiz

View Set

The Systems Development Life Cycle

View Set